You are on page 1of 91

www.bdniyog.

com
www.bdniyog.com
IBA Exam date
All MCQ Math & Solution
01 cvwb Dbœqb †evW© (wnmve KiwYK) 10-08-2018
02 BADC –( Computer-Operator) 11-05-2018
03 Titas Gas Field-Assistant Officer (General) 27-04-2018
04 (GTCL)-Assistant Manager (General) 20-04-2018
05 (DAE)(Officer Assistant cum computer typist) 13-04-2018

06 DAE(Store Keeper) 01-12-2017


07 BADC (Administrative Officer) 27-10-2017
08 BADC (Assistant Cashier) 11-08-2017
09 BADC (Store Keeper) 11-08-2017
10 BREB
11 BSCIC (Extension officer) 10-11-2018
12 City Bank Ltd. (Management Trainee officer) 20-07-2018

Khairul’s IBA Math Book - 1

www.bdniyog.com
cvwb Dbœqb †evW©
c‡`i bvg: wnmve KiwYK cixÿvi ZvwiL: 10-08-2018
cixÿvwU wb‡q‡Q: IBA, DU.

1. `yBwU µwgK c~Y© msL¨vi e‡M©i AšÍi 27 n‡j eo msL¨vwU KZ ? [cvwb Dbœqb †evW©: (wnmve KiwYK)-2018]
A) 11 B) 13 C) 14 D) 15 E) ‡KvbwUB bq DËi: C
mgvavb: (wjwLZ wbq‡g)
awi, cÖgvY: 14 Gi eM© = 142 = 196
eo msL¨vwU = x, myZivs †QvU msL¨vwU = x-1 14 Gi †_‡K 1 †QvU 13 Gi eM© 132 = 169
cÖkœg‡Z, myZivs e‡M©i AšÍi : 196-169 = 27|
x2-(x-1)2 = 27 Note: µwgK c~Y© msL¨v ejv eowU 14 n‡j †QvUwU 13 n‡e|
ev, x2-x2+2x-1 = 27
ev, 2x = 27+1
27  1 msL¨v؇ qi e‡ M© i cv_© K¨ 1
x = (GB jvBbwU †_‡KB kU©KvU© myÎ †jLv hvq: eo msL¨vwU= )
2 2
28 27  1 28
myZivs x ev eo msL¨vwU =  14 ( kU©Kv‡U© Kivi Rb¨ mivmwi,   14 DËi: 14)
2 2 2
msL¨v؇ qi e‡ M© i cv_© K¨ 1 msLv؇ qi e‡ M© i cv_© K¨-1
kU©KvU©: eo msL¨vwU = Ges †QvU msL¨vwU =
2 2
2. GKwU †Uwej 10% ÿwZ‡Z wewµ Kiv nj| weµqg~j¨ 51 UvKv †ewk n‡j 7% jvf nZ| †UwejwUi µqg~j¨ KZ UvKv ?
[cvwb Dbœqb †evW©: (wnmve KiwYK)-2018]
A) 100 B) 200 C) 300 D) 450 E) ‡KvbwUB bq DËi: C
mgvavb: (wjwLZ wbq‡g)
awi, µqg~j¨ = 100 UvKv|
10% ÿwZ‡Z cÖ_g weµqg~j¨ = 100-10 = 90 UvKv|
Avevi, 7% jv‡f wØZxq weµqg~j¨ = 100+7 = 107 UvKv|
`yB weµqg~‡j¨i e¨eavb =107-90 = 17UvKv| (cÖ‡kœ 51 UvKv n‡”Q 2 weµ‡qi cv_©K¨ ZvB cv_©K¨ †ei Kiv n‡jv)
`yB weµqg~‡j¨i g‡a¨ e¨eavb 17 UvKv n‡j µqg~j¨ = 100 UvKv
100
ÕÕ ÕÕ ÕÕ ÕÕ 1 ÕÕ ÕÕ ÕÕ = ÕÕ
17
100  51
 ÕÕ ÕÕ ÕÕ ÕÕ 51 ÕÕ ÕÕ ÕÕ = = 300 UvKv| DËi 300 UvKv|
17

Khairul’s IBA Math Book - 2

www.bdniyog.com
gy‡L gy‡L Kivi Rb¨:
G ai‡bi As‡Ki ‡ÿ‡Î cÖ_‡gB % Gi †gvU e¨eavb ‡ei K‡i Zv = e¨eav‡bi ‡gvU UvKv (hv cÖ‡kœ †`qv _vK‡e Zv) wj‡L
100% Gi gvb Avb‡Z nq|
17% = 51 UvKv n‡j (3 ¸Y)
51 90% 100% 107%
1% = (gy‡L gy‡L 3)
17 1g weµqg~j¨ µqg~j¨ 2q weµqg~j¨
51  100
100%= = 300 UvKv
17 `yB weµqg~‡j¨i e¨veavb 107-90=17%
Avevi UvKvq e¨eavb cÖkœvbyhvqx GB 17% = 51 UvKv|

3. RvwKi, Rwm‡gi †P‡q hZ eQ‡ii †QvU, ewk‡ii ‡_‡K wVK ZZ eQ‡ii eo| Rwmg I ewk‡ii eq‡mi mgwó 56 eQi n‡j,
RvwK‡ii eqm KZ ? [cvwb Dbœqb †evW©: (wnmve KiwYK)-2018]
A) 20 B) 28 C) 32 D) 36 E) ‡KvbwUB bq DËi: B
mgvavb:
GB cÖkœwU g~jZ gvbwmK `ÿZv As‡ki|
Rwmg hZ †QvU RvwKi ZZ eo ewki
cv‡ki wPÎwU‡Z RvwKi n‡”Q Rwmg I
ewk‡ii wVK gvSLv‡b| Zvn‡j Rwmg I
ewk‡ii eq‡mi Mo †ei Ki‡j ga¨we›`y †ei 56
n‡e hv RvwK‡ii eqm| A_©vs RvwK‡ii eqm
= 56 2 = 28 eQi|
wjwLZ mgvavb: (hviv wjwLZ mgvavb bv †`‡L Qvo‡eb bv Zv‡`i Rb¨| )
GLv‡b,
Rwmg-RvwKi = RvwKi-ewki (KviY cÖ_g `yR‡bi eq‡mi e¨eavb hZ c‡ii `yR‡bi e¨eavb I ZZ)
ev, Rwmg+ewki = RvwKi + RvwKi
ev, 56 = 2 RvwKi ( †h‡nZz Rwmg I ewk‡ii eq‡mi mgwó = 56 eQi)
ev, 2 RvwKi = 56
myZivs RvwKi = 56  2 = 28 eQi|

4. `yBwU msL¨v Z…Zxq GKwU msL¨v †_‡K h_vµ‡g 40% I 25% Kg| cÖ_g msL¨vwU wØZxq msL¨vwUi Zzjbvq kZKiv KZ
†QvU ? [cvwb Dbœqb †evW©: (wnmve KiwYK)-2018]
A) 15% B) 18% C) 20% D) 25% E) ‡KvbwUB bq DËi: C
mgvavb:
awi, Z…Zxq msL¨vwU = 100
myZivs 1g msL¨vwU 100-40 = 60, Ges 2q msL¨vwU 100-25 = 75
cÖ_g msL¨vwU †QvU – 75-60 = 15
15  100
kZKiv ‡QvU = = 20%
75

5. GKwU e„‡Ëi e¨mva© kZKiv 10% evov‡bv n‡j Gi †ÿÎdj kZKiv KZ e„w× cv‡e ? [cvwb Dbœqb †evW©: (wnmve KiwYK)-
2018]
A) 10% B) 16.5% C) 21% D) 25% E) ‡KvbwUB bq DËi: C
Khairul’s IBA Math Book - 3

www.bdniyog.com
mgvavb:
awi,
cÖ_‡g †QvU e„‡Ëi evmva© = 100 gy‡L gy‡L: e„‡Ëi e¨vm ev e¨vmva© Kgv ev evovi K_v ejv n‡j
Zv e‡M©i gZ nq| A_©vr 1 evi evov A_©B 2 evi evov| Zvn‡j
myZivs †QvU e„ËwUi †ÿÎdj = (100)2 = 10000 GLv‡b cÖ_‡g 10% evo‡j 100 †_‡K n‡e 100+10 = 110 Ges
10% e„w×i ci bZzb e¨vmva© = 100+10 = 110 cieZ©x‡Z 110 Gi Dci 10% evo‡j n‡e 110+(110 Gi 10%)
bZzb e„‡Ëi †ÿÎdj = (110) = 12100
2 = 110+11 = 121 | †gv‡Ui Dci evo‡jv 121-100 = 21% |

myZivs ‡ÿÎdj e„w× †c‡q‡Q = = 12100 - 10000 = 2100


2100   100
e„w×i, kZKiv nvi = = 21%
10000 
6. Lv‡qi Zvi Av‡qi kZKiv 60% LiP K‡i| Zvi Avq 32% e„w× cvIqv‡Z ‡m Zvi LiP AviI 20% evwo‡q w`j| G‡Z
Zvi mÂq kZKiv KZ e„w× cv‡e ev Kg‡e ? [cvwb Dbœqb †evW©: (wnmve KiwYK)-2018]
A) 32% e„w× cv‡e B) 50% e„w× cv‡e C) 32% Kg‡e D) 50% Kg‡e E) ‡KvbwUB bq DËi: B
mgvavb:
awi, Lv‡q‡ii eZ©gvb Avq = 100 UvKv, myZivs e¨q = 60 UvKv Ges mÂq = 100-60 = 40 UvKv|
32% e„wׇZ bZzb Avq = 100+32 = 132| bZzb e¨q = 60 + (60 Gi 20%) = 60+12 = 72 UvKv|
Zvn‡j bZzb mÂq = 132-72 = 60 UvKv| Zvi mÂq evo‡jv 60-40 = 20 UvKv
20  100
mÂq e„w×i kZKiv nvi = = 50% (40 UvKvq 20 UvKv evo‡j 100 †Z evo‡j 50UvKv ev 50%)
40
7. wcZv I cy‡Îi eZ©gvb eq‡mi AbycvZ 3:1| wZb eQi Av‡M Zv‡`i eq‡mi AbycvZ wQj 7:2| wZb eQi ci Zv‡`i eq‡mi
AbycvZ KZ n‡e ? [cvwb Dbœqb †evW©: (wnmve KiwYK)-2018]
A) 7:3 B) 5:2 C) 9:5 D) 8:3 E) ‡KvbwUB bq DËi: D
mgvavb:
awi, wcZv I cy‡Îi eZ©gvb eq‡mi AbycvZ = 3x Ges x
cÖkœg‡Z,
3x - 3 7
 (Dfq cv‡k 3 eQi Av‡Mi eq‡mi AbycvZ)
x -3 2
ev, 7x-21 = 6x – 6
ev, 7x- 6x = 21- 6 myZivs x = 15
myZivs Zv‡`i eZ©gvb eqm: = 315 = 45 eQi Ges 15 eQi|
3 eQi ci Zv‡`i eq‡mi AbycvZ n‡e (45+3) : (15+3) = 48:18 = 8:3 DËi: 8:3
8. hw` n GKwU †Rvo msL¨v nq Z‡e wb‡Pi †KvbwU †Rvo msL¨v n‡Z cvi‡e bv ? [cvwb Dbœqb †evW©: (wnmve KiwYK)-2018]
A) n2 B) 5(n-1)+1 C) 2n+2 D) 7n+3 E) ‡KvbwUB bq DËi: D
mgvavb:
awi, n = 2
Zvn‡j: A) n2 = 22=4 = †Rvo| ( jwRKvwj: ‡h †Kvb ‡Rvo msL¨vi eM© †Rvo B n‡e)
B) 5(n-1)+1 = 5 (2-1)+1 = 51+1 = 6 = †Rvo| (‡Rvo †_‡K 1 we‡qvM Ki‡j we‡Rvo Ges 5 we‡Rv‡oi mv‡_
¸Y Ki‡j ¸Ydj I we‡Rvo †k‡l we‡Rv‡oi mv‡_ 1 †hvM Ki‡j †Rvo n‡e| )
C) 2n+2 = 22+2 = 4+2 = 6 = †Rvo| (2 w`‡q †h †Kvb msL¨v‡K ¸Y Ki‡j †Rvo Ges 2 †hvM Ki‡jI †Rvo)
D) 7n+3 = 72+3 = 14+3 = 17 = we‡Rvo| (7 we‡Rvo msL¨vi mv‡_ †h †Kvb †Rvo msL¨v n ¸Y Ki‡j ¸YdjwU
Aek¨B †Rvo n‡q hv‡e Ges †mB †Rv‡oi mv‡_ 3 †hvM Ki‡j †hvMdj we‡Rvo n‡e| ) GUvB DËi: D

Khairul’s IBA Math Book - 4

www.bdniyog.com
9. 60 wjUv‡ii GKwU wgkª‡Y `ya I cvwbi AbycvZ 2:1 | AbycvZ 1:2 Ki‡Z KZ wjUvi cvwb †gkv‡Z n‡e ? [cvwb Dbœqb
†evW©: (wnmve KiwYK)-2018]
A) 20 B)30 C)50 D)60 E) ‡KvbwUB bq DËi: D
mgvavb: ( wjwLZ wbq‡g)
cÖ`Ë Abycv‡Zi †hvMdj = 2+1 = 3
2 1
`y‡ai cwigvb: = 60 Gi = 40 Ges cvwbi cwigvY = 60 Gi = 20
3 3
awi,
gy‡L gy‡L Kivi Rb¨:
cvwb †gkv‡Z n‡e, = x wjUvi|
1+2 = 3 Ask = 60 n‡j 2 Ask = 40 Ges 1 Ask = 20
cÖkœg‡Z,
GLb cvwbi 20 wjUv‡ii mv‡_ wKQz †hvM n‡e wKš‘ `ya Av‡Mi
40 1
= (x ‡KwR †hvM K‡i `ycv‡k bZzb AbycvZ) 40 c‡iI 40 B _vK‡e| ZvB GB 40 B 2q Abycv‡Z n‡q
20  x 2 †M‡j 1:2 G 1 Ask| GLb 1 Ask = 40 n‡j cvwbi 2 Ask =
ev, 20+x = 80 80 n‡Z n‡e| Av‡M †_‡K 20 wjUvi cvwb _vKvq bZzb K‡i
myZivs x = 60 wjUvi| †hvM Ki‡Z n‡e 80-20 = 60 †KwR|

3 1
10. , Gi kZKiv KZ n‡e ? [cvwb Dbœqb †evW©: (wnmve KiwYK)-2018]
4 2
A) 150% B) 110% C) 300% D) 400% E) †KvbwUB bq DËi: A
mgvavb:
Dci fMœvskwU †`L‡j A‡b‡Ki AsKUv ïiæ Ki‡Z mgm¨v n‡e| Zvn‡j Gfv‡e hw` fvev hvq, 5 msL¨vwU 10 Gi kZKiv
5  100
KZ? Aek¨B A‡a©K ev 50% | wKfv‡e? kZKiv nvi †ei Kivi wbq‡g, Gfv‡e, = 50% (hvi mv‡_ Zzjbv
10
Kiv nq Zv‡K wb‡P Ges hv‡K Zzjbv Kiv nq ‡mB msL¨vwU‡K Dc‡i)
3
 100
GLb cÖ‡kœ cÖ`Ë fMœvskØq kZKivi wbq‡g mvRv‡j, 4 = 752 = 150%
1
2

11. †Kvb msL¨vi `yB-Z„Zxqvsk H msL¨vi †P‡q 50 Kg n‡j msL¨vwU KZ ? [cvwb Dbœqb †evW©: (wnmve KiwYK)-2018]
A) 208 B) 350 C) 250 D) 150 E) ‡KvbwUB bq DËi: D
mgvavb: (wjwLZ wbq‡g)
awi, msL¨vwU = x  kU©KvU©: 10 †m‡K‡Û mgvav‡bi Rb¨ Gfv‡e fveyb
cÖkœg‡Z, msL¨vwU‡K †gvU 3 fvM Ki‡j ( ni‡K †gvU a‡i)
‡mLvb †_‡K 2 fvM ev` w`‡j _vK‡e Avi 1 fvM| GLb 1 fvM
2x 3x  2x
x- =50 ev, =50 x = 150 = 50 n‡j 3 fvM = 503 = 150 |
3 3

12. 7 Rb †jvK 7 w`‡b 7 wU †Uwej ˆZwi K‡i | 5Rb †jv‡Ki 5 wU †Uwej ˆZwi Ki‡Z Kqw`b jvM‡e ? [cvwb Dbœqb †evW©:
(wnmve KiwYK)-2018]
A) 1 B) 5 C) 7 D) 35 E) ‡KvbwUB bq DËi: C

Khairul’s IBA Math Book - 5

www.bdniyog.com
mgvavb: ( wjwLZ wbq‡g)
7 Rb ‡jvK‡K 7 wU †Uwej evbv‡Z mgq jv‡M = 7 w`b| (w`b †ei Ki‡Z n‡e ZvB w`b †k‡l)
7  7 Rb
1 ÕÕ ÕÕ 1 ÕÕ ÕÕ ÕÕ ÕÕ ÕÕ = (‡jvK Kg‡j †ewk w`b ZvB ¸Y, Kg †Uwej n‡j Kg mgq ZvB fvB)
7 wU
7 7  5
5 ÕÕ ÕÕ 5ÕÕ ÕÕ ÕÕ ÕÕ ÕÕ = [‡ewk †jvK = Kg mgq (fvM) Ges ‡ewk †Uwej =‡ewk mgq(¸Y)]
7 5
= 7 w`b|
 ev¯Í‡e fve‡j: 5 Rb †jv‡Ki 5wU ‡Uwej evbv‡bvi Rb¨ 5w`b jv‡M| GLb †mLv‡b 7-5 = 2wU ‡Uwej evwo‡q †`qvq †ewk
w`b jvMvi K_v wKš‘ GB `ywU AwZwi³ ‡Uwe‡ji Rb¨ †h‡nZz 2Rb AwZwi³ †jvK cÖ`vb Kiv n‡q‡Q ZvB Av‡M hZ w`b jvMZ
GLb ZZ w`bB jvM‡e|
GB ai‡bi Confusing Question ¸‡jv gvbwmK `ÿZv As‡k cÖPzi Av‡m|
‡`‡L g‡b n‡e GK DËi wKš‘ Avm‡jB DËiwU GKUz wfbœ| ZvB Av‡M eySzb|
13. QqwU msL¨vi Mo 6 | hw` cÖwZwU msL¨v †_‡K 2 we‡qvM Kiv nq Z‡e bZzb msL¨v¸‡jvi Mo KZ n‡e ? [cvwb Dbœqb †evW©:
(wnmve KiwYK)-2018]
A) 2 B) 4 C) 5 D) 6 E) ‡KvbwUB bq DËi: B
mgvavb: (wjwLZ wbq‡g)
6wU msL¨vi Mo =6 n‡j 6wU msL¨vi mgwó = 66 = 36  kU©KvU©: 5 †m‡K‡Û mgvav‡bi Rb¨ Gfv‡e fveyb
cÖwZwU msL¨v †_‡K 2 we‡qvM Ki‡j †gvU Kg‡e = 62 = 12 cÖwZwU msL¨v †_‡K 2 we‡qvM Ki‡j †h‡nZz cÖwZwU msL¨vB
6wU msL¨vi eZ©gvb mgwó = 36-12 = 24 2 K‡i K‡g hv‡e ZvB Zv‡`i Mo I 2 Kg‡e|
myZivs bZzb Mo n‡e 6-2 = 4|
myZivs msL¨v 6wUi bZzb Mo = 246 = 4|
14. 40 dzU j¤^v GKwU euvk Ggbfv‡e †K‡U `yfv‡M Kiv nj †hb †QvU Ask eo As‡ki 2/3 fvM nq | †QvU As‡ki ˆ`N©¨
KZ d~U ? [cvwb Dbœqb †evW©: (wnmve KiwYK)-2018]
A) 8 B) 16 C) 24 D) 28 E) ‡KvbwUB bq DËi: B
mgvavb:
awi, euvkwUi eo AskwU = 3x Ges †QvU AskwU 2x (fMœvsk aivi †_‡K Gfv‡e c~Y© msL¨v ai‡j mnR n‡e|)
cÖkœg‡Z,
3x+2x = 40 (`yÕ As‡ki †hvMdj = 40 dzU|)
5x = 40 x = 8 myZivs †QvU AskwU = 28 = 16 dzU| DËi: 16 dzU|
15. GKwU †Zjc~Y© cv‡Îi IRb 30 †KwR | A‡a©K †Zj mn cv‡Îi IRb 20 †KwR n‡j cvÎwUi IRb KZ †KwR ? [cvwb
Dbœqb †evW©: (wnmve KiwYK)-2018]
A) 5 B) 10 C) 12 D) 15 E) ‡KvbwUB bq DËi: B
mgvavb: (wjwLZ wbq‡g)
wPÎwU †`Lyb: ev‡g †Zj c~Y©
awi, cvÎwUi IRb: = x †KwR| Ges Wv‡b A‡a©K c~Y©
cÖkœg‡Z,
cvÎ mn 30 ‡KwR

cvÎ mn 20 ‡KwR

30-x = 2 (20-x) ( ev‡g, †gvU IRb - cvÎ = ‡gvU †Zj | 10 †KwR Lvwj
Avevi Wv‡b †gvU IRb †_‡K cvÎ ev` w`‡j hv _v‡K Zv †gvU
†Z‡ji A‡a©K, ZvB 2 w`‡q ¸Y Ki‡j ev‡gi †gvU †Z‡ji mgvb n‡e ) 10 †KwR 10 †KwR

ev, 30-x = 40-2x


x = 10 myZivs cvÎwUi IRb| = 10 †KwR| `ywU wPÎ Zzjbv Ki‡j †evSv hvq †Zj ev‡`
cv‡Îi IRb n‡e 30-20 = 10 †KwR|

Khairul’s IBA Math Book - 6

www.bdniyog.com
kU©KvU©: gy‡L gy‡L Kivi Rb¨: 2q wbqg: A‡a©‡Ki IRb †h‡nZz 20 Zvn‡j m¤ú~Y© Gi IRb =
1g wbqg: ‡Zj Kg‡jv 30-20 = 10 †KwR| hv ‡gvU 202 = 40‡KwR| wKš‘ †Zj wظY n‡jI cvÎwU wKš‘ wظY
†Z‡ji A‡a©K | myZivs †gvU †Zj wQj 102 = 20 n‡e bv| ZvB cv‡Îi IRb n‡”Q 40-30 = 10 †KwR|
†KwR| AZGe cv‡Îi IRb = 30-20 = 10 †KwR|
 ev¯Íe m¤§Z e¨vL¨v: A‡a©K †Zj †d‡j w`‡j cvÎwU wKš‘ A‡a©K n‡q hvq bv eis ïiæ‡Z hv wQj †k‡lI Zv B _vK‡e|
GRb¨ cÖ_‡g †h 30-20 = 10 †KwR Kg‡jv GUv m¤ú~Y© †Zj| A‡a©K †d‡j w`‡j hw` 10 †KwR †Zj K‡g Zvn‡j m¤ú~Y© †d‡j
w`‡j 20 †KwR K‡g hv‡e| A_©vr GB 20 †KwR †Zj Ges 20 †KwR †d‡j †`qvi ciI AwZwi³ 30-20 = 10 †KwR Av‡Q hv
cv‡Îi IRb|

16. GKwU †Uªb N›Uvq 180 wK‡jvwgUvi †e‡M Pj‡j cÖwZ †m‡K‡Û †UªbwU KZ wgUvi Pj‡e ? [cvwb Dbœqb †evW©: (wnmve KiwYK)-
2018]
A) 20 B) 40 C) 50 D) 60 E) ‡KvbwUB bq DËi: C
mgvavb:
1 N›Uv = 60 wgwbU ev 6060 = 3600 †m‡KÛ Ges 180 wK.wg. = 1801000 = 180000 wg (1wKwg =1000 wg)
GLv‡b, 3600 †m‡K‡Û hvq = 180 wK.wg.
180000
1 †m‡K‡Û hvq, = 50wgUvi|
3600

================================
==

Gfv‡e cÖwZwU As‡Ki we¯ÍvwiZ wbq‡g mgvav‡bi cv‡k †hŠw³K e¨vL¨vi mv‡_ mv‡_ K‡qK
†m‡K‡Û gy‡L gy‡L mgvavb Kivi ev¯Íe m¤§Z †UKwbK wkL‡Z Avgv‡`i mv‡_ _vKzb| ab¨ev`|

Khairul’s IBA Math Book - 7

www.bdniyog.com
BADC
‡cv‡÷i bvg: Awdm mnKvix Kvg Kw¤úDUvi Acv‡iUi:
cixÿvwU wb‡q‡Q: IBA, DU cixÿvi ZvwiL 11-05-2018
1. GKwU eM©‡ÿ‡Îi ˆ`N©¨ I cÖ¯’ h_vµ‡g 30% I 20% evov‡bv nj| bZzb ˆZwi AvqZ‡ÿÎwUi †ÿÎdj, g~j eM©‡ÿÎwUi
†ÿÎdj †_‡K kZKiv KZ †ewk ? [ BADC –( Computer-Operator)-2018]
a. 72% b. 60% c. 56% d. 44% Ans: c
Solution:
cÖ_‡g 30% evov‡j 100+30 = 130 nq Gici 130 Gi 20% = 26 evov‡j †gv‡U evo‡e 30+26 = 56%|
2. `yRb cyiæl I GKRb gwnjv 4 N›Uvq GKwU KvR Ki‡Z cv‡i| GKB KvR GKRb cyiæl I wZbRb gwnjv wg‡jI 4 N›Uvq
Ki‡Z cv‡i| †mB KvRwU Ki‡Z wZbRb cyiæl I PviRb gwnjvi KZ N›Uv jvM‡e ? [ BADC –( Computer-
Operator)-2018]
a. 3 b. 2.5 c. 2 d. 1 Ans:c
Solution:
(2Rb cyiæl + 1 Rb gwnjv)4 = (1 Rb cyiæl+3Rb gwnjv)  4
A_ev, 8Rb cyiæl + 4 Rb gwnjv = 4 Rb cyiæl +12 Rb gwnjv
A_ev, 4 Rb cyiæl = 8 Rb gwnjv
 1 Rb cyiæl = 2 Rb gwnj|
GLb,
cÖ_‡gi, 2 Rb cyiæl + 1 Rb gwnjvi = 4 w`b
4 Rb gwnjv + 1 Rb gwnj = 4 w`b
5 Rb gwnjv = 4 w`b ( mevB‡K gwnjv‡Z KbfvU© Kiv n‡q‡Q)
Ges †k‡li , ( †hUv †ei Ki‡Z n‡e)
3 Rb cyiæl + 4 Rb gwnjv
6Rb gwnjv + 4 Rb gwnjv
10 Rb gwnjv
5 Rb gwnjv‡K GK‡Î 4 w`b jvM‡j
10 Rb ev wظY msL¨K gwnjv‡K jvM‡e A‡a©K w`b A_©vr 2 w`b|
3. 10 wU msL¨vi †hvMdj 462| G‡`i cÖ_g 4wUi Mo 52 Ges †kl 5 wUi Mo 38 n‡j cÂg msL¨vwU KZ ? [ BADC –(
Computer-Operator)-2018]
a. 65 b. 64 c. 58 d. 57 Ans: b
Solution:
cÖ_g 4wU Ges †kl wUi †hvMdj = (452)+ (538) = 208+190 = 398
myZivs cÂg msL¨vwU = 462-398 = 64|
4. GKwU Rjvav‡ii `yB-cÂgvsk cvwb Øviv c~Y© Ges G‡Z Av‡iv 25 wjUvi cvwb Xvj‡j Gi 90% cvwbc~Y© nq| RjvaviwUi
aviYÿgZv KZ wjUvi ? [ BADC –( Computer-Operator)-2018]
a. 50 b. 100 c. 150 d. 200 Ans: a

Khairul’s IBA Math Book - 8

www.bdniyog.com
Solution:
2/5 Ask A_© = 40% GLb 25 wjUvi Xvjvi ci 90% c~Y© n‡j bZzb fv‡e c~Y© n‡jv 90-40 = 50%
myZivs 50% = 25 wjUvi n‡j aviY ÿgZvi 100% = 50 wjUvi n‡e|
5. 7% nvi mij my‡` 3000 UvKv KZ eQ‡ii Rb¨ wewb‡qvM Ki‡j †gvU 420 UvKv gybvdv cvIqv hv‡e ? [ BADC –(
Computer-Operator)-2018]
a. 5 b. 4 c. 3 d. 2 Ans:d
Solution:
3000 UvKvi 7% nv‡i GKeQ‡ii gybvdv = 307 = 210 UvKv|
Zvn‡j 420 UvKv †c‡Z mgq jvM‡e 420210 = 2 eQi|
6. hw` ÔKÕ Ges ÔLÕ Dfq †Rvo msL¨v nq, Zvn‡j wb‡Pi †KvbwU Aek¨B we‡Rvo msL¨v n‡e ? [ BADC –( Computer-
Operator)-2018]
a. K+2L b. KL+1 c. K+L d. 2K+L Ans:b
Solution:
‡Rvo msL¨vi mv‡_ †Rvo msL¨v †hvM we‡qvM ev ¸Y Ki‡j Zv †Rvo B _v‡K| ZvB KL = KL = †Rvo|
ZvB G‡`i mv‡_ we‡Rvo msL¨v †hvM Ki‡j Zv we‡Rvo n‡e|
myZivs Ackb B ‡Z cÖ`Ë KL+1 = we‡Rvo|
7. myg‡bi Kv‡Q †h UvKv Av‡Q Zv w`‡q †m 18 wU WvKwUwKU µq Ki‡Z cv‡i| hw` cªwZwU WvKwUwK‡Ui g~j¨ 4 UvKv Kg nZ
Zvn‡j †m Av‡iv `ywU WvKwUwKU †ewk µq Ki‡Z cviZ| Zvi Kv‡Q KZ UvKv Av‡Q ? [ BADC –( Computer-
Operator)-2018]
a. 180 b. 360 c. 540 d. 720 Ans: d
Solution:
awi, myg‡bi Kv‡Q _vKv UvKvi cwigvY, = K
cÖkœg‡Z, gy‡L gy‡L Kivi Rb¨ Gfv‡e fveyb:
K K 18wU Ges 18+2 = 20 wU w`‡q Ack‡bi
 =4
18 20 msL¨v¸‡jvi g‡a¨ hv‡K fvM Ki‡j fvMdj `ywUi
10K  9K cv_©K¨ 4 n‡e Zv B DËi: GLv‡b 720 a‡i wn‡me
A_ev, =4 Ki‡j wg‡j hvq ZvB GUvB DËi|
180
 K = 720
myZivs myg‡bi Kv‡Q ‡gvU 720 UvKv wQj|
cÖgvY: 720 UvKv w`‡q 18 wU wKb‡j cÖwZwUi `vg = 40 UvKv
Avevi, 720 UvKv w`‡q 20wU wKb‡j cÖwZwUi `vg = 36 UvKv | `yB `v‡gi cv_©K¨ 40-36 = 4 UvKv|
8. 12 dzU ‰`N©¨ Ges 8 dzU cÖ¯’ wewkó GKwU Kv‡c©U Øviv GKwU †g‡Si 60% †gvov‡bv hvq| †g‡SwUi AvqZb KZ eM©dyU ?
[ BADC –( Computer-Operator)-2018]
a. 96 b. 160 c. 64 d. 180 Ans: b
Solution:
60% Gi †ÿÎdj = 12 8 = 96 eM©dzU|
96 96 100
myZivs 1% =  100% = = 160 eM©dzU|
60 60

Khairul’s IBA Math Book - 9

www.bdniyog.com
9. GKwU c¨v‡K‡U 520 wU gv‡e©j Av‡Q| G‡Z Kgc‡ÿ Av‡iv KZ ¸‡jv gv‡e©j †hvM Kiv n‡j †m¸‡jv 3,4 A_ev 6 Rb
Qv‡Îi g‡a¨ mgvbfv‡e fvM K‡i †`qv hv‡e ? [ BADC –( Computer-Operator)-2018]
a. 4wU b. 6wU c. 8wU d. 12wU Ans: c
Solution:
3, 4 Ges 6 Gi j.mv.¸ = 12 | GLb 520 Gi mv‡_ KZwU †hvM Ki‡Z n‡e Zv ‡ei Kivi Rb¨ 520 †K 12
w`‡q fvM Ki‡j fvM‡kl Av‡m 4|Zvn‡j 12w`‡q wb:‡k‡l fvM Kivi Rb¨ Av‡iv gv‡e©j jvM‡e 12-4 = 8 wU|
10. GKwU Mv‡Qi D”PZv cÖwZeQi 20% e„w× cvq| hw` eZ©gv‡b MvQwUi D”PZv 1080 †m.wg. n‡q _v‡K Zvn‡j `yB eQi Av‡M
MvQwUi D”PZv KZ wQj ? [ BADC –( Computer-Operator)-2018]
a. 675 ‡m.wg b. 750 †m.wg c. 775 †m.wg d. 800 †m.wg. Ans: b
Solution:
awi, 2 eQi Av‡M D”PZv wQj = K
2 eQi ci 20% K‡i 2 evi evo‡j nq
K Gi 120% Gi 120% = 1080
100 100
K = 1080   = 750 ‡m.wg.
120 120
11. wZb eQi Av‡M GKRb wkÿK I GKRb Qv‡Îi eq‡mi AbycvZ wQj 4 : 1 Ges Zv‡`i eq‡mi ¸bdj wQj 196| 8 eQi
ci Zv‡`i eq‡mi AbycvZ KZ n‡e ? [ BADC –( Computer-Operator)-2018]
a. 13.6 b. 6.13 c. 12.5 d. 5.12 Ans: a
Solution:
awi,
3 eQi Av‡M Qv‡Îi eqm = K eQi Ges wkÿ‡Ki eqm = 4K eQi
cÖkœg‡Z,
K4K = 196
ev, 4K2 = 196
ev, K2 = 49 K = 7
myZivs 3 eQi Av‡M QvÎ I wkÿ‡Ki eqm h_vµ‡g 7 eQi Ges 47 = 28 eQi|
eZ©gvb eqm 7+3 = 10 eQi Ges 28+3 = 31 eQi|
8 eQi ci wkÿK I Qv‡Îi eq‡mi AbycvZ n‡e 31+8: 10+8 = 39:18 = 13:6 eQi|
12. 10% jeYhy³ 12 wjUv‡ii GKwU `ªeY †_‡K 2 wjUvi cvwb ev®úxf~Z Kiv n‡j, Aewkó `ªe‡Y KZ kZvsk jeY _vK‡e ? [
BADC –( Computer-Operator)-2018]
a. 6% b. 10% c. 12% d. 14.4% Ans: c
Solution:
12 wjUv‡ii g‡a¨ jeY = 12 Gi 10% = 1.2 Ges cvwb = 12-1.2 = 10.8
2wjUvi cvwb Zz‡j †bqvq eZ©gv‡b cvwb Av‡Q 10.8- 2 = 8.8
eZ©gvb †gvU `ªe‡Yi cwigvY = 12-2 = 10
myZivs 10 wjUvi `ªe‡Y jeY Av‡Q 1.2 Zvn‡j 100 †Z n‡e 12%| ( eyS‡j mivmwi GB jvBb †_‡K ïiæ)
13. ‰`wbK 9 N›U KvR K‡i 5 Rb kÖwgK 3 w`‡b 9 wU ev· evbv‡Z c‡i| ˆ`wbK 10 N›Uv KvR K‡i 8 Rb kªwgK 6 w`‡b
GKB iK‡gi KZwU ev· evbv‡Z cvi‡e ?[ BADC –( Computer-Operator)-2018]
a. 54wU b. 48wU c. 36wU d. 32wU Ans:d

Khairul’s IBA Math Book - 10

www.bdniyog.com
Solution:
9 N›Uv K‡i KvR K‡i 5 Rb kÖwgK 3 w`‡b evbv‡Z cv‡i = 9 wU e·|
9
1 ÕÕ ÕÕ ÕÕ ÕÕ 1 ÕÕ ÕÕ 1 ÕÕ ÕÕ ÕÕ = wU e·|
953
9  10  8  6
10 ÕÕ ÕÕ ÕÕ ÕÕ 8 ÕÕ ÕÕ 6 ÕÕ ÕÕ ÕÕ = wU e·| = 32wU|
95 3
mn‡R †evSvi Rb¨ e¨vL¨v:
9 N›Uvi cwie‡Z© 1 N›Uv A_©vr Kg KvR Ki‡j Kg e· ˆZix n‡e ZvB fvM, Avei 5 R‡bi cwie‡Z© 1 Rb
KvR Ki‡j Av‡iv Kg e· ˆZix n‡e ZvB Avevi fvM, Ges 6 w`‡b cwie‡Z© 1 w`b KvR Ki‡j Avevi Kg n‡e
ZvB Avevi fvM| Gfv‡e me¸‡jv Kgvi Kvi‡Y fvM n‡j †k‡li jvB‡b me¸‡jv evovi Kvi‡Y ¸Y n‡e | ZvB
mivmwi me¸‡jv msL¨v Dc‡i wj‡L KvUvKvwU|
14. GKwU †Uªb 20 wK.wg/N›Uv †e‡M Pj‡Q| GKRb e¨w³ GKB w`‡K 15 wK.wg/N›Uv ‡e‡M Pj‡Q| †UªbwU hw` e¨w³wU‡K 3
wgwb‡U AwZµg K‡i, Zvn‡j †Uª‡bi ˆ`N©¨ KZ ? [ BADC –( Computer-Operator)-2018]
a. 200 wgUvi b. 220 wgUvi c. 225 wgUvi d. 250 wgUvi Ans: d
Solution:
‡Uªb I e¨w³ GKB w`‡K Pjvq Zv‡`i AvcwÿK MwZ, 20-15 = 5 wKwg,
5 25
5 = wg./‡m‡KÛ
18 18
25
1 †m‡K‡Û hvq =
18
25
3 wgwbU ev 180 †m‡K‡Û hvq = 180 = 250 wgUvi|
18
15. `yBwU msL¨vi ¸bdj 156 Ges Zv‡`i e‡M©i †hvMdj 313| msL¨v `ywUi †hvMdj KZ ? [ BADC –( Computer-
Operator)-2018]
a. 23 b. 25 c. 26 d. 27 Ans: b
Solution:
awi, msL¨v `yw&U h_vµ‡g K Ges L
cÖkœg‡Z, KL = 156 Ges K2+L2 = 313
Avgiv Rvwb,
(K+L)2 = K2+L2+2KL
(K+L)2 = 313+2156
 (K+L)2 = 313+312
 (K+L)2 = 625
 K+L = 25
gv_v LvuUv‡Z cvi‡j:
msL¨v `ywUi ¸Ydj = 156 †K fv½v‡bv| hvq 1213 Avevi 122 = 144 Ges 132 = 169 Gi †hvMdj
144+169 = 313 myZivs †hvMdj 12+13 =25|
==================================
Khairul’s IBA Math Book - 11

www.bdniyog.com
Titas Gas Field
Post name: Assistant Officer (General) Exam date: 27-04-2018
Exam taker: IBA.DU.

1. GKwU cixÿvq cvk b¤^i 42%| Avw`e †m cixÿvq 133 b¤^i cvq Ges 35 b¤^i Kg †c‡q †dj K‡i| †gvU KZ b¤^‡ii
cixÿv n‡qwQj ? [wZZvm M¨vm wdì-mn: Awd:-2018]
A. 500 B. 400 C. 300 D. 200 E. ‡KvbwUB bq

mgvavb: (B)
133 b¤^i †c‡qI 35 b¤^‡ii Rb¨ †dj Ki‡j †gvU cvk b¤^i = 133+35 = 168
168 168  100
‡h‡nZz cvk b¤^i 42% ZvB 42% = 168, 1% = 100% = = 400 DËi: 400
42 42

2. `ywU msL¨vi AbycvZ 5 : 3 Ges G‡`i j mv ¸ 1815| cÖ_g msL¨vwU KZ ? [wZZvm M¨vm wdì-mn: Awd:-2018]
A. 55 B. 45 C. 35 D. 25 E. ‡KvbwUB bq
mgvavb: (E)
awi, msL¨v `ywU h_vµ‡g 5K Ges 3K
myZivs msL¨v `ywUi j.mv.¸ = 15K (¸Y K‡i 15K2 wb‡j fzj n‡e|)
cÖkœg‡Z,
15K = 1815 cÖgvY: cÖ_g msL¨vwU 605 Ges 2q msL¨wU 3121 = 363
1815 GLb 605 Ges 363 Gi j.mv.¸ 1815 |
K = = 121
15
myZivs cÖ_g msL¨wU 5121 = 605 DËi: E (‡KvbwUB bq|)

3. GKwU AvqZ‡ÿ‡Îi cÖ¯’ Zvi ‰`N©¨ A‡cÿv 40% Kg| hw` AvqZ‡ÿ‡Îi cÖ¯’ 36 wgUvi nq, Zvn‡j AvqZ‡ÿ‡Îi †ÿÎdj
KZ ? [wZZvm M¨vm wdì-mn: Awd:-2018]
A. 2400 eM ©wg. B. 2610 eM ©wg C. 2410eM© wg D. 2160eM© wg E. ‡KvbwUB bq
mgvavb: (D)
awi, AvqZ‡ÿÎwUi ˆ`N¨© = 100 wg. Ges cÖ¯’ 40% Kg A_©vr 60wg.
cÖkœg‡Z,
36 36 100
60% = 36, 1% = 100% = = 60 wg. myZivs ˆ`N©¨ = 60
60 60
‡ÿÎdj = ‰`N©¨ cÖ¯’ = 6036 = 2160 eM© wgUvi| DËi: 2160 eM© wgUvi|

4. iæ‡nj 25% ÿwZ‡Z GKwU Nwo weµq K‡i| hw` NwowU 350 UvKv †ewk `v‡g weµq Kiv †hZ Zvn‡j Zvi 10% jvf
nZ| NwowUi µqg~j¨ KZ wQj ? [wZZvm M¨vm wdì-mn: Awd:-2018]
A. 400 UvKv B. 500UvKv C. 1000 UvKv D. 100 UvKv E. ‡KvbwUB bq

Khairul’s IBA Math Book - 12

www.bdniyog.com
mgvavb: (C)
awi, µqg~j¨ = 100
Shortcut:
myZivs 25% ÿwZ‡Z cÖ_g weµqg~j¨ = 100-25 = 75
25% ÿwZ + 10% jvf = 35% = 350
Ges 10% jv‡f wØZxq weµqg~j¨ = 100+10 = 110 350
myZivs 1% =
`yB weµqg~‡j¨i e¨eavb = 110-75 = 35 35
350  100
100% = = 1000UvKv|
weµqg~j¨ Av‡iv 35 UvKv †ewk n‡j µqg~j¨ = 100 UvKv 35
100
 ÕÕ ÕÕ 1 ÕÕ ÕÕ ÕÕ ÕÕ = ÕÕ
35
100  350
 ÕÕ ÕÕ 350 ÕÕ ÕÕ ÕÕ ÕÕ = = 1000 UvKv| DËi: 1000 UvKv|
35
5. GKwU AvqZ‡ÿ‡Îi cÖ¯’ I cwimxgvi AbycvZ 1 : 5| AvqZ‡ÿ‡Îi ˆ`N©¨ I cÖ‡¯’i AbycvZ KZ ? [wZZvm M¨vm wdì-mn:
Awd:-2018]
A. 1 : 5 B. 5 : 1 C. 3 : 2 D. 2 : 3 E. ‡KvbwUB bq
mgvavb: (C)
awi, AvqZ‡ÿ‡Îi cÖ¯’ = K Ges cwimxgv = 5K
Avgiv Rvwb,
AvqZ‡ÿ‡Îi cwimxgv = 2 (‰`N©¨ + cÖ¯’)
 2 (‰`N©¨ + K) = 5K
5K
‰`N©¨ + K =
2
5K 5K  2K 3K
 ˆ`N©¨ = -K = =
2 2 2
3K 3K
myZivs ˆ`N©¨ I cÖ‡¯’i AbycvZ = :K = 2 : K2 = 3K : 2K = 3 : 2 DËi: 3:2
2 2
6. GKRb LyPiv we‡µZv Zvi c‡Y¨i wjwLZ g~‡j¨i Dci 10% Kwgkb †`qvq Zvi 12.5% jvf nq| wjwLZ g~‡j¨i Dci
20% Kwgkb wb‡j Zvi kZKiv KZ jvf ev ÿwZ n‡e ? [wZZvm M¨vm wdì-mn: Awd:-2018]
A. 10% ÿwZ B. 12% ÿwZ C. 12% jvf D. jvf ev ÿwZ wKQzB n‡e bv E. †KvbwUB bq
mgvavb: (D) (wjwLZ gyj¨‡K 100 a‡i mvgvavb)
awi,
wjwLZg~j¨ = 100 UvKv
10% Kwgkb w`‡q 1g weµqg~j¨ = 100-10 =90UvKv|
Avevi 12.5% jv‡f weµqg~‡j¨i nvi =112.5%
cÖkœg‡Z, 112.5% = 90 (Gi g‡a¨ µ‡qi 100% + jv‡fi 12.5% Av‡Q)
90
 1% =
112.5
90  100
100% = = 80
112.5
myZivs µqg~j¨ 100% = 80UvKv|
GLb 100UvKvi c‡Y¨ 20% Qvo w`‡j weµqg~j¨ I n‡e 80 UvKv| A_©vr 20% Qvo w`‡j jvf ev ÿwZ wKQzB n‡e bv|
Khairul’s IBA Math Book - 13

www.bdniyog.com
weKí mgvavb: (µqg~j¨‡K 100 a‡i mgvavb|)
awi µqg~j¨ 100 UvKv| ( †h‡nZz cÖ‡kœ 2wU % Av‡Q ZvB `y cvk †_‡KB aiv hvq)
myZivs 12.5% jv‡f weµqg~j¨ = 100+12.5 = 112.5 UvKv| civgk©: †h cvk †_‡KB hv‡K a‡iB wn‡me
GLb, 10% Qvo w`‡q weµgy‡j¨i % = 100-10 = 90% Kiæb bv †Kb ‡Kvb % Gi bvg wK? Ges Kvi
Zvn‡j †jLv hvq 90% = 112.5 mv‡_ †Kvb % jvMv‡j wK n‡e GB welq¸‡jv
112.5 gv_vq ivL‡Z n‡e| Zvn‡j K‡qK ‡m‡K‡Û
1% =
90 mgvavb Kiv m¤¢e n‡e|
112.5  100
100% = = 125 UvKv| A_©vr 10% Qvo †`qvi Av‡M wjwLZ gyj¨ wQj 125 UvKv|
90
GLb 125 UvKvi cY¨ 20% Qvo w`‡q bZzb weµqg~j¨ n‡e 125-125 Gi 20% = 125-25 = 100 UvKv|
Zvn‡j †`Lv hv‡”Q µqg~j¨ I weµqg~j¨ G‡ÿ‡Î mgvb n‡q hv‡”Q myZivs jvf ev ÿwZ wKQzœB n‡e bv| DËi: E

7. GKRb kÖwgK 25 w`‡b GKwU Kv‡Ri 5/16 Ask †kl Ki‡Z cv‡i| GB nv‡i KvR Ki‡j m¤ú~Y© KvR †kl Ki‡Z Zvi
AwZwi³ Avi KZ w`b jvM‡e ? [wZZvm M¨vm wdì-mn: Awd:-2018]
A. 80 w`b B. 120 w`b C. 55 w`b D. 45 w`b E. †KvbwUB bq
mgvavb: (C)
ev¯Í‡e ey‡S Ki‡j gy‡L gy‡L n‡e|
aiæb GKUv 16 Zjv wewìs‡qi 5 Zjv evbv‡Z 25 w`b ‡j‡M‡Q, Zvn‡j cÖwZ Zjv evbv‡Z mgq eiv× 255 = 5 w`b|
GLb 16 Zjvi g‡a¨ 5 Zjv evbv‡bv n‡q †M‡j Avi Aewkó 11 Zjv evb‡Z mgq jvM‡e, 115 = 55 w`b|
eB‡qi fvlvq mgvavb:
5 5 11
KvR n‡q †M‡Q Ask| myZivs KvR Aewkó& Av‡Q, 1 - Ask = Ask
16 16 16
5
Ask KvR Ki‡Z mgq jv‡M = 25 w`b|
16
16
1 ÕÕ ÕÕ ÕÕ ÕÕ ÕÕ = 25 w`b& (cÖ_gevi fMœvsk Dwë‡q ¸Y Ki‡Z nq|)
5
11 16 11
 ÕÕ ÕÕ ÕÕ ÕÕ ÕÕ = 25  w`b ( 2q evi fMœvsk †K mivmwi ¸Y Ki‡Z nq|)
16 5 16
= 55 w`b| DËi: 55 w`b|
8. 36 wU Kj‡gi µqg~j¨ K wU Kj‡gi weµq g~‡j¨i mgvb| hw` jv‡fi nvi 20% nq Zvn‡j K Gi gvb KZ ? [wZZvm
M¨vm wdì-mn: Awd:-2018]
A. 30 B. 32 C. 28 D. 24 E. ‡KvbwUB bq
mgvavb: (A)
awi, 36wU Kj‡gi cÖwZwUi µqg~j¨ = 100 UvKv| ‡gvU LiP = 36100 = 3600
myZivs 20% jv‡f cÖwZwU Kj‡gi weµqg~j¨ = 120UvKv| †gvU weµq = 120K
cÖkœg‡Z,
36100 = K120 (36wUi me©‡gvU LiP = 20% jv‡f K wUi me©‡gvU weµqgy‡j¨i mgvb)
3600 = 120K
3600
K= = 30 DËi: 30wU|
120

Khairul’s IBA Math Book - 14

www.bdniyog.com
weKí mgvavb:
awi, 36wUi µqg~j¨ = 36 UvKv| ( cÖwZwUi 1UvKv `‡i)
myZivs K wUi weµqg~j¨ I = 36UvKv ( Kvib 36wUi µqg~j¨ = K wUi weµqg~j¨)
GLb
20% jv‡f 36UvKvi Kj‡gi weµqg~j¨ = 36+36 Gi 20% = 36+7.2 = 43.2 UvKv|
43.2 UvKvq wewµ Ki‡Z n‡e = 36wU
36
 1 ÕÕ ÕÕ ÕÕ ÕÕ = wU|
43.2
36 36
 36 ÕÕ ÕÕ ÕÕ ÕÕ = = 30wU| ( 36UvKvq hZwU wewµ Ki‡Q Zvi cwigvY B n‡”Q K)
43.2
DËi: 30wU|

36 36 100
gy‡L gy‡L 10 †m‡K‡Û mgvavb: 120% = 36,  1% = 100% = = 30wU|
120 120
Gfv‡e Kivi Rb¨ wKQz e¨wmK wRwbm Rvbv _vK‡Z n‡e| †hgb: msL¨vevPK As‡Ki †ÿ‡Î memgq weµq msL¨vi
Dci jvf ÿwZ wn‡me nq | GLv‡b 20% jv‡f 100% msL¨v 30 Gi 120% =36| ZvB DËi: 30wU|

9. GKwU cixÿvq 24% QvÎQvÎx weÁv‡b AK…ZKvh© nq Ges 43% QvÎQvÎx MwY‡Z AK…ZKvh© nq| hw` 15% QvÎQvÎx Dfq
wel‡q AK…ZKvh© n‡q _v‡K, Zvn‡j kZKiv KZRb QvÎQvÎx Dfq wel‡q cvm K‡i‡Q ? [wZZvm M¨vm wdì-mn: Awd:-
2018]
A. 52% B. 67% C. 33% D. 48% E. ‡KvbwUB bq
mgvavb: (D) (cv‡ki wPÎwU †`Lyb)
weÁvb=24 MwYZ =43
Dfq wel‡q AK…ZKvh© = 15%
ïay weÁv‡b AK…ZKvh© = 24% - 15% = 9%
ïay MwY‡Z AK…ZKvh© - 43-15 = 28% 9 15 28
%
myZivs †gvU AK…ZKvh©: = 15%+9%+28% = 52% 48
‡gvU = 100
Zvn‡j Dfq wel‡q cvm K‡i‡Q = 100% - 52% = 48% DËi: 48%
10. ‡Kvb QvÎvev‡m 320 Rb Qv‡Îi 18 w`‡bi Lvevi Av‡Q| 6 w`b ci QvÎvev‡m Av‡iv 160 Rb QvÎ Avm‡j, evKx Lv`¨
Avi KZ w`b Pj‡e ? [wZZvm M¨vm wdì-mn: Awd:-2018]
A. 12 w`b B. 10 w`b C. 8 w`b D. 6 w`b E. ‡KvbwUB bq
mgvavb: (C)
w`b Aewkó 18-6 = 12 w`b|
160 Rb Avmvq ‡gvU QvÎ= 320+160 = 480
320 R‡bi Lvevi Av‡Q = 12 w`‡bi Shortcut: ey‡S †M‡j mivmwi,
12  320
1 ÕÕ ÕÕ ÕÕ = 12320 (Kg †jvK †L‡j †ewkw`b hv‡e) w`b| = 8 w`b|
480
12  320
480 ÕÕ ÕÕ ÕÕ = w`b|
480
= 8 w`b| DËi: 8w`b|

Khairul’s IBA Math Book - 15

www.bdniyog.com
11. 180 †KwR IR‡bi GKwU mv‡ii wgkª‡Y ˆRe I BDwiqv mv‡ii AbycvZ 2 : 1| Av‡iv KZ †KwR BDwiqv mvi wgkv‡j †mB
AbycvZ 1 : 2 n‡e ? [wZZvm M¨vm wdì-mn: Awd:-2018]
A. 180 †KwR B. 140 †KwR C. 120 ‡KwR D. 240 ‡KwR E. ‡KvbwUB bq
mgvavb: (A)
Abycv‡Zi †hvMdj = 2+1 = 3 eyS‡j 15 †m‡K‡Ûi †Ljv: cÖ_‡g ‰Re 180 Ges BDwiqv 60 |
2 GLb BDwiqv †gkv‡bvi ci BDwiqvi Abycv‡Zi gvb 2 Ges ˆRe Gi
‰Re Av‡Q, 180 Gi = 120 †KwR| gvb 1| †h‡nZz ˆRe ‡gkv‡bv nq wb ZvB Av‡Mi 120 B 1 Ask n‡q
3
†M‡Q| Zvn‡j 2 Ask n‡Z BDwiqv jvM‡e 1202 = 240 | Av‡M
1
BDwiqv Av‡Q, 180 Gi = 60 †KwR| ‡_‡K 60 Av‡Q, ZvB ‡gkv‡Z n‡e 240-60 = 180|
3
awi, bZzb BDwiqv †gkv‡Z n‡e = K ‡KwR| ( GiKg cÖ‡kœ †hUv †gkv‡Z n‡e bv †mUv wb‡q wn‡me Ki‡Z nq)

cÖkœg‡Z,
120:60+K = 1:2 ( `ycv‡kB bZzb AbycvZ)
120 1
ev, 
60  K 2
ev, 60+K = 240
K = 240-60 = 180 ‡KwR DËi: 180 †KwR|

12. ‰`wbK 14 N›Uv KvR K‡i 9 wU †gwkb 10 w`‡b 2800 wjUvi cvwb D‡Ëvjb Ki‡Z cv‡i| ˆ`wbK 18 N›Uv KvR K‡i 8 wU
†gwkb KZ w`‡b 3200 wjUvi cvwb D‡Ëvjb Ki‡Z cvi‡e ? [wZZvm M¨vm wdì-mn: Awd:-2018]
A. 9 w`‡b B. 10 w`‡b C. 12 w`‡b D. 16 w`‡b E. ‡KvbwUB bq

mgvavb: (B) (A‡bK eo HwKK wbq‡gi cÖkœ n‡jI jwRK wVK †i‡L Ki‡j mgq Kg jvM‡e+mwVK DËi
Avm‡e)
14 N›Uv K‡i KvR K‡i 9wU ‡gwkb w`‡q 2800 wjUvi cvwb D‡Ëvjb Ki‡Z mgq jv‡M = 10w`b ( ‡hUv Pvq Zv †k‡l)
10  14  9
 1 ÕÕ ÕÕ ÕÕ ÕÕ 1ÕÕ ÕÕ ÕÕ 1 ÕÕ ÕÕ ÕÕ ÕÕ ÕÕ ÕÕ = w`b|
2800
10  14  9  3200
 18 ÕÕ ÕÕ ÕÕ ÕÕ 8ÕÕ ÕÕ ÕÕ 3200 ÕÕ ÕÕ ÕÕ ÕÕ ÕÕ ÕÕ = w`b|
2800  18  8
= 10 w`b| DËi: 10 w`b|
e¨vL¨v: e¨vL¨vwU B Avcbvi m¤ú`| KviY e¨vL¨v c‡o †h jwRK †Wfjc n‡e Zv w`‡qB cieZ©x‡Z GiKg AsK
`ªZ Ki‡Z cvi‡eb|
cÖ_‡g, 14 N›Uv KvR bv K‡i 1 N›Uv KvR Ki‡j mgq †ewk jvM‡e, ZvB ¸Y A_©vr Dc‡i, Avevi 9wU ‡gwk‡bi e`‡j
1wU w`‡q KvR Ki‡j Av‡iv †ewk mgq jvM‡e Zvn‡j Avevi ¸Y| wKš‘ 2800 wjUvi cvwbi e`‡j 1 wjUvi cvwb Zzj‡Z Kg
mgq jvM‡e ZvB fvM|
GLb Dc‡ii ¸Y fvM †`‡L Zvi wb‡Pi msL¨v¸‡jv wecixZ cv‡k ewm‡q †hgb 14 Gi wecixZ cv‡k 18, 9 wecix‡Z
8 Ges 2800 wb‡P e‡m‡Q ZvB 3200 †K Dc‡i ewm‡q KvUvKvwU Ki‡jB DËi †ei n‡e| A_ev Avevi †f‡½ †f‡½I
fve‡Z cv‡ib| DËi: 10 w`bB Avm‡e|

Khairul’s IBA Math Book - 16

www.bdniyog.com
13. Kwe‡ii evmv BDmy‡di evmvi 4 gvBj cwð‡g| gvRnv‡ii evmv BDmy‡ci evmvi 6 gvBj Dˇi Ges ûgvqy‡bi evmvi 4
gvBj cwð‡g| Kwei Ges ûgvqy‡bi evmvi mij‰iwLK `yiZ¡ KZ ? [wZZvm M¨vm wdì-mn: Awd:-2018]
A. 14 gvBj B. 10 gvBj C. 8 gvBj D. 5 gvBj E. ‡KvbwUB bq
mgvavb: (B)
cv‡ki wPÎwU †`Lyb Ges e¨vL¨vwU co–b|
cÖ_‡g BDmyd I Kwe‡ii evmvi `~iZ¡ = 4 gvBj gvRnvi 4 gvBj
ûgvqyb
GLb BDmy‡di evmv †_‡K gvRnv‡ii evmv Dˇi = 6 DËi
6 gvBj
gvRnv‡ii evmvwU BDmy‡di Dˇi Ges GKB mv‡_ 6 gvBj cwðg c~e©
ûgvqy‡bi evmvi cwð‡g nIqvq ûgvqy‡bi evmv n‡e 4 gvBj
gvRnv‡ii evmvi c~‡e©| Kwei `wÿY
4 gvBj BDmyd (ïiæ)
8 gvBj
gvRnv‡ii evmv †_‡K ûgvqy‡bi evmvI 4 gvBj c~‡e©|
GLb Kwe‡ii evmv ‡_‡K AvovAvwo ûgvqy‡bi evoxi `~iZ¡ †ei Kivi Rb¨ wc_v‡Mviv‡mi m~Î cÖ‡qvM Ki‡Z n‡e|
`~iZ¡2 = 82+ 62
ev, `~iZ¡ = 100 = 10 gvBj|
14. †¯ªv‡Zi wecix‡Z GKwU †bŠKv 52 wgwb‡U 13 wKwg †h‡Z cv‡i| †¯ªv‡Zi †eM 4 wKwg/N›Uv| w¯’i cvwb‡Z †bŠKvi †eM KZ ?
[wZZvm M¨vm wdì-mn: Awd:-2018]
A. 19 wKwg/N›Uv B. 23 wKwg/N›Uv C. 13 wKwg/N›Uv D. 11 wKwg/N›Uv E. ‡KvbwUB bq
mgvavb: (A)
52 wgwb‡U hvq = 13 wKwg,
13
1 ÕÕ ÕÕ = ÕÕ
52
13  60
60 ÕÕ ÕÕ = =15wKwg| (MwZ‡eM †ei Ki‡Z ejvq 60 wgwb‡U KZUzKz hvq Zv †ei Kiv n‡q‡Q)
52
(Dc‡ii 3 jvB‡bi GB HwKK wbqg 5 ‡m‡K‡Û Gfv‡e Kiv hvq 52 wg = 13 wKwg A_©vr mg‡qi 4 fv‡Mi 1 fvM c_)
myZivs 60 wgwb‡Ui 4 fv‡Mi 1 fvM n‡e 15 ( G‡Z KvUvKvwU Avi wjLv wjwL Ki‡Z n‡e bv| ïay †eª‡bi †Ljv) GLb
‡¯ªv‡Zi wecix‡Z ev cÖwZK~‡j MwZ 15 Ges ‡¯ªv‡Zi MwZ 4 nIqvq w¯’i MwZ = 15+4 = 19wKwg/N›Uv|
2
15. wb‡Pi †Kvb fMœvsk †_‡K eo ? [wZZvm M¨vm wdì-mn: Awd:-2018]
3
31 9 2 11
A. B. C. D. E. ‡KvbwUB bq
50 11 5 27
mgvavb: (B)
†h‡nZz eo fMœvskwU †ei Ki‡Z ejv n‡q‡Q| ZvB †h¸‡jv †QvU n‡e †m¸‡jv ev` w`‡Z n‡e|
31 2 31
cÖ_‡g, Gi mv‡_ AvovAvwo ¸Y K‡i cvB 93  100 myZivs fMœvskwU †QvU|
50 3 50
9 2 9
GLb, Gi mv‡_ AvovAvwo ¸Y K‡i cvB 27  22 myZivs fMœvskwU eo ( GwUB DËi)
11 3 11
2
GKB fv‡e Ab¨ Ackb `ywUi mv‡_ AvovAvwo ¸Y Ki‡j †m¸‡jv †QvU nIqvq DËi: B
3

Khairul’s IBA Math Book - 17

www.bdniyog.com
16. GK e¨emvqx Zvi Avg`vwbK…Z 240 wU evB‡Ki g‡a¨ 15% evB‡K ÎæwU Ly‡R †cj| cieZ©x‡Z Avg`vwbK…Z 160 wU
evB‡Ki 5% evB‡K ÎæwU Ly‡R †cj| `yev‡i †m kZKiv KZ¸‡jv ÎæwUhy³ evBK †cj ? [wZZvm M¨vm wdì-mn: Awd:-
2018]
A. 20% B. 15% C. 11% D. 10% E. ‡KvbwUB bq
mgvavb: (C)
15
cÖ_gev‡ii ÎæwUc~Y© evBK =240 Gi 15% = 240 = 36 (gy‡L gy‡L 100 †Z 15 n‡j 200 †Z 30 Ges 10 G 1.5
100
n‡j 40 G 6 †gvU 30+6 = 36wU)
5
Avevi , wØZxq ev‡ii ÎæwUc~Y© evB‡Ki msL¨v = 160 Gi 5% = 160 = 8 (gy‡L gy‡L: 100 †Z 5 n‡j 10 G .5
100
Ges 60 G 3 myZivs 160 G 5+3 = 8)

myZivs ‡gvU 240+160 = 400 wU evB‡Ki g‡a¨ ÎæwU c~Y© evB‡Ki msL¨v = 36+8 = 44wU|
44  100
ÎæwUc~Y© evB‡Ki kZKiv nvi = =11% DËi: 11%
400
17. GKwU eM© †ÿ‡Îi cÖwZ evûi ‰`N©¨ 40% K‡i evov‡bv n‡j Gi †ÿÎdj kZKiv KZUzKy e„w× cv‡e ? [wZZvm M¨vm wdì-
mn: Awd:-2018]
A. 96% B. 80% C. 69% D. 40% E. ‡KvbwUB bq
mgvavb: (A)
eM©‡ÿ‡Îi GKevû‡K 40% evov‡j Zvi ˆ`N©¨ I cÖ¯’ Dfq B 40% K‡i †e‡o hvq|
myZivs cÖ_‡g 100 †_‡K 40 evov‡j nq 140 Gici 140 Gi 40% ev 56 evov‡j †gvU evo‡e 40+56 = 96%| D:
18. ivRy I gy³vi eq‡mi AbycvZ h_vµ‡g 5 : 4| 8 eQi ci ivRyi eqm n‡e 28 eQi| eZ©gvb gy³vi eqm KZ ? [wZZvm
M¨vm wdì-mn: Awd:-2018]
A. 8 B. 12 C. 16 D. 20 E. ‡KvbwUB bq
mgvavb: (C)
awi,
ivRy I gy³vi eqm h_vµ‡g 5K Ges 4K

cÖkœg‡Z,
5K = 28-8 ( †h‡nZz 5K n‡jv eZ©gvb ZvB 28 †_‡K 8 we‡qvM K‡i Avm‡Z n‡e|)
5K = 20
K=4
myZivs gy³vi eZ©gvb eqm = 44 = 16 eQi| DËi: 16 eQi|
19. `yBwU bj w`‡q GKwU U¨vsK h_vµ‡g 12 I 15 N›Uvq c~Y© nq| Z…Zxq GKwU bj Øviv †mwU 20 N›Uvq Lvwj nq| wZbwU bj
GKmv‡_ Lyy‡j w`‡j U¨vsKwU c~Y© KZ mgq jvM‡e ? [wZZvm M¨vm wdì-mn: Awd:-2018]
A. 12 N›Uvq B. 10 N›Uvq C. 8 N›Uvq D. 7 N›Uvq E. ‡KvbwUB bq
mgvavb: (B)
1 1
cÖ_g bj `ywU 1 N›Uvq c~Y© K‡i h_vµ‡g , Ask Ges Ask|
12 15
1
Avevi 3q bj Øviv 1 N›Uvq Lvwj nq = Ask|
20
Khairul’s IBA Math Book - 18

www.bdniyog.com
myZivs wZbwU bj GKmv‡_ Pvjy _vK‡j 1 N›Uvq|
1 1 1
  (c~Y© Ki‡j †hvM Ges Lvwj Kivq we‡qvM Kiv n‡q‡Q)
12 15 20
10  8  6 12 1
  Ask|
120 120 10
1
myZivs c~Y© nq = 1 N›Uvq  1 Ask ev m¤ú~Y© Ask c~Y© nq = 110 = 10 N›Uvq| DËi: 10 N›Uv|
10
20. GKwU _‡j‡Z 25 cqmv, 10 cqmv I 5 cqmvi gy`ªv 3 : 4 : 5 Abycv‡Z Av‡Q| hw` me¸‡jv wgwj‡q 28 UvKv nq, Zvn‡j
10 cqmvi gy`ªv KZwU ? [wZZvm M¨vm wdì-mn: Awd:-2018]
A. 80 wU B. 60 wU C. 100 wU D. 110 wU E. ‡KvbwUB bq
mgvavb: (A)
awi, 25 cqmv, 10 cqmv Ges 5 cqmvi g~`ªvi msL¨v h_vµ‡g 3K, 4K Ges 5K|
25 1
GLb, 25 cqmvi g~`ªvi gvb = ev UvKv ( †h‡nZz 28 UvKv †`qv Av‡Q, ZvB UvKv evwb‡q mgxKiY n‡e|)
100 4
10 1
10 cqmvi g~`ªvi gvb = ev UvKv
100 10
5 1
Ges 5 cqmvi g~`ªvi gvb = ev UvKv
100 20
cÖkœg‡Z,
1 1 1
(3K  ) + (4K  ) + ( 5K ) = 28 UvKv|
4 10 20
3K 2K K cÖgvY: 25 cqmvi gy`ªv = 320 = 60wU = 604 = 15UvKv
 + + = 28 UvKv|
4 5 4 10cqmvi g~`ªv = 420 = 80wU = 8010 = 8UvKv|
15K  8K  5K 5 cqmvi g~`ªv = 520 = 100wU = 10020 = 5UvKv|
 = 28 UvKv|
20 ‡gvU UvKv = 15+8+5 = 28UvKv|
28K = 2820 K = 20 D‡jø L¨; 4wU 25cqmvi g~`ªvq 1UvKv, 10wU 10cqmvi g~`ªvq 1UvKv Ges
myZivs 10 cqmvi g~`ªv = 420 = 80wU| 20wU 5 cqmvi g~`ªv 1UvKv nq weavq 4, 10 Ges 20 w`‡q fvM|
(‡Kb fMœvsk evbv‡bv n‡jv?? ey‡S wbb:)
1
aiæb, 3K = 310 ev 30wU 25cqmv wK 30 UvKv n‡e? Aek¨B bv| ZvB 30 Gi mv‡_ ¸Y Ki‡j Zv UvKv n‡e Ges
4
Gfv‡e me¸‡jv UvKvi cwigv‡Yi †hvMdj = 28 UvKv n‡e| Gfv‡e UvKvq gvb †ei bv K‡i Ki‡j fzj n‡e| )

Khairul’s IBA Math Book - 19

www.bdniyog.com
Gas Transmission Company Ltd, (GTCL)
Post Name: Assistant Manager (General) Exam Date: 20-04-2018
Exam taker : IBA, DU.
1. A 60 litre mixture of sugar and water contains sugar and water in the ratio of 2:3. How
many litre of the mixture should be replaced by sugar so that the ratio of sugar and
water becomes 1:1?(60 wjUv‡ii GKwU wgkÖ‡Y wPwb I cvwbi AbycvZ 2:3|G‡Z KZ wjUvi wgkÖY, wPwb Øviv
cÖwZ¯’vwcZ Ki‡j bZzb wgkÖ‡Y wPwb I cvwbi AbycvZ 1:1 n‡e?) [GTCL (AM-General)-2018]
a. 12 liters b. 15 liters c. 10 liters d. 20 liters Ans: c
Solution:
Given that, Total mixture = 60 litres
Ratio of Sugar: Water = 2:3
Sum of Ratio = 5
60  2 60  3
So, Sugar = =24 litres and water = = 36 litres
5 5
Suppose, x litres mixture should be replaced by sugar, (cÖ_gevi x Zz‡j wb‡j Zv‡Z wPwb I cvwb Df‡q
Av‡Q, wKš‘ c‡i Avevi x wPwb w`‡j Zv‡Z ‡Kvb cvwb †bB|)

then the new mixture = (60-x) litres


2 3
So,the new ratio of sugar and water = {(60-x) }:{(60-x)  )}
5 5
According to the question,
2 3
{(60-x) )+x)}:{(60-x)  )} =1:1 [m¤úyY© wgkÖ‡Yi mv‡_ wPwb cÖwZ ¯’vcb Ki‡j bZzb wgkÖ‡Y wPwb: cvwb=1:1]
5 5
120  2x  5x 180  3x
Or, : =1:1
5 5
Or, 120+3x=180 -3x Or, 6x = 60 x = 10 Ans:10 liters
cÖgvY:
60 Gi g‡a¨ wPwb I cvwb 24 I 36 Av‡Q,| ‡gvU wgkÖY †_‡K 10 Zz‡j wb‡j Aewkó 60-10 = 50 G wPwb _v‡K 20 Ges
cvwb _v‡K 30 ( KviY AbycvZ Av‡Mi gZB 2:3 Av‡Q) GLb GB 50 G 10wjUvi wPwb w`‡j bZzb =50+10 = 60 wjUvi
wgkÖ‡Y wPwb _vK‡e 20+10 = 30 I cvwbI _vK‡e 30 hvi AbycvZ 30:30 ev 1:1|
x y
2. If is 1 more than ,than =? [GTCL (AM-General)-2018]
z z
a. x+z b. x-y c. y-z d. x-z Ans: d
Solution:
x y
- =1 (more A_© †ewk| Avi †ewk n‡j e¨eavb 1 n‡e|)
z z
xy
Or, =1 Or, x-y = z y = x-z Ans:x-z
z

Khairul’s IBA Math Book - 20

www.bdniyog.com
3. Arif bought 17 pens of three colors –black ,green and red. they cost TK . 5, TK. 10 and
TK. 25 each respectively . The total amount that Arif paid was TK .205.If Arif bought
twice as many green pens as red pens , how many black pens did he buy? (Avwid Kv‡jv,
meyR I jvj i‡Oi 17 wU Kjg wKb‡jv hvi g~j¨ h_vµ‡g 5 UvKv, 10 UvKv Ges 25 UvKv| †m †gvU 205 UvKv cwi‡kva
K‡i| hw` Avwid jvj i‡Oi wظY meyR i‡Oi Kjg wK‡b _v‡K, Zvn‡j †m KZwU Kv‡jv i‡Oi Kjg wK‡bwQj?) [GTCL
(AM-Gen)-2018]
a. 5 b. 6 c. 7 d. 8 Ans: a
Solution:
Let, red pens = x So, green pens =2x
And black pens =17 – (x+2x) = 17 -3x (me©‡gvU †_‡K Ab¨ `ywU we‡qvM|)
According to the question,
25x+2x10 + (17- 3x) 5 = 205 (me¸‡jv Kj‡gi †gvU `vg = 205 UvKv|)
Or, 25x+20x+85-15x = 205
Or, 30x = 205 – 85 Or, 30x = 120 x = 4
So, number of black pens = (17 – 3  4) =17 – 12 = 5 Ans:5
4. Alam starts working on a job and works on if for 12 days and completes 40% of the
work. Then Babu joins alam and together the complete the rest of the work in 12 days .
How long (in days) will it taka Babu to complete the job if he works alone? (Avjg GKwU
KvR Ki‡jv †m 12 w`‡b KvRwUi 40% †kl Kivi ci evey mv‡_ ‡hvM †`q Ges Df‡q Aewkó KvRUzKz 12 w`‡b †kl
K‡i, evey GKvKx KvRwU Ki‡j m¤ú~Y© KvRwU †kl Ki‡Z Zvi KZ mgq jvM‡e? [GTCL (AM-General)-2018]
a. 45 days b. 50 days c. 55days d. 60 days Ans:d
Solution:
Alam completes alone 40% in 12 days.
12
So, Alam can complete 1% in days.
40
12  100
, Alam can complete 100% in = 30days.(A_©vr Avjg GKv m¤ú~Y© KvRUv 30w`‡b Ki‡Z cv‡i)
40
Again, Alam and Babu both completes rest 100% - 40% = 60% work in = 12 days.
12
Alam and Babu = 1% work in = days.
60
12  100
, Alam can complete 100% in = 20days (2 R‡b wg‡j m¤ú~Y© KvRwU 20 w`‡b Ki‡Z cv‡i)
60
1
Now, Alam and Babu’s 1 day’s work =
20
1
And Alam’s 1 day’s work =
30
1 1 1
So, Babu’s 1 day work = - = [2 R‡bi 1 w`‡bi Kiv KvR -1 R‡bi KvR = Ab¨ Rv‡bi KvR]
20 30 60
1
So, Babu can complete part in = 1 day
60
Babu completes the whole work in = 160 = 60 days . Ans:60 days

Khairul’s IBA Math Book - 21

www.bdniyog.com
5. If Rahim walks at 14 km/hr instead of 10 km/hr for a certain time, he would have
walked 20 km more .If Rahim walks at a speed of 10 km/hr ,the distance travelled by
him within that time is –(hw` iwng ‡Kvb wbw`©ó mg‡q 10 wK.wg./N›Uv Gi cwie‡Z© 14 wK.wg/N›Uv †e‡M nvu‡U,
Zvn‡j †m 20 wK.wg. c_ †ewk †h‡Z cv‡i| wKš‘ hw` †m 10 wK.wg./N›U †e‡M nuv‡U, Zvn‡j H mg‡qi g‡a¨ ‡m KZUyKz
c_ AwZµg Ki‡e?) [GTCL (AM-General)-2018]
a. 42km b. 50km c. 60km d. 64km Ans: b
Solution:
Speed difference = 14-10 = 4,
If difference is 4 then actual distance travelled = 10 km
10  20
Then when difference is 20km ‘’ ‘’ =  50km
4
[Note: 10 wKwgi cwie‡Z© 14 wKwg †M‡j GiKg n‡Zv| wKš‘ Avm‡j †m 10wKwg MwZ‡ZB wM‡qwQj| ZvB DËi 50]
6. A merchant has 1000 kg of sugar, part of which he sells at 8% profit and the rest at
18% profit. He gains 14% on the whole. The quantity sold at 18% profit is: (GKRb e¨emvqx
Gi Kv‡Q 1000 †KwR wPwb Av‡Q| Zvi ga¨ †_‡K wKQz wPwb 8% jv‡f wewµ Ki‡jb Ges Aewkó wPwb 18% jv‡f wewµ
Ki‡jb| †gv‡Ui Dci Zvi 14% (1000 ‡KwR wewµ‡Z) jvf n‡j 18% jv‡f KZ †KwR wewµ K‡iwQ‡jb?) [BD
House Building FC (OF)-2017] + [Pubali Bank (TAT)-2017] & [GTCL (AM-General)-2018]
a. 600 kg b. 560 kg c. 400 kg d. 640 kg Ans: a
 Written Solution:
Let the sugar of 18% profit is = x
So, the sugar of 8% profit = 1000-x
ATQ,
18% of x + 8% of ( 1000-x) = 14% of 1000  x = 600
Solution, By the rule of allegation, we have :
Profit on 1st part Profit on 2nd part
8% 18%
Mean
profit
14%

4 6
Ratio of 1st and 2nd parts = 4 : 6 = 2 : 3
3 
 Quantity of 2nd kind =   1000kg = 600 kg
5 
e¨vL¨v: cÖ_‡g Zv‡`i wgkÖ‡Yi ev wewµi AbycvZ †ei Kiv n‡q‡Q| A_©vr †Kvb ai‡Yi wPwb KZ Abycv‡Z wewµ Ki‡j
‡gv‡Ui Dci jvf 14% n‡e| Zv †ei Kivi ci †gvU 1000 †KwR †_‡K 18% jv‡f wewµ Kiv wPwbi cwigvY †ei Kiv
n‡q‡Q|
7. The sum of three consecutive odd integers is 40 more than the first of the number. What
is the middle number? (3 wU avivevwnK we‡Rvo msL¨vi mgwó cÖ_g msL¨viwUi †P‡q 40 †ewk| gv‡Si msL¨vU
KZ?) [GTCL (AM-General)-2018]
a. 16 b. 18 c. 19 d. 24 Ans: c

Khairul’s IBA Math Book - 22

www.bdniyog.com
Solution:
Let, the numbers are x, x+2, x+4 (we‡Rvo msL¨v ZvB e¨eavb: 2 K‡i)
According to the question,
x + x + 2 + x + 4 = x + 40 (3 wU mgwó = cÖ_g msL¨v +40)
Or, 3x – x + 6 = 40
Or, 2x=34 x = 17
So, the middle number is = x +2 =17+2 =19 Ans:19
gy‡L gy‡L: 3Uv msL¨v-1g msL¨v = 40 A_©: 2q+3q msL¨i ‡hvMdj = 40 n‡j Mo n‡e 20 Ges msL¨v `ywU n‡e GKwU
20 Gi †_‡K 1 Kg I Ab¨wU 20 Gi †_‡K 1 †ewk| ( KviY msL¨v¸‡jv we‡Rvo)
gv‡Si msL¨vwU n‡e 20-1 = 19|

8. When the positive integer n is divided by 5,the remainder is 2. Which of the following
must be true? (n GKwU abvZ¥K c~Y©msL¨v hv 5 Øviv fvM Ki‡j fvM‡kl 2 n‡e| Zvn‡j wb‡Pi ‡KvbwU Aek¨B mZ¨
n‡e?) [GTCL (AM-General)-2018]
i. n is odd
ii. n+1 can not be a prime number
iii. (n+2) divided by 7 has remainder 2
a. none b. i only c. i and ii only d. ii and iii only Ans: a
Solution:
Let, n = 12 (5 w`‡q fvM Ki‡j 2 Aewkó _v‡K Ggb msL¨v )
now, (i) n = not an odd number
(ii) n+1 = 12+1 = 13 is a prime number
(iii) n+2 = 12+2 = 14 is divided by 7 has no reminder,
So, all the given information is not true, thus ans is none.

9. When x is divided by 7, the remainder is 6.Which of the following must be an even


number? (x ‡K 7 Øviv fvM Ki‡j fvM‡kl 6 nq, Zvn‡j wb‡Pi †KvbwU Aek¨B ‡Rvo msL¨v n‡e ?) [GTCL (AM-
General)-2018]
a. x+6 b. x3+x2+x c. x2+x d. x- 4 Ans:c
Solution:
if x= 13 then,
(a) X+6 = 13+6 = 19 = Odd number
(b) x3+x2+x = (13)3+(13)2+13 = 2197+169+13=2379 = Odd number
(c) x2+x = (13)2+13 = 169+13 =182 = Even number
(d) x- 4 = 13 – 4 = 9 = Odd number
GLv‡b option (c) QvWv evwK me¸‡jv we‡Rvo msL¨v| myZivs mwVK DËi: c

10. Pens that used to cost TK .150 for a package of 3 now cost TK.300 for a package of 5.
What is the present increase in the price of these pens? (3 wU Kj‡gi GKwU c¨v‡K‡Ri g~j¨ wQj
150 UvKv, eZ©gv‡b 5 wU Kj‡gi GKwU c¨v‡K‡Ri g~j¨ 300 UvKv| eZ©gv‡b Kj‡gi g~j¨ KZ e„w× †c‡q‡Q ?) [GTCL
(AM-General)-2018]
a. 5% b. 10% c. 15% d. 20% Ans: d

Khairul’s IBA Math Book - 23

www.bdniyog.com
Solution:
150 300
Old price of 1 pen = = Tk.150 New price of 1 pen = = Tk.60
3 5
Price increased = 60-50 = Tk.10
10  100
 increased % = = 20% Ans: 20%
50
11. A year ago the price of a toothbrush and the price of a comb were both TK. 50.the price
of the toothbrush was increased by 20% while the price of the comb was decreased by
10%.what is the difference in taka between the current price of the toothbrush and the
comb?(GK eQi c~‡e© eªvk Ges wPiæwb cÖwZwUi g~j¨ 50 UvKv wQj| cieZ©v‡Z eªv‡ki g~j¨ 20% e„w× †cj Ges wPiæwbi
g~j¨ 10% n«vm †cj| eªvk I wPiæwbi eZ©gvb g~‡j¨i cv_©K¨ KZ UvKv?) [GTCL (AM-General)-2018]
a. 12 b. 14 c. 15 d. 20 Ans: c
Solution:
Current price of a toothbrush = 50+20% of 50 = 50+10 = 60
Current price of a comb = 50 – 10% of 50 = 50-5 = 45
Difference of two prices = 60-45 = Tk. 15 Ans:15
12. If 7 > x > 2 and 3 < x < 8, which of the following best describes x? [GTCL (AM-General)-
2018]
a. 2 < x < 8 b. 2 < x < 7 c. 3< x < 5 d. 3 < x < 6 Ans: a
Solution:
From the given data, 7 > x > 2 and 3 < x < 8, we can write 7 > x and 8 > x means 8> x
Again, x > 2 and x > 3 means x > 2
So, we can write 2 < x < 8
evsjvq e¨vL¨:
x Gi gvb hw` 7 †_‡K †QvU nq Zvn‡j Aek¨B 8 ‡_‡KI ‡QvU n‡e|
Avevi x Gi gvb hw` 3 ‡_‡K eo nq Zvn‡j Aek¨B 2 ‡_‡KI eo n‡e|
myZivs ejv hvq 2 < x < 8 Zvn‡j mwVK DËi n‡e a.
13. Apu took 3/5 of the marbles kept in a box. His younger took another 3/5 of the
remaining marbles. Then his sister took another 3/5 of remaining marbles. What
fractions of the marbles are left in the box?(Acy GKwU ev· †_‡K 3/5 Ask gv‡e©j wbj| Gici Zvi
†QvU fv&B Aewkó gv‡e©j †_‡K 3/5 Ask gv‡e©j wbj| Avevi Zvi †evb Aewkó gv‡e©‡ji 3/5 Ask gv‡e©j wbj| ev‡·
Avi KZ fvM gv‡e©j _vK‡jv?) [GTCL (AM-General)-2018]
8 20 45 25
a. b. c. d. Ans:a
125 125 125 125
Solution:
Let, box contain = 125 marbles [5 Gi Nb 53=125 KviY wZbevi 5 fvM Ki‡Z n‡e]
3 Tips:
Apu took =125 = 75 marbles
5 [x a‡iI Kiv hvq wKš‘& wcÖwji cixÿvq Aí
remaining marbles = 125-75 = 50 mg‡q fMœvs‡ki †ewk †ewk wn‡me Ki‡Z A‡bK
3 mgq jvM‡e| ZvB ni w`‡q wefvR¨ †h †Kvb
Then, younger brother took = 50 = 30 marbles msL¨v a‡i wn‡me Kiv DËg|]
5
and now remaining = 50-30 = 20

Khairul’s IBA Math Book - 24

www.bdniyog.com
3
Again, his sister took = 20 = 12
5
So, at last remaining marbles = 20-12 = 8
8 8
 require fraction is = (‡gvU gv‡e©j= 125wU Ges Aewkó Av‡Q 8 wU ) Ans:
125 125
14. The average daily wages of female workers in a factory is TK. 30 and that of male
workers is Tk.42. if the average wages of all the workers is TK.37, what is the ratio of
male workers? (GKwU KviLvbvi gwnjv Kg©Pvix‡`i ˆ`wbK Mo gRywi 30 UvKv Ges cyiæl Kg©Pvix‡`i ˆ`wbK Mo
gRyix 42 UvKv| mKj Kg©Pvixi Mo gRyix 37 UvKv n‡j cyiæl I gwnjv Kg©Pvixi AbycvZ KZ ?
a. 6:5 b. 5:7 c. 5:6 d. 7:5 Ans:d
Solution:
Let, number of male = x and number of female = y
ATQ,
42x+30y = 37(x+y) [cyiæl‡`i †gvU Avq+gwnjv‡`i †gvU Avq = cyiæl gwnjv mevi GKwÎZ Avq]
 42x+30y = 37x+37y
 42x-37x = 37-30y
 5x = 7y
x 7
 
y 5
 x: y = 7:5
So, the ratio of male to female workers is 7:5 Ans:7:5
Alternative solution:
Rule of Allegation Gi gva¨‡g GB AsKwU K‡qK †m‡K‡Û mgvavb Kiæb|
Male wages Female wages
42 30
Mean
profit37

7 5
So, ratio of male and female is 7:5
================================
==

Khairul’s IBA Math Book - 25

www.bdniyog.com
Department of Agricultural Extension (DAE)
Post name : Officer Assistant cum computer typist
Exam Date : 13-04-2018 Exam taker: IBA, DU.
1. iv‡mj, Avmv` I ivRy‡K 315 UvKv fvM K‡i †`Iqv n‡jv | G‡Z iv‡m‡ji UvKv Avmv‡`i UvKvi 3/5 Ges Avmv‡`i
UvKv ivRyi UvKvi 2 ¸Y n‡jv| ivRy KZ UvKv †cj? [K…wl m¤úªmviY Awa`ßi- (Awd: mn:+UvBwc÷)-2018]
K. 60 UvKv L. 90 UvKv M. 150 UvKv N. 75 UvKv DËi:N
Solution: (fMœvsk Qvov mnR wbq‡g mgvavb)
g‡bKwi , ivRy cvq = 3K UvKv Zvn‡j Avmv` cv‡e = 5K UvKv (j‡ei UvKv iv‡mi Ges n‡ii UvKv Avmv`)
myZivs ivRy cv‡e, 5K  2 = 2.5K (Avmv‡`i UvKv ivRyi UvKvi wظY n‡j ivRyi UvKv Avmv‡`i UvKvi A‡a©K n‡e)
cÖkœg‡Z,
3K+5K+2.5K = 315 (mevi UvKvi †hvMdj = 315 UvKv)
315
ev, 10.5K = 315  K = = 30UvKv|
10.5
Zvn‡j ivRy cv‡e, 2.530 = 75 UvKv|
3 6K
fMœvsk a‡i mgvavb Kivi Rb¨, ivRy = K n‡j Avmv` = 2K Ges iv‡mj = 2K  = UvKv|
5 5
cÖkœg‡Z,
6K 21K 315  5
K+2K+ =315  = 315 K=  K = 75 A_©vr ivRy cv‡e, 75 UvKv|
5 5 21
[
2. x, y Ges z wZbwU cyY© msL¨v | hw` x  y  z Ges y  2 nq Z‡e wb‡Pi †KvbwU Aek¨B fyj? [K…wl m¤úªmviY Awa`ßi-
(Awd: mn:+UvBwc÷)-2018]
K. xyz  0 L. xy –z  0 M. y-xz  0 N. †KvbwU bq DËi:M
Solution:
‡`Iqv Av‡Q , x  y  z
awi, x = 2, y =3, z = 4
GLb Ackb ¸‡jv‡Z GB gvb ewm‡q †`Lv hvK |
(K) xyz  0 234  0  24  0 GUv n‡Z cv‡i|
(L) xy – z  0  23 - 4 0 2 0 GUvI n‡Z cv‡i|
(M) y- xz  0  3 - 24  0  - 5  0 = Bnv KL‡bvB mwVK bq| KviY FYvZ¥K msL¨v memgq 0 Gi †_‡K
†QvU| myZivB GUvB DËi|

3. evey I Rvgv‡ji gvwmK †eZ‡bi bycvZ 7:5 Ges `yR‡bi gvwmK †eZb GK‡G 24,000 UvKv | GK eQi c‡i eveyi †eZb
500UvKv Ges Rvgv‡ji †eZb 350UvKv e„w× †cj | GK eQi c‡i Zv‡`i gvwmK †eZ‡bi AbycvZ KZ n‡e ? [K…wl
m¤úªmviY Awa`ßi- (Awd: mn:+UvBwc÷)-2018]
K. 290:207 L. 145:103 M. 500:350 N. 7:5 DËi:K

Khairul’s IBA Math Book - 26

www.bdniyog.com
Solution:
awi, eZ©gv‡b eveyj I Rvgv‡ji gvwmK †eZb h_vµ‡g 7K I 5K UvKv|

cÖkœg‡Z,
7K + 5K =24000 ev, 12K = 24000 K = 2000 UvKv
myZivs eveyj I Rvgv‡ji †eZb h_vµ‡g 14,000 UvKv I 10,000 UvKv|
GK eQi c‡i Zv`i †eZ‡bi AbycvZ n‡e =(14000+500) :(10000+ 350) =14500 : 10,350 = 290 : 207

4. XvKv ‡_‡K Kwi‡gi evwoi `„iZ¡ 355 wK. wg. | †m ev‡m XvKv †_‡K evwo iIqvbv n‡jv | 319 wK. wg hvIqvi ci evmwU
bó n‡q ‡M‡j Kwig evwK c_ wi·vq †Mj| ev‡mi MwZ‡eM 22 wK. wg./ N›Uv I wiKkvi MwZ‡eM 6 wK. wg ./N›Uv n‡j
evwo †cxuQv‡Z Kwi‡gi †gvU KZ mgq jvM‡e ? [K…wl m¤úªmviY Awa`ßi- (Awd: mn:+UvBwc÷)-2018]
K. 20 N›Uv L. 20 N›Uv 30 wgwbU M. N›Uv 50 wgwbU N. 21 N›Uv DËi:L
Solution:
ev‡m hvq = 319 wK. wg.c_ Ges wi·vvq †M‡Q = 355-319 = 36 wK. wg.|
319
ev‡m †h‡Z mgq jv‡M = = 14.5 N›Uv
22
36
wi·vq mgq jv‡M= = 6 N›Uv
6
‡gvU mgq jv‡M = 14.5 + 6 = 20.5 N›Uv ev 20 N›Uv 30 wgwbU | DËi: 20 N›Uv 30 wgwbU

5. 8 Rb †jvK GKwU KvR 6 w`‡b Ki‡Z cv‡i | KvRwU wZb w`‡b Ki‡Z n‡j KZR‡b bZzb †jvM wb‡qvM Ki‡Z n‡e ?
[K…wl m¤úªmviY Awa`ßi- (Awd: mn:+UvBwc÷)-2018]
K.6 L. 8 M.7 N.12 DËi:L
Solution:
6 w`‡b Ki‡Z cv‡i = 8 Rb †jvK|
1 Ó Ó Ó = 8 6 Ó Ó (Kg w`‡b Ki‡Z †ewk †jvK jvM‡e ZvB ¸Y|)
8 6
3Ó Ó Ó = Ó Ó = 16 Rb myZivs AwZwi³ †jv‡Ki msL¨v = 16-8 = 8 Rb|
3
gy‡L gy‡L: 6 w`‡bi KvR 3 w`‡b A_©vr A‡a©K mg‡qi g‡a¨ Ki‡Z PvB‡j Av‡Mi 8 Rb †jv‡Ki wظY †jvK jvM‡e|
8 Rb †h‡nZz Av‡M †_‡KB Av‡Q ZvB Av‡iv bZzb †jvK jvM‡e 8 Rb| DËi:8
6. GKwU Kjg I GKwU eB‡qi g~j¨ GK‡Î 95 UvKv | KjgwUi g~j¨ 15 UvKv †ewk Ges eBwUi g~j¨ 14 UvKv Kg n‡j
KjgwUi g~j¨ eBwUi g~‡j¨i wØMyY n‡Zv | eBwUi g~j¨ KZ UvKv? [K…wl m¤úªmviY Awa`ßi- (Awd: mn:+UvBwc÷)-2018]
K.49 L. 46 M.50 N.40 DËi:L
Solution:
g‡b Kwi, eBwUi g~j¨ = K UvKv
 Kj‡gi g~j¨ = (95- K) UvKv
cÖkœg‡Z,
95 - K + 15 = 2(K - 14) ev, 110 - K = 2K - 28 ev, 3K = 138  K = 46
myZivs, eBwUi g~j¨ 46 UvKv| DIi : 46
7. GKwU c‡Y¨i weµqg~j¨ wظY n‡j we‡µZvi gybvdv †e‡o 3 ¸Y n‡e | g~j¨e„w× bv K‡i cY¨wU weµq Ki‡j we‡µZv
kZKiv KZ gybvdv Ki‡e ? [K…wl m¤úªmviY Awa`ßi- (Awd: mn:+UvBwc÷)-2018]
K.50 L. 100 M.150 N. †KvbwU bq DËi:L

Khairul’s IBA Math Book - 27

www.bdniyog.com
Solution:
g‡b Kwi, cY¨wUi weµqg~j¨ = 100 UvKv| hv‡Z jvf Av‡Q K UvKv|
weµqg~j¨ wظY n‡j bZzb weµqg~j¨ = 200UvKv hv‡Z jvf Av‡Q 3K UvKv|
cÖkœg‡Z,
3K-K = 200-100 ( `yB jv‡fi cv_©K¨ = `yB weµqg~‡j¨i cv_©K¨)
ev, 2K = 100 myZivs K = 50 UvKv|
Zvn‡j cÖ_gevi 100 UvKvq jvf n‡qwQj 50UvKv Ges µqg~j¨ = 100-50 = 50 UvKv|
50UvKvq jvf 50 UvKv n‡j jv‡fi nvi = 100% DIi : 100%
8. GKwU †Kv¤úvwbi 46 kZvsk Kg©KZ©v cyiæl | hw` 60 kZvsk Kg©KZv© †jevi BDwbqb K‡i Ges Zv‡`i g‡a¨ 70 kZvsk
cyiyl nq, BDwbqb K‡i bv Ggb Kg©KZ©v‡`i g‡a¨ kZKiv KZ Rb gwnjv? [K…wl m¤úªmviY Awa`ßi- (Awd: mn:+UvBwc÷)-
2018]
K.90 L. 87.5 M.80 N.75 DËi:K
Solution:
‡gvU Kg©KZv© 100 Rb n‡j cyiæl 46 Rb Ges gwnjv = 100-46 = 54 Rb|
‡jevi BDwbqb K‡i 60 ( cyiæl+gwnjv) Rb Ges K‡i bv = 100-60 = 40 (cyiæl+gwnj) Rb|
GLb †jevi BDwbDb Kiv cyiæ‡li msL¨v = 60 Gi 70% = 42 Rb|
‡jevi BDwbqb K‡i bv Ggb cyiæl = ‡gvU cyiæl-BDwbqb Kiv cyiæl = 46-42 = 4 Rb|
Zvn‡j †gvU ‡jevi BDwbqb bv Kiv 40 R‡bi g‡a¨ cyiæl 4 ev` w`‡j gwnjv _v‡K 40-4 = 36 Rb|
36 36  100
40 R‡bi g‡a¨ 36 Rb gwnjv n‡j 1 R‡b gwnjv = Ges 100 R‡bi g‡a¨ gwnjv = =90%
40 40
9. GKRb weµZv 17 wU Kjg 720 UvKvq wewµ K‡i ‡h †jvKmvb Ki‡jv Zv 5 wU Kj‡gi µqg~‡j¨i mgvb |GKwU Kj‡gi
µqg~j¨ KZ UvKv ? [K…wl m¤úªmviY Awa`ßi- (Awd: mn:+UvBwc÷)-2018]
K.52 L. 54 M.60 N. †KvbwU bq DËi: M
Solution:
‡h‡nZz ÿwZ n‡q‡Q ZvB
17wUi µqg~j¨ - 17wUi weµqg~j¨ = 5wUi µqg~j¨ [µqg~j¨ eo Ges Zv †_‡K weµqgyj¨ we‡qvM Ki‡j ÿwZ †ei n‡e]
ev, 17wUi µqg~j¨ - 5wUi µqg~j¨ = 17wUi weµqg~j¨|
ev, 12wUi µqg~j¨ = 720 UvKv ( ‡h‡nZz 17wUi weµqg~j¨ 720 UvKv cÖ‡kœB †`qv Av‡Q)
720
myZivs 1wUi µqgyj¨ = = 60 UvKv
12

10. `kwU msLvi Mo x Ges G‡`i cvuPwU ksL¨vi Mo y | hw` evwK cvuPwU msL¨vi Mo z nq Z‡e wb‡Pi †KvbwU mwVK ? [K…wl
m¤úªmviY Awa`ßi- (Awd: mn:+UvBwc÷)-2018]
K.2x = y + z L. 2x =y + z M. x = 2y + 2x N. †KvbwU bq DËi:L
Solution:
10 wU msL¨vi mgwó = 10x
cÖ_g cvuPwU msL¨vi mgwó = 5y
evwK 5 wU msL¨vi msL¨vi mgwó = 5z
cÖkœg‡Z,
10x = 5y+5z (cÖ_g 10wUi †hvMdj = 5+5wUi †hvMdj)
ev, 10x = 5(y+z)  2x = y+z

Khairul’s IBA Math Book - 28

www.bdniyog.com
11. hw` 7,11,15 Ges x Gi Mo y nq Z‡e x Gi gvb KZ ? [K…wl m¤úªmviY Awa`ßi- (Awd: mn:+UvBwc÷)-2018]
K. 4y -33 L. (y – 26)/4 M. y -33 N. †KvbwU bq DËi:K
Solution:
cÖkœg‡Z,
7  11  15  x  y  4 [`y cv‡ki mgwó mgvb]
ev, 33+x = 4y x = 4y -33

12. hw` 15,28,33 GB wZbwU msL¨vi ¸bdj z nq,Z‡e wb‡Pi ‡KvbwU GKwU c~Y© msL¨v n‡e bv ? [K…wl m¤úªmviY Awa`ßi-
(Awd: mn:+UvBwc÷)-2018]
z z z
K. L. M. N. me¸wjB c~Y© msL¨v DËi:L
21 24 55
Solution:
‡`Iqv Av‡Q, z = 152833 GLb Ackb¸‡jvi ni †`qv Av‡Q, 21,24 Ges 55| Avgiv 152833 ‡K Ggbfv‡e
K‡qKwU Drcv`K evbv‡e hv‡Z GB Drcv`K¸‡jv Av‡m wK bv Zv †`Lv hvq| hw` Av‡m Zvn‡j Zv c~Y© msL¨v Avi bv
Avm‡j fMœvsk|
152833 = 53722311 = GLvb †_‡K 73 = 21 evbv‡bv hvq, Avevi, 511 w`‡q 55 evbv‡bv hvq|
wKš‘ †Kvbfv‡e 24 evbv‡bv hv‡”Q bv| Zvn‡j z Gi g‡a¨ 24 Drcv`KwU bv _vKvq eo msL¨vwU 24 w`‡q wefvR¨ n‡e bv
ZLb Zv GKwU fMœvsk n‡e|

13. hw` -2  x  2 and 3  y  8 nq, Z‡e †KvbwU mwVK ? [K…wl m¤úªmviY Awa`ßi- (Awd: mn:+UvBwc÷)-2018]
K.1  y – x  10 L. 1  y - x  5 M.5  y – x  6 N.‡KvbwUB bq DËi:M
Solution:
‡`qv Av‡Q, 3  y 8
-2  x 2
[we‡qvM K‡i cvB] 5  y - x 6 (Ackb M Gi mv‡_ wg‡j †M‡Q|)
14. GKwU ¯‹zv‡ji 70% QvÎ dzUej, 75% nwK Ges 80% QvÎ wµ‡KU †Lj‡Z cQ›` K‡i H ¯‹z‡ji kZKiv KZ Rb QvÎ
wZbwU ‡LjvB ‡Lj‡Z cQ›` K‡i ? [K…wl m¤úªmviY Awa`ßi- (Awd: mn:+UvBwc÷)-2018]
K. 25% L. 30% M. 35% N. †KvbwU bq DËi:K
Solution:
dzUej cQ›` K‡i bv = 100-70% = 30%
nwK cQ›` K‡i bv = 100-75% = 25%
wµ‡KU cQ›` K‡i bv = 100-80% = 20%
 †Kvb †LjvB †Lj‡Z cQ›` K‡i bv = {100-(30+25+20)}% = (100-75)% = 25%
15. Kvjv‡gi †eZb x UvKv n‡j, hv mvjv‡gi †eZ‡bi A‡a©K Ges Avwi‡~di †eZ‡bi Pvi ¸Y| Zv‡`i wZb R‡bi †eZ‡bi
†hvMdj KZ ? [K…wl m¤úªmviY Awa`ßi- (Awd: mn:+UvBwc÷)-2018]
13x 73
K. L. M. 3x N. †KvbwU bq DËi:K
4 4
Solution:
x
Kvjv‡gi †eZb = x UvKv, mvjv‡gi †eZb =2x UvKv Ges Avwi‡di ‡eZb = UvKv|
4
x 4x  8x  x 13x
 wZb R‡bi ‡gvU †eZb = x+2x+ = = UvKv
4 4 4
Khairul’s IBA Math Book - 29

www.bdniyog.com
16. iwd‡Ki IRb hw` 17 †KwR K‡g hvq Z‡e Zvi IRb Avwi‡di IR‡bi A‡a©K n‡q hv‡e | Zv‡`i `yR‡bi IR‡bi
†hvMdj 140 †KwR n‡j iwd‡Ki IRb KZ †KwR ? [K…wl m¤úªmviY Awa`ßi- (Awd: mn:+UvBwc÷)-2018]
K.55 L. 58 M.61 N.‡KvbwU bq DËi:L
Solution:
awi, iwd‡Ki IRb = x †KwR Ges Avwi‡di IRb = y †KwR|
1g kZ©g‡Z,
x+y = 140 ev, y = 140-x
2q kZ©g‡Z,
2(x-17) = y (hvi IRb K‡g †m A‡a©K n‡q hvq, ZvB Zv‡K 2 w`‡q ¸Y Ki‡j Ab¨R‡bi mgvb n‡e|)
ev, 2x -34 = 140-x [y = 140-x ewm‡q|]
ev, 3x = 174
 x = 58
myZivs iwd‡Ki IRb = 58 †KwR |
♦ gy‡L gy‡L Kivi wbqg:
GKRb Av‡iKR‡bi A‡a©K A_© GKRb GK¸Y n‡j Ab¨Rb 2 ¸Y|
140 †KwR †_‡K 17 ev` w`‡j hv _v‡K Zv‡K 3 fvM K‡i ‡h A‡a©K Zv‡K 1 ¸Y Ges Ab¨Rb‡K 2 ¸Y w`‡j
gy‡L gy‡L DËi †ei n‡e, 140-17 = 1233 = 41, †h‡nZz hvi IRb Kg ZviUv †ei Ki‡Z e‡j‡Q ZvB DËi
n‡e 41+17 = 58| (1fv‡Mi mv‡_ Zvi IRb †_‡K ev` †`qv 17 †hvM Kiv n‡q‡Q|)
17. `ywU abvZœK msL¨vi cv_©K¨ 6 Ges Zv‡`i e‡M©i cv_©K¨ 108| msL¨v `ywUi †hvMdj KZ ? [K…wl m¤úªmviY Awa`ßi- (Awd:
mn:+UvBwc÷)-2018]
K.6 L. 8 M.12 N.18 DËi:N
Solution:
GLv‡b, x-y = 6 Ges x2-y2 = 108
GLb, x2-y2 = 108
 (x+y)(x-y) = 108
 (x+y)6 = 108  x+y = 18
myZivs msL¨v `ywUi †hvMdj = 18 | DËi: 18
18. hw` xy  0 Ges y > 0 nq, Z‡e wb‡Pi †KvbwU Aek¨B fzj? [K…wl m¤úªmviY Awa`ßi- (Awd: mn:+UvBwc÷)-2018]
2y  3 2y  3 2x  1
K. 6 L. 6 M. 6 N. me¸‡jv mwVK DËi:L
2x x2 2x
Solution:
‡`Iqv Av‡Q, xy 0 GLv‡b `ywU msL¨vi ¸Ydj 0 Gi †_‡K †QvU ev FYvZ¥K ZLwb nq hLb Zv‡`i g‡`¨ †h ‡Kvb
GKwU msL¨v FYvZ¥K nq| Avevi †`qv Av‡Q, y  0 A_©vr y Gi gvb 0 Gi †_‡K eo myZivs y GKwU abvZ¥K msL¨v|

GLb Ackb¸‡jv †_‡K cÖgvb Ki‡Z n‡e †KvbUv Aek¨B fzj|


2y  3
 6 A_©vr L AckbwU fzj| KviY x Gi gvb FYvZ¥K Ges FYvZ¥K msL¨v †_‡K †Kvb wKQz we‡qvM Ki‡j Zv
x2
Avevi FYvZ¥K B _v‡K| Avevi FYvZ¥K msL¨v w`‡q Dc‡ii 2y+3 abvZ¥K msL¨v‡K fvM Ki‡j fvMdj FYvZ¥K B
Avm‡e, Ges FYvZ¥K msL¨v 6 Gi †_‡K eo n‡Z cv‡i bv| ZvB GUvB fzj|

Khairul’s IBA Math Book - 30

www.bdniyog.com
19. GKwU K¬v‡ei 80% m`m¨ cyiæl | cyiæl m`m¨‡`i 50% †ckvq Wv³vi ,30% †ckvq BwÄwbqvi Ges evwK 40 Rb
e¨emvqx | H K¬v‡ei †gvU m`m¨ msL¨v KZ? [K…wl m¤úªmviY Awa`ßi- (Awd: mn:+UvBwc÷)-2018]
K.100 L. 200 M.250 N. ‡KvbwU bq DËi: M
Solution:
g‡b Kwi, K¬v‡ei †gvU m`m¨ msL¨v = 100 Rb |
Gi g‡a¨ cyiæl = 80%
GLb, cyiæl‡`i g‡a¨ Wv³vi I BwÂwbqvi = 50+30 = 80% myZivs e¨emvqxi msL¨v =100-8 20% |
40 40  100
GLb, 20% e¨vemvqx = 40 Rb n‡j 1% = Ges 100% = = 200 Rb|
20 20
myZivs, me©‡gvU cyiæl = 200 Rb =wKš‘ †ei Ki‡Z n‡e cyiæl gwnjv mevB‡K|
‡h‡nZz cyiæ‡liv †gvU m`‡m¨i 80%|
200 200  100
Zvn‡j 80% = 200 Rb n‡j 1% = Rb = Ges 100% = = 250 Rb|
80 80
 K¬v‡ei †gvU m`m¨ msL¨v = 250 Rb|

================================

Khairul’s IBA Math Book - 31

www.bdniyog.com
Department OF Agricultural Extension (DAE)
Post name: Store Keeper Exam Date: 01-12-2017

1. GK e¨vw³ GKwU `ªe¨ µq K‡i 10% ÿwZ‡Z weµq Ki‡jv| hw` †m `ªe¨wU 20% K‡g µq K‡i 55UvKv †ekx‡Z weµq
Ki‡Zv Zvn‡j 40% jvf nZ| `ªe¨wUi µqg~j¨ KZ? [DAE-(Store Keeper)-2017]
a. 200 b. 220 c. 250 d. 300 Ans: c
Solution:
cÖ_g µqg~j¨ 100UvKv n‡j 10 ÿw‡Z cÖ_g weµq g~j¨ = 90UvKv|
Avevi 20% K‡g 2q µqg~j¨ hLb 80UvKv ZLb 40% jv‡f 2q weµqg~j¨ 80UvKvi 140% = 112UvKv|
GLb `yB weµqg~‡j¨i g‡a¨ cv_©K¨ 112-90 = 22UvKv n‡j µqg~j¨ = 100 UvKv|
100  55 Shortcut: 22% = 55
myZivs `yB weµqg~‡j¨i g‡a¨ cv_©K¨ 55 UvKv n‡j µqg~j¨ = = 250UvKv| n‡j 100% = 250|
22
2. `ywU msL¨vi AbycvZ 3:4 Ges Zv‡`i j.mv.¸ 180 n‡j cÖ_g msL¨vwU KZ? [DAE-(Store Keeper)-2017]
a. 30 b. 45 c. 50 d. 60 Ans:b
Solution:
awi, msL¨v `ywU h_vµ‡g 3K Ges 4K|
Ges Zv‡`i j.mv.¸ = 12K|
cÖkœg‡Z,
12K = 180 K = 15 myZivs cÖ_g msL¨wU = 315 = 45|
3. A I B Gi Av‡qi AbycvZ 3:2 Ges Zv‡`i e¨v‡qi AbycvZ 5:3| hw` Zviv cÖ‡Z¨‡K 1000 UvKv K‡i mÂq K‡i Z‡e
A Avq KZ? [DAE-(Store Keeper)-2017]
a. 6000 UvKv b. 7000UvKv c. 8000UvKv d. 10000 UvKv Ans: a
Solution:
A I B Gi Avq h_vµ‡g = 3K Ges 2K
Ges Zv‡`i e¨vq h_vµ‡g = 5L Ges 3L
cÖkœg‡Z,
3K-5L = 1000 - - - - (1)
2K-3L = 1000 - - - - (2)
-----------------------
1 bs mgxKi‡K 3 w`‡q ¸Y K‡i Ges 2bs mgxKiY‡K 5 w`‡q ¸Y K‡i cvIqv hvq
9K-15L = 3000 - - - - (1)
10K-15L = 5000 - - - - (2)
------------------------------------
K = 2000
myZivs A Gi Avq = 32000=6000UvKv| DËi: 6000 UvKv|
4. 1 eQi c~‡e© ¯§„wZ I cÖxwZi eq‡mi AbycvZ wQj 4:3| GK eQi c‡i Zv‡`i eq‡mi AbycvZ n‡e 5:4| Zv‡`i eZ©gvb
eq‡mi mgwó KZ eQi? [DAE-(Store Keeper)-2017]

Khairul’s IBA Math Book - 32

www.bdniyog.com
a. 15 b. 16 c. 17 d. 19 Ans: b
Solution:
awi,
1 eQi c~‡e© ¯§„wZ I cÖxwZi eqm h_vµ‡g 4K Ges 3K eQi|
myZivs Zv‡`i eZ©gvb eqm = 4K+1 Ges 3K+1
Ges 1 eQi ci Zv‡`i eqm n‡e 4K+2 Ges 3K+2
cÖkœg‡Z,
4K + 2 5
= ev, 16K+8 = 15K+10 K = 2
3K + 2 4
myZivs 1 Zv‡`i eZ©gvb eqm = (42+1) = 9 Ges (32+1) = 7 eQi|
Zv‡`i eq‡mi mgwó = 9+7 = 16 eQi|
5. Rwbi ¯‹z‡j hvIqvi mgq †eM N›Uvq 3 wKwg Ges evmvq †divi mgq †eM N›Uvq 2 wKwg| ¯‹z‡j hvIqv-Avmv‡Z hw` Zvi †gvU
5 N›Uv mgq jv‡M, Zvn‡j ¯‹zj I evmvi g‡a¨ `~iZ¡ KZ wKwg? [DAE-(Store Keeper)-2017]

a. 6 b. 7 c. 8 d. 9 Ans: a
Solution:
awi, `~iZ¡ = K wKwg
cÖkœg‡Z,
K K Shortcut: G ai‡Yi cÖkœ `ªæZ mgvavb Kivi Rb¨ MwZ‡eM؇qi j.mv.¸
 =5 †K `~iZ¡ a‡i wn‡me Ki‡Z nq| GLv‡b 2 I 3 Gi j.mv.¸ 6 B DËi|
3 2
2K  3K
ev,  5 ev, 5K = 30
6
K = 6wKwg myZivs `~iZ¡ 6 wKwg|
6. ‡mvbv cvwbi ‡P‡q 19 ¸Y fvix Ges Zvgv cvwbi †P‡q 9 ¸Y fvix| avZz `y‡Uv wK Abycv‡Z wgwkÖZ Ki‡j D³ wgkÖY cvwbi
†P‡q 15¸Y fvix n‡e? [DAE- (Store Keeper)- 2017]
a. 2:3 b. 4:3 c. 3:2 d. 3:4 Ans: c
Solution:
awi, †mvbv cvwbi †P‡q 19x Ges Zvgvi †P‡q 9y ¸Y fvix|
cÖkœg‡Z,
19x+9y= 15(x+y)
 19x + 9y = 15x + 15y
x 6
 19x – 15x = 15y – 9y  4x= 6y   x : y = 3:2 DËi: 3:2
y 4
Shortcut: by rule of allegation
1g Ask 2q Ask
19 9
Mo
6: 15
4 = 3:2

15-9=6 19-15=4

Khairul’s IBA Math Book - 33

www.bdniyog.com
7. wgbv ivRy I wgVz GKwU KvR h_vµ‡g 15w`b, 10 w`b I 6 w`‡b m¤úbœ Ki‡Z cv‡i| Zviv GK‡Î KvR Ki‡j H KvRwU m¤úbœ
n‡Z †gvU KZw`b mgq jvM‡e? [DAE-(Store Keeper)-2017]

a. 2w`b b. 3w`b c. 4w`b d. 6w`b Ans: b


Solution:
1 1 1 2  3  5 10 1
wgbv, ivRy Ges wgVz GK w`‡b Ki‡Z cv‡i,      Ask|
15 10 6 30 30 3
1
GLb, Ask Ki‡Z 1 w`b jvM‡j 1 Ask ev m¤ú~Y© KvRwU Ki‡Z mgq jvM‡e = 3 w`b|
3
8. ‡Kvb cixÿvq 35% QvÎ GK wel‡q 42% QvÎ Ab¨ wel‡q ‡dj Ki‡jv Ges 15% QvÎ Dfq wel‡q †dj Kij| hw` 2500
Rb QvÎ cixÿvq AskMÖnY K‡i Zvn‡j KZ Rb QvÎ †h †Kvb GKwU‡Z †dj K‡i‡Q? [DAE-(Store Keeper)-2017]

a. 1000 b. 1100 c. 1175 d. 1250 Ans: c


Solution:
ïay GK wel‡q †dj = 35% -15% = 20%
ïay Ab¨ wel‡q †dj = 42% - 15% = 27%|
Zvn‡j ïay GK wel‡q †dj Kiv QvÎ = 20%+27% = 47% (wP‡Î Kv‡jv Ask) 20% 15% 27%
GK ‡h †Kvb GK wel‡q †dj Kiv Qv‡Îi msL¨v = 2500 Gi 47% = 1175 Rb|
(D‡jøL¨ †h, hviv ïay GK wel‡q †dj ZvivB ïay GK wel‡q cvk|
KviY `ywU wel‡qi g‡a¨ ïay GKwU wel‡q ‡dj Kiv A_© Ab¨ welqwU‡Z cvk|)
9. kni A ‡_‡K kni B Gi 2wU wU‡KU Ges kni A ‡_‡K kni C Gi 3wU wU‡K‡Ui g~j¨ 77 UvKv| Avevi kni A ‡_‡K B
Gi 3wU wU‡KU Ges kni A ‡_‡K C Gi 2wU wU‡K‡Ui g~j¨ 73 UvKv| kni A ‡_‡K B Ges kni A ‡_‡K C Gi wU‡K‡Ui
g~j¨ h_vµ‡g KZ? [DAE-(Store Keeper)-2017]
a. 13 UvKv, 17UvKv b. 17 UvKv, 13 UvKv c. 9UvKv, 11UvKv d. 11UvKv, 9UvKv Ans: a
Solution:
awi,
kni A †_‡K kni B ‡Z hvIqv wU‡K‡Ui g~j¨ = x UvKv|
Ges kni A †_‡K kni C ‡Z hvIqv wU‡K‡Ui g~j¨ = y UvKv|
cÖkœg‡Z,
2x+3y=77…………(i)
3x+2y=73…………(ii)
(i) bs mgxKiY‡K 2 Ges (ii) bs mgxKiY‡K 3 w`‡q ¸Y K‡i cvIqv hvq|
4x+6y=154
9x+6y=219 …………………………………. Shortcut: G ai‡Yi cÖkœ mgxKiY mvwR‡q mgvavb bv K‡i
5x=65 (we‡qvM K‡i|) Ackb a‡i wn‡me Ki‡j mn‡R DËi †ei n‡e|
 x=13
Avevi 2x+3y=77  213+3y=77  3y = 51 y = 17
myZivs A †_‡K B Gi fvov 13UvKv Ges A †_‡K kni C Gi fvov 17 UvKv|
10. GKRb K…lK Zvi eM©vK…wZ evMv‡bi Pvicv‡k †eov w`j| ‡m cÖwZ cv‡k^© 27wU K‡i LyuwU emv‡j, Zvi me©‡gvU KZwU LyuwUi
cÖ‡qvRb n‡qwQj? [DAE-(Store Keeper)-2017.
a. 108 b. 107 c. 106 d. 104 Ans:d

Khairul’s IBA Math Book - 34

www.bdniyog.com
Solution:
GiKg cÖkœ¸‡jv `y fv‡e Kiv hvq|
cÖ_‡g me¸‡jv KY©v‡i GKwU K‡i LyuwU emv‡j †gvU 4wU|
Ges cÖwZ jvB‡b 27wU †gjv‡bvi Rb¨ `yÕ KY©v‡ii `ywUi gv‡S 25wU K‡i Pvicv‡k †gvU
100wU LyuwU jvM‡e| Zvn‡j †gvU LyuwU = 100+4 = 104 wU|
A_ev cÖ_‡gB 274 = 108wU †_‡K Pvi‡Kv‡Yi 4wU wiwcU n‡q hvIqvq 4 we‡qvM Ki‡j 108-4 = 104wU|
11. ‡kvfv cÖwZwU 70UvKv `‡i wKQz LvZv Ges cÖwZwU 30UvKv `‡i wKQz Kjg wKbj| †m †gvU 810 UvKv LiP K‡i Ges
m‡e©v”P hZ¸‡jv LvZv †Kbv m¤¢e ZZ¸‡jvB LvZv µq K‡i | Zvi µxZ LvZv I Kj‡gi AbycvZ KZ? [DAE-(Store
Keeper)-2017]
a. 4:3 b. 2:3 c. 3:4 d. 3:2 Ans: d
Solution:
m‡e©v”P msL¨K LvZv wKb‡Z n‡e|
GLb me¸‡jvB LvZv wKb‡j UvKv Awekó Zv‡K 81070 =11wU †Kbvi ci Aewkó = 40UvKv|
40UvKv Aewkó †_‡K GKwU Kjg †Kbvi ciI 10UvKv _vK‡e|
Zvn‡j Av‡iv 20UvKv jvM‡e bZzb GKwU Kjg †Kbvi Rb¨|
GLb GKwU LvZv Kg wKb‡j Zv w`‡q `ywU Kjg †Kbvi ciI 10UvKv Aewkó _vK‡e|
‡h‡nZz 20UvKv jvM‡e Zvn‡j `ywU LvZv bv wK‡b †mB UvKv w`‡q Kjg wKb‡j †mLvb †_‡K ‡h 20UvKv Aewkó _vK‡e
Zv Av‡Mi 10UvKvi mv‡_ †hvM K‡i Av‡iv GKwU Kjg †Kbv hv‡e|
Zvn‡j LvZv wK‡bwQj 9wU = 970 = 630 UvKv Ges Aewkó UvKv 810-630 = 180 UvKv w`‡q 18030 = 6wU|
LvZv I Kj‡gi AbycvZ = 9:6 ev 3:2|
12. x I y Dfq we‡Rvo msL¨v nq, Zvn‡j wb‡Pi †KvbwU Aek¨B †Rvo n‡e? [DAE-(Store Keeper)-2017]
a. xy+2 b. x+y c. xy d. x+y+1 Ans: b
Solution:
`ywU we‡Rvo msL¨v †hvM Ki‡j Zv Aek¨B †Rvo n‡e| †hgb: 3 I 5 Gi †hvMdj 3+5 = 8|
myZivs cÖ`Ë Ack‡bi g‡a¨ x+y B mwVK|
13. GK e¨vw³‡K Zvi †gvU F‡Yi cÖ_g 600 UvKvi Rb¨ 8% nv‡i my` w`‡Z nq Ges 600 UvKvi AwaK AskwUi Rb¨ 7%
nv‡i my` w`‡Z nq| hw` †Kvb eQi Zvi F‡Yi cwigvY 6000 UvKv nq, †m eQi Zv‡K KZ UvKv my` w`‡Z n‡e? [DAE-
(Store Keeper)-2017]
a. 480 b. 420 c. 378 d. 426 Ans: d
Solution:
6000 UKvi g‡a¨ cÖ_g 600 UvKvi my` = 600 Gi 8% = 48UvKv|
Avevi c‡ii 6000-600 = 5400 UvKvi my` = 5400 Gi 7% = 378UvKv|
Zvn‡j †gvU my`: 48+378 = 426 UvKv|
gy‡L gy‡L GK jvB‡b Kivi Rb¨: 6000UvKviB my` 7% n‡j n‡Zv 420UvKv Gi mv‡_ cÖ_g 600UvKvi Rb¨ 1UvKv K‡i
6UvKv AwZwi³ n‡j †gvU my` n‡e 420+6 = 426UvKv|

====================================

Khairul’s IBA Math Book - 35

www.bdniyog.com
Bangladesh Agricultural Development Corporation (BADC)
Post name: Administrative Officer Post: Exam date: 27-10-2017
Exam Taker: IBA, DU
1. GKRb wUwf we‡µZv 45% jv‡f wUwf wewµ KiZ| g›`vi Kvi‡Y †m Zvi jv‡fi nvi 40% K‡i Ges G‡Z Zvi weµq
10% †e‡o hvq| Zvi bZzb jvf I Av‡Mi jv‡fi AbycvZ KZ? [BADC (AO)-2017]
a. 9:8 b. 11:10 c. 45:44 d. 44:45 Ans: d
Solution:
awi, Av‡Mi wewµ 10wU wUwf Ges cÖwZwU‡Z jvf = 45UvKv| Zvn‡j †gvU jvf = 1045 = 450UvKv|
Avevi, 10wUi wewµ evo‡j bZzb wewµ 11wU Ges cÖwZwU‡Z 40UvKv K‡i bZzb †gvU jvf = 1140 = 440UvKv|
bZzb jvf I Av‡Mi jv‡fi AbycvZ = 440:450 ev 44:45 UvKv|
2. hw` K2 + L2 = 4 Ges K2 - L2 = - 4 nq, Zvn‡j K 4 + L4 Gi gvb KZ n‡e ? [BADC (AO)-2017]
a. 16 b. -16 c. 18 d. -8 Ans: a
Solution:
K2 + L2 = 4
K2 - L2 = - 4
--------------------------------
Shortcut:
2K2 = 0 [ †hvM K‡i]
`y wU eM© msL¨vi †hvMdj 4 n‡j Zv Aek¨B 2 I 2 A_ev 0 I 2 n‡Z cv‡i|
myZivs K2 = 0 K = 0 wKš‘ Zv‡`i e‡M©i we‡qvMdj -4 n‡j Aek¨B GKwU msL¨v 0 Ges AciwU 2 n‡Z n‡e|
Avevi, ZvB cvIqvi 4 w`‡j 04 +24 = 16 B n‡e|
2L2 = 8 [we‡qvM K‡i]
ev, L 2 = 4 L = 2
myZivs
K 4 + L4 = 04 +24 = 16 [(a+b)(a-b) = a2-b2 myÎ cÖ‡qvM K‡iI Kiv hvq|]
3. wibv gx‡gi †_‡K 10 eQ‡ii eo| 7 eQi ci, wibvi eqm gx‡gi eq‡mi wظY n‡e| wibvi eZ©gvb eqm KZ ? [BADC
(AO)-2017]
a. 17 b. 16 c. 13 d. 11 Ans: c
Solution:
awi, wibvi eqm = x eQi|
Zvn‡j gx‡gi eqm = x-10 eQi|
cÖkœg‡Z,
x+7 = 2(x-10+7) [gxg ‡QvU ZvB gx‡gi mv‡_ 2¸Y]
ev, x+7 = 2x-6
 x = 13 Ans:
4. wZbwU avivevwnK c~Y©msL¨vi ¸Ydj 120 n‡j G‡`i †hvMdj KZ ? [BADC (AO)-2017]
a. 9 b. 14 c. 12 d. 15 Ans: d
Solution:
120 ‡K fv½‡j cvIqv hvq 22235 GLb wZbwU avivevwnK msL¨v evbv‡bvi Rb¨ G‡`i‡K evbv‡Z n‡e 456
myZivs †hvMdj: 4+5+6 = 15|

Khairul’s IBA Math Book - 36

www.bdniyog.com
5. 8 R‡bi GKwU `‡j 65 †KwR IR‡bi GKR‡bi cwie‡Z© bZzb GKRb †hvM †`qvq Zv‡`i Mo IRb 2.5 †KwR †e‡o
hvq| bZzb e¨w³i IRb KZ †KwR ? [BADC (AO)-2017]
a. 45 b. 76 c. 80 d. 85 Ans: d
Solution:
8 R‡bi M‡o 2.5 †KwR K‡i evo‡j †gvU IRb evo‡e 82.5 = 20 †KwR|
Zvn‡j bZzb e¨vw³i IRb = 65+20=85 †KwR|
6. ˆ`wbK 8 N›Uv KvR K‡i 3wU cv¤ú 1wU Rjvavi‡K 2w`‡b Lvwj Ki‡Z cv‡i| Rjvavi‡K 1w`‡b Lvwj Ki‡Z n‡j 4wU
cv‡¤úi ˆ`wbK KZ N›Uv KvR Ki‡Z n‡e ? [BADC (AO)-2017]
a. 10 b. 12 c. 15 d. 16 Ans:b
Solution:
3wU cv¤ú w`‡q 2 w`‡b Lvwj Ki‡Z n‡j KvR Ki‡Z n‡e = 8 N›Uv
1 ÕÕ ÕÕ 1 ÕÕ ÕÕ ÕÕ ÕÕ ÕÕ ÕÕ ÕÕ = 832 (Kg cv¤ú w`‡q †ewk mgq Ges Kg w`‡b Av‡iv
†ewk| )
8 3 2
4 ÕÕ ÕÕ 1 ÕÕ ÕÕ ÕÕ ÕÕ ÕÕ ÕÕ ÕÕ = =12N›Uv|
4
7. 1056 Gi mv‡_ me©wb¤œ KZ †hvM Ki‡j †hvMdj 23 Øviv wb:‡k‡l wefvR¨ n‡e ? [BADC (AO)-2017]
a. 2 b. 3 c. 18 d. 21 Ans: a
Solution:
23 w`‡q 1056 †K fvM Ki‡j fvM‡kl _vK‡e = 21 Zvn‡j ‡`Lv hv‡”Q 23-21 = 2 NvUwZi Kvi‡Y Av‡iKevi 23 w`‡q
fvM Kiv hv‡”Q bv| myZivs 1056 Gi mv‡_ †hvM Ki‡Z n‡e 2|
8. GKwU Aswk`vix e¨emv‡qi †gvU gybvdv †_‡K ÔKÕ 40%, ÔLÕ 25%, ÔMÕ 20%, ÔNÕ 10%, Ges ÔOÕ 5% cvq| ÔKÕ Gi
gybvdv ÔLÕ Gi gybvdvi kZKiv KZ Ask ? [BADC (AO)-2017]
a. 62.5% b. 120% c. 160% d. 175% Ans: c
Solution:
GLv‡b K Gi 40% Ges L Gi 25%
Zvn‡j L Gi 25G hw` K Gi 40 nq
40  100
myZivs kZKiv nvi n‡e =160%
25
9. ‡Kvb GK KviLvbvq Kgx©‡`i RbcÖwZ ˆ`wbK gRywi wQj 100 UvKv| g›`vi Kvi‡Y Zv‡`i ˆ`wbK gRywi 50% Kgv‡bv
n‡qwQj| m¤úªwZ ˆ`wbK gRywi 60% evov‡bv n‡q‡Q| eZ©gv‡b RbcÖwZ ˆ`wbK gRywi KZ ? [BADC (AO)-2017]
a. 160 b. 110 c. 80 d. 60 Ans: c
Solution:
100 UvKv †_‡K 50% gRywi Kgv‡j bZzb gRywi 50 UvKv| GLb GB 50 UvKvi 60% ev 30UvKv evo‡j bZzb gRywi n‡e
50+30 = 80 UvKv|
10. GKwU wngvMvi‡K bevqb Kivi mgq Gi ˆ`N©¨ 30% I cÖ¯’ 50% evov‡bv nj Ges Gi D”PZv 20% Kgv‡bv nj|
bevqbK…Z wngvMv‡ii AvqZb cy‡iv‡bv wngvMv‡ii †_‡K kZKiv KZ Ask †ewk ? [BADC (AO)-2017]
a. 56% b. 50% c. 45 % d. 65% Ans: a
Solution:
cÖ_‡g ˆ`N¨©, cÖ¯’ I D”PZv 10, 10 Ges 10 n‡j AvqZb = 101010 = 1000
bZzb ‰`N©¨, cÖ¯’ I D”PZv = 13,15 Ges 8 myZivs bZzb AvqZb = 13158 = 1560

Khairul’s IBA Math Book - 37

www.bdniyog.com
560  100
AvqZb e„w× cvq 1560-1000 = 560 | e„w×i kZKiv nvi =  56%
1000
11. 36 msL¨vwUi †gvU KZ¸‡jv fvRK i‡q‡Q ? [BADC (AO)-2017]
a. 6 b. 8 c. 9 d. 10 Ans: c
Solution:
36 †K fv½‡j cvIqv hvq
49 = 2232 GLb GKB msL¨vi Dc‡ii cvIqvi¸‡jvi mv‡_ 1 †hvM K‡i Zv‡`i ¸Ydj nj 33 = 9|
12. GKwU QvÎvev‡m hZRb QvÎ _v‡K, Zv‡`i cÖ‡Z¨‡Ki gvwmK LiP Zv‡`i †gvU msL¨vi `k¸Y| H QvÎvev‡mi †gvU gvwmK
LiP 6,250 UvKv n‡j H QvÎvev‡m KZRb QvÎ _v‡K ? [BADC (AO)-2017]
a. 15 b. 25 c. 35 d. 45 Ans: b
Solution:
awi, QvÎ msL¨v = x Rb|
Zvn‡j Zv‡`i cÖ‡Z¨‡Ki gvwmK LiP = 10x UvKv|
cÖkœg‡Z,
x10x = 6250 ev, x2 = 625  x = 25
13. 13, 17, 25, 41 ....... avivwUi cieZx© msL¨vwU KZ ? [BADC (AO)-2017]
a. 73 b. 89 c. 101 d. 145 Ans: a
Solution:
13, 17, 25, 41 - - - avivwUi gv‡Si e¨eavb ¸‡jv n‡jv 4,8,16 myZivs Gi c‡ii msL¨vi mv‡_ e¨eavb n‡e 32 Ges
msL¨vwU n‡e 41+32 = 73|
14. ‡Kv‡bv K¬v‡m evjK‡`i Mo eqm evwjKv‡`i msL¨vi wظY| 36 R‡bi H K¬v‡m evjK I evwjKv‡`i msL¨vi AbycvZ 5.1 |
H K¬v‡mi evjK‡`i †gvU eqm KZ eQi ? [BADC (AO)-2017]
a. 300 b. 320 c. 360 d. 400 Ans: c
Solution:
5
5+1 = 6 As‡ki g‡a¨ ‡gvU evjK = 36 = 30 Rb Ges evwjKv = 36-30 = 6 Rb|
6
evjK‡`i Mo eqm = 12eQi | myZivs evjK‡`i †gvU eqm = 3012 = 360eQi|
15. ivRy GKwU eB nvmv‡bi Kv‡Q 10% ÿwZ‡Z wewµ Kij| ivRy hw` eBwU 20% Kg `v‡g wKbZ Ges 44 UvKv †ewk `v‡g
wewµ KiZ, Zvn‡j Zvi 40% jvf nZ| ivRy eBwU KZ `v‡g wK‡bwQj? [BADC (AO)-2017]
a. 50 b. 100 c. 200 d. 500 Ans: c
Solution:
awi, cÖ_g µqg~j¨ = 100UvKv|
Zvn‡j cÖ_g weµqg~j¨ = 100-10 =90UvKv| Shortcut: 22% = 44 n‡j 100% = 200
Avevi 2q µqg~j¨ = 100-20 = 80UvKv|
Avevi 40% jv‡f 2q weµqg~j¨ = 80+80 Gi 40% = 112 UvKv|
GLb `yB weµqg~‡j¨i e¨veavb = 112-90=22UvKv
e¨veavb 22UvKv n‡j µqg~j¨ = 100UvKv
myZivs e¨eavb 44UvKv n‡j µqg~j¨ n‡e =200 UvKv|

Khairul’s IBA Math Book - 38

www.bdniyog.com
www.bdniyog.com
16. 240 Rb †jvK GKwU eb‡fvR‡b hvq| †mLv‡b hZRb gwnjv wQj Zvi †_‡K 20 Rb cyiæl †ewk wQj| Avevi hZRb wkï
wQj Zvi †_‡K 20 Rb cÖvßeq®‹ †ewk wQj| eb‡fvR‡b KZRb cyiæl wQj? [BADC (AO)-2017]
a. 100 b. 140 c. 145 d. 75 Ans: d
Solution:
g‡b Kwi, cyiæl = x
gwnjv = x - 20 (cyiæl †_‡K gwnjv 20 Rb Kg|)
Ges wkï = (x + x – 20) – 20 (cyiæl gwnjv wg‡j cÖvß eq¯‹ hv‡`i †_‡K wkï 20 Rb Kg|)
= 2x– 40
cÖkœg‡Z,
x + (x – 20) + (2x – 40) = 240 (cyiæl+gwnjv+wkï = 240)
4x – 60 = 240  4x = 300  x = 75  cyiæl = 75
17. GKRb wVKv`vi 1920 wgUvi `xN© iv¯Ív 120 w`‡b wbg©vY K‡i †`qvi Rb¨ 160 Rb kÖwgK wb‡qvM Kij| 24 w`b ci, gvÎ
1
Ask KvR m¤úbœ nj| wba©vwiZ mg‡qi g‡a¨ KvR †kl Ki‡Z n‡j AwZwi³ KZRb †jvK wb‡qvvM Ki‡Z n‡e?
8
[BADC (AO)-2017]
a. 110 b. 160 c. 180 d. 120 Ans: d
Solution:
1
wbg©vY n‡q †M‡Q 1920 Gi = 240wgUvi|
8
wbg©vY Aewkó = 1920-240 = 1680wgUvi Ges w`b Aewkó 120-24 = 96
24 w`‡b 240wgUvi wbgvY© Ki‡Z †jvK jv‡M = 160 Rb|
160  24
1 ÕÕ 1 ÕÕ ÕÕ ÕÕ ÕÕ ÕÕ = (Kg w`‡b †ewk †jvK ZvB ¸Y Ges Kg Kv‡R Kg †jvK ZvB fvM)
240
160  24  1680
96 ÕÕ 1680 ÕÕ ÕÕ ÕÕ ÕÕ = (24Gi wecix‡Z 96 Ges 240 Gi wecixZ cv‡k 1680)
240  96
= 280 Rb|
AwZwi³ †jvK jvM‡e 280-160= 120Rb|

weKí mgvavb:
fMœvsk a‡iI GB cÖkœwU mgvavb Kiv hvq| Gfv‡e;
1 1 7
wbgvY n‡q †M‡Q, Ask| Zvn‡j Aewkó KvR Av‡Q 1- = Ask|
8 8 8
GLb HwKK wbq‡g mvRv‡bv hvq,
1
24 w`‡b Ask wbgvY© Ki‡Z †jvK jv‡M = 160 Rb|
8
1 ÕÕ 1 ÕÕ ÕÕ ÕÕ ÕÕ ÕÕ = 160248 (Kg w`‡b †ewk †jvK ZvB ¸Y Ges 1 Ask Av‡iv †ewk ZvB
Avevi Dwë‡q ¸Y)
7 160  24  8  7 7
96 ÕÕ Ask| ÕÕ ÕÕ ÕÕ ÕÕ = (24 Gi wecix‡Z 96 Ges mivmwi ¸Y|)
8 96  8 8
= 280 Rb|
AwZwi³ †jvK jvM‡e 280-160= 120Rb|

Khairul’s IBA Math Book - 39

www.bdniyog.com
‡kLvi gZ welq n‡jv ‡h:
cÖ_g evi fMœvsk Avm‡j Dwë‡q ¸Y nq Avi wØZxqevi fMœvsk Avm‡j mivmwi ¸Y nq| cÖkœwU A‡b¸‡jv wjwLZ cixÿvq
ûeû Avmvi Kvi‡Y A‡bK †ewk ¸iZ¡c~Y© |
18. GKRb e¨w³ GKwU eM©‡ÿÎ Ry‡o Avov Avwofv‡e ‡nu‡U wM‡qwQj| cÖvšÍ eivei bv nvuUvi Kvi‡Y KZ kZvsk Kg nvuU‡Z
n‡qwQj ? [BADC (AO)-2017]
a. 20% b. 25% c. 32% d. ‡KvbwUB bq Ans: d
Solution:
awi, eM©‡ÿÎwUi GK evû = 10wgUvi Zvn‡j cÖvšÍ eivei †M‡j nvU‡Z n‡e = 10+10 = 20wg.
Avevi KY© eivei †h‡Z n‡j nvU‡Z n‡e 2  10  14.14
6  100
Zvn‡j Kg nvuU‡Z n‡e, 20-cÖvq 14 = 6 kZKiv nvi  30% cÖvq|
20
19. GKwU †kÖYxi cÖwZ †e‡Â 4 Rb K‡i QvÎ em‡j 3wU †e Lvwj _v‡K| wKš‘ cÖwZ †e‡Â 3 Rb K‡i em‡j 6 Rb Qv‡Îi
`vuwo‡q _vK‡Z nq| G †kÖYxi QvÎ msL¨v KZ ? [BADC (AO)-2017]
a. 50 b. 60 c. 70 d. 80 Ans:b
Solution:
g‡b Kwi, QvÎmsL¨v = x 4 Ges 3 Dfq w`‡qB fvM Kiv hvq Ggb
x x6 msL¨v gvÎ 1wUB Av‡Q hv n‡jv 60|
cÖkœg‡Z, +3 = [KviY †e msL¨v mgvb]
4 3
x  12 x  6
ev = ev 4x-24=3x+36  x = 60
3 3
20. hw` n GKwU †Rvo msL¨v nq Z‡e wb‡Pi †KvbwU †Rvo msL¨v n‡Z cvi‡e bv ? [BADC (AO)-2017]
a. n2 b. 3 (n+1)+3 c. 2n+2 d. 2n+3 Ans: d
Solution:
Avgiv Rvwb †h †Kvb msL¨vi mv‡_ †Rvo msL¨v ¸Y Ki‡j Zv †Rvo n‡q hvq| ZvB GLv‡b 2n Aek¨B ‡Rvo msL¨v| Avevi
hw` 2n ‡Rvo msL¨v nq Zvn‡j Zvi mv‡_ Av‡iKwU †Rvo msL¨v †hvM Ki‡j †RvoB _vK‡e| wKš‘ we‡Rvo msL¨v †RvM
Ki‡j Zv we‡Rvo n‡q hv‡e| ZvB 2n+3 Aek¨B we‡Rvo n‡e|

====================================

Khairul’s IBA Math Book - 40

www.bdniyog.com
Bangladesh Agricultural Development Corporation (BADC)
Post name: Assistant Cashier. Exam Date: 11-08-2017
Exam Taker: IBA, DU

1. GK e¨vw³ 40 w`‡b Zvi `vjv‡bi KvR †kl Kivi Rb¨ 25 Rb †jvK wb‡qvM w`‡jb| 20 w`b ci wZwb Av‡iv 15 Rb
†jvK wb‡qvM w`‡jb Ges KvRwU 5w`b Av‡M †kl n‡q †Mj| AwZwi³ †jvK wb‡qvM bv w`‡j wZwb wba©vwiZ mg‡qi KZw`b
c‡i KvRwU †kl Ki‡Zb? [BADC (AC)-2017]
K. 2 w`b L. 3 w`b M. 4 w`b N.5 w`b O. †KvbwUB bq DËi: 4w`b|

mgvavb: (mvaviY HwKK wbq‡g mgvavb|)


w`b Aewkó Av‡Q = 40-20= 20 w`b
5 w`b Av‡M †kl nq = 20- 5 = 15 w`‡b
AwZwi³ †jvK wb‡qvM = 25+15 = 40 Rb|
40 Rb KvRwU K‡i = 15 w`‡b (Av‡Mi 25 + bZzb 15 Rb mn KvRwU Ki‡j 15 w`‡b †kl nq)
1 Ó Ó Ó = 1540 ÕÕ
15  40
25 Ó Ó Ó = (bZzb 15 Rb wb‡qvM bv w`‡q Av‡Mi 25 Rb B Ki‡j 24 w`b jvM‡Zv|)
25
= 24 w`b
AwZwi³ †jvK wb‡qvM bv w`‡j mgq †ewk jvMZ = 24 - 20 = 4 w`b DËi t 4 w`b

weKí mgvavb: (Kv‡Ri BDwbU a‡i GgwmwKD cixÿvi Rb¨ 20 †m‡K‡Û mgvavb|)
KvRwU‡Z jvMv †gvU gRy‡ii cwigvb †ei n‡e Gfv‡e,
25 Rb cÖ_g 20 w`‡b K‡i 2520 = 500 BDwbU
Avevi 20 w`b ci 15 Rb Avmvq 25+15 = 40 Rb hLb K‡i ZLb KvRwU †kl nq 40-5 = 35 w`‡b|
myZivs 40 Rb Ki‡jv 35-20 = 15 w`‡b †gvU 4015 = 600 BDwbU|
A_©vr †gvU KvRwU 500+600 = 1100 BDwb‡Ui|
Zvn‡j 25 Rb GKUvbv KvR Ki‡j jvM‡Zv 110025 = 44 w`b|
myZivs KvRwU †kl n‡Z †jU n‡Zv 44-40 = 4 w`b|

weKí mgvavb: (K a‡i mgxKiY mvwR‡q mgvavb)


awi, KvRwU †kl Ki‡Z ‡gvU mgq jvM‡e K w`b|
Zvn‡j 25 Rb‡K †gvU jvM‡e = 25K w`b|
cÖkœg‡Z,
25K = (2025) +(1540) =(25 Rb 20w`b KvR Kivi ci 25+15 = 40 Rb K‡i‡Q 15w`b, KviY †gvUw`b 40-
5=35)
ev, 25K = 1100
myZivs K = 44 w`b|
AwZwi³ †jvK jvMv‡bv bv n‡j mgq †ewk jvM‡Zv: 44-40 = 4 w`b|

Khairul’s IBA Math Book - 41

www.bdniyog.com
2. 40 ‡KwR ˆRe I BDwiqv mv‡ii wgkÖ‡Y ˆRe mv‡ii cwigvY 10%| KZ †K‡wR mvi wgkv‡j bZzb wgkÖ‡Y ˆRe mv‡ii
cwigvY 20% n‡e? [BADC (AC)-2017]
K. 4 L.5 M.6 N.7 O. †KvbwUB bq

mgvavb: (L)
cÖ_‡g ˆRe I BDwiqv mv‡ii cwigvY = 4 I 36 †KwR
awi, ˆRe mvi †gkv‡Z n‡e = K †KwR gy‡L gy‡L: 10 †m‡K‡Û:
cÖkœg‡Z, BDwiqv: 80% = 36 n‡j ˆRe 20% = n‡e BDwiqvi
4+K = (40+K) Gi 20% cwigv‡Yi 4 fv‡Mi 1 fvM| A_©vr 36/4 = 9 †KwR|
ev, 20+5K = 40+K bZzb †gkv‡Z n‡e 9-4 = 5 †KwR|
ev, 4K = 20 K = 5 †KwR|

‡hfv‡e †f‡e Kbwd‡W›Uwj fzj Ki‡Z cv‡ib:


40 Gi 10% = 4 n‡j 20% = 8 n‡e Zvn‡j Av‡M wQj 4 ‡gkv‡Z n‡e 8-4 = 4 (wbwðZ fzj DËi)

‡Kb fzj??
KviY hLb ˆRe †gkv‡bv n‡e ZLb †gvU cwigvY I evo‡e| ZvB ‡gkv‡bvi ci 20% Avi †gkv‡bvi Av‡Mi 20% GK bv|
‡hgb: cÖ_‡g ‡gvU mvi = 40 Gi g‡a¨ ˆRe = 40 Gi 10% = 4 Ges BDwiqv = 40-4 = 36|
GLb 5 †KwR ˆRe mvi †gkv‡bvi ci †gvU mvi n‡e 40+5 = 45
Ges ˆRe mvi n‡e GB 45 Gi 20% = 9 (Av‡Mi 40+bZzb 5)|
wKš‘ hw` 4 †gkvb Zvn‡j †gvU cwigvY n‡e 40+4 = 44 hvi 20% = 8 nq bv eis: 8.8 nq|

Solution By rule of allegation in 10sec:


Profit on 1st part Profit on 2nd part
10% 0%
Mean profit
20%

20-0 20-10
st nd
Ratio of 1 and 2 parts = 20 :10 = 2 : 1
2 part = 10 So, 1 part = 5 Ans: 5
3. ‡Kvb msL¨vi wZb PZy_©vs‡ki GK cÂgvs‡ki gvb 60| msL¨vwU KZ? [BADC (AC)-2017]
K.300 L.400 M.500 N.600 O. †KvbwUB bq
mgvavb: (L)
GiKg cÖkœ †k‡li ‡_‡K wn‡me Ki‡j gy‡L gy‡L DËi ejv hvq|
‡k‡li 5 fv‡Mi 1 fv‡Mi gvb 60 n‡j 5 fv‡Mi gvb = 605 = 300|
GLb GB 300 n‡j 4 fv‡Mi 3 fv‡Mi gvb| Zvn‡j 1 fv‡Mi gvb n‡e 100 Ges 4 fvM ev msL¨vwU n‡e 400

Khairul’s IBA Math Book - 42

www.bdniyog.com
4. GKwU `ªe¨ 420 UvKvq µq K‡i 15% jv‡f weµq Kiv nj| weµqg~j¨ µqg~j¨ A‡cÿv KZ UvKv ‡ewk? [BADC
(AC)-2017]
K.42 UvKv L.21UvKv M.84UvKv N.63UvKv O. †KvbwUB bq

mgvavb: (N)
µqg~j¨ = 420
Ges weµqg~j¨ = 420+420 Gi 15% = 420+63 = 483 UvKv|
weµqg~j¨ I µqg~‡j¨i cv_©K¨ = 483-420 = 63 UvKv| (jv‡fi 63 UvKv B cv_©K¨)

5. `yBwU msL¨vi AbycvZ 5:7 Ges G‡`i †hvMdj 108| e„nËg msL¨vwU KZ? [BADC (AC)-2017]
K.42 L.49 M.56 N.63 O. †KvbwUB bq
mgvavb: (N)
5+7 = 12 As‡ki gvb 108 n‡j 1 As‡ki gvb 10812 = 9
Zvn‡j eo msL¨vwU = 97 = 63

6. gvZv I Kb¨vi eq‡mi mgwó 60 eQi| 5 eQi Av‡M gvZvi eqm Kb¨vi eq‡mi 4 ¸Y wQj| 7 eQi ci gvZvi eqm KZ
n‡e? [BADC (AC)-2017]
K.35 L.40 M.42 N.47 O. †KvbwUB bq
mgvavb: (O)
eZ©gv‡b gvZv I Kb¨vi mgwó 60 eQi n‡j 5 eQi Av‡M mgwó wQj = 60-10 = 50 eQi| ZLb gvZvi eqm Kb¨vi 4
¸Y| Ges 7 eQi ci gvZvi eqm n‡e 45+7 = 52 eQi|

7. GKwU cvBc Øviv GKwU U¨vsK 3 N›Uvq c~Y© nq| wØZxq cvBcwU Øviv U¨vsKwU c~Y© n‡Z 6 N›Uv mgq jv‡M| `yBwU cvBc
GKmv‡_ †Q‡o ‡`qv n‡j U¨vsKwU c~Y© n‡Z KZ wgwbU jvM‡e? [BADC (AC)-2017]
K.60 L.90 M.120 N.140 O. †KvbwUB bq
mgvavb: (M)
1 1 21 3 1
cÖ_g I wØZxq cvBc w`‡q 1 N›Uvq c~Y© nq: =     Ask|
3 6 6 6 2
Zvn‡j m¤ú~Y© Ask c~Y© Ki‡Z mgq jvM‡e 2 N›Uv ev 120 wgwbU|

8. K, L Ges M GKwU e¨vemvq h_vµ‡g 36,000UvKv, 42,000 UvKv Ges 72,000 UvKv wewb‡qvM K‡i| GKeQi ci
g~jab Abycv‡Z jvf ewÚZ nq Ges L 1400 UvKv jvf cvq| K I M Gi jv‡fi mgwó KZ? [BADC (AC)-2017]
K.2500 L.3000 M.3300 N.3600 O. †KvbwUB bq
mgvavb: (N)
Zv‡`i wewb‡qv‡Mi AbycvZ = K:L:M = 36,000 : 42,000 : 72,000 ev 6:7:12
GLb L Gi 7 Ask = 1400 n‡j 1 As‡ki gvb = 14007 = 200
myZivs K+M Gi 6+12 = 18 As‡ki gvb n‡e 18200 = 3600UvKv|

9. wZbwU msL¨vi AbycvZ 4:5:6 Ges ga¨g msL¨vwUi eM© 225| e„nËg msL¨vwU KZ? [BADC (AC)-2017]
K.18 L.20 M.22 N.24 O. †KvbwUB bq
mgvavb: (K)
awi, msL¨v wZbwU h_vµ‡g, 4K, 5K Ges 6K

Khairul’s IBA Math Book - 43

www.bdniyog.com
cÖkœg‡Z,
(5K)2 =225 ev, 5K = 15 ev, K = 3, myZivs e„nËg msL¨vwU = 63 = 18

10. ‡Kvb cixÿvq 80% MwY‡Z I 70% evsjvq cvk Kij| Dfq wel‡q cvk Kij 60%| Dfq wel‡q kZKiv KZRb †dj
Kij? [BADC (AC)-2017]
K. 5% L.10% M.15% N.20% O. 25%
M Total =100%
B

mgvavb: (L) 80-60 70-60


Dfq wel‡q †dj = [100- (80+70- 60)]% = 10% =20% 60% =10% 90%
10%

11. `ywU msL¨vi ¸Ydj 1536| msL¨v `ywUi j.mv.¸ 96 n‡j Zv‡`i M.mv.¸ KZ? [BADC (AC)-2017]
K.16 L.12 M.24 N.18 O. 32
mgvavb: (K)
`ywU msL¨vi j.mv.¸  msL¨v `ywUi M.mv.¸ = msL¨v `ywUi ¸Ydj|
myZivs msL¨v `ywUi M.mv.¸ = ¸Ydj  j.mv.¸ ev 153696 = 16
( GLv‡b m¤ú~Y© fvM bv K‡i ïay †k‡li msL¨v 6 I 6 ‡`‡L 5 †m‡K‡Û †gjv‡bv hvq|)

12. hw` 330 UvKv 3 eQi c‡i my`-Avm‡j 429 UvKv nq, Zvn‡j 650 UvKv my‡` Avm‡j 5 eQi c‡i KZ UvKv n‡e?
[BADC (AC)-2017]
K. 825 L.875 M.900 N.975 O. †KvbwUB bq
mgvavb: (N)
cÖ_‡g 3 eQ‡ii ‡gvU my` = 429-330 = 99UvKv, myZivs 1 eQ‡ii my` = 993 = 33 UvKv|
Avevi 330 UvKvi 1 eQ‡ii my` = 33UvKv (10 fv‡Mi 1 fvM n‡j )
100 UvKvi 1 eQ‡ii my` n‡e 100UvKvi 10 fv‡Mi 1 fvM ev 10UvKv A_©vr my‡`i nvi 10% |
GLb 650 UvKvi 1 eQ‡ii my`I n‡e 10 fv‡Mi 1 fvM = 65UvKv Zvn‡j 5 eQ‡ii my` n‡e 655 = 325 UvKv|
myZivs my`-Avmj n‡e 650+325 = 975 UvKv|

13. GKwU eM©vKvi Rwgi GK cvk¦© gvcvi mgq fz‡j ˆ`‡N©¨ 10 kZvsk †ewk gvcv nq| †ÿÎd‡ji †ÿ‡Î fz‡ji cwigvY KZ
kZvsk? [BADC (AC)-2017]
K. 10% L.10.25% M.21% N.25% O. †KvbwUB bq
mgvavb: (M)
mvaviY fv‡e e‡M©i †ÿÎdj cwigv‡ci Rb¨ GK evû cwigvc K‡iB ‡ÿÎdj †ei Kiv nq| GLb GKwU e‡M©i GK evû
100 n‡j Zvi ‡ÿÎdj = 1002 ev 10000 | GLb hw` fzj K‡i hw` GK evû 10% evov‡bv nq A_©vr †h Rwg ‡g‡c‡Q
†m 100wgUvi Gi cwie‡Z© 110 wgUvi ej‡j †h wn‡me Ki‡e †m 1102 = 12100 †ÿÎdj †ei Ki‡e| G‡Z e‡M©i ‡ÿÎdj
evo‡e 10000 G 2100 ev 100 †Z 21%|

gy‡L gy‡L: e‡M©i GKevû evov gv‡bB `y evi evov‡bv| Zvn‡j cÖ_‡g 100 †_‡K 10 ‡e‡o 110 nq| Ges c‡i 110 Gi
10% A_©vr 11 ev‡o| Zvn‡j †gvU evo‡e 10+11 = 21%|
Khairul’s IBA Math Book - 44

www.bdniyog.com
14. 300 wgUvi `xN© GKwU gvjevnx †Uªb N›Uvq 72wK‡jvwgUvi †e‡M 25 †m‡K‡Û GKwU ‡mZz AwZµg K‡i | †mZzwUi ‰`N¨©
KZ? [BADC (AC)-2017]
K.200 wgUvi L.220 wgUvi M.250wgUvi N.300wgUvi O. †KvbwUB bq
mgvavb: (K)
5
‡Uª‡bi MwZ‡eM = 72 wKwg/N›Uv ev 72 =20wgUvi/‡m‡K‡Û| Zvn‡j 25 †m‡K‡Û ‡gvU hvq 2025 = 500wg.
18
‡h‡nZz †Uª‡bi ˆ`N©¨ = 300wgUvi Zvn‡j †mZzwUi ˆ`N¨© n‡e = 500-300 = 200 wgUvi|

15. `yBwU msL¨vi AbycvZ 5:8| cÖwZwU msL¨vi mv‡_ 15 †hvM Ki‡j G‡`i AbycvZ nq 10:13| msL¨v؇qi †hvMdj KZ?
[BADC (AC)-2017]
K.31 L.33 M.36 N.39 O. †KvbwUB bq
mgvavb: (N)
awi, msL¨v `ywU: = 5K Ges 8K
cÖkœg‡Z,
5K+15 : 8K + 15 = 10:13
mgvavb K‡i cvB K = 3
myZivs msL¨v `ywUi †hvMdj = 5K+8K = 13K = 133 = 39| DËi: 39
(gy‡L gy‡L: Ack‡bi g‡a¨ ïay 39 †K AbycvZ ؇qi †hvMdj 8+5 = 13 w`‡q fvM Kiv hvq| )

16. GKwU AvqZ‡ÿ‡Îi ˆ`N©¨ A‡cÿv cÖ¯’ 4 wgUvi Kg| Gi †ÿÎdj 192eM© wgUvi n‡j, cwimxgv KZ? [BADC (AC)-
2017]
K.48wgUvi L.64wgUvi M.52wgUvi N.56wgUvi O. †KvbwUB bq
mgvavb: (N)
awi, ‰`N¨© = K myZivs cÖ¯’ = K-4
cÖkœg‡Z, K  (K-4) = 192 ev, K2- 4K -192 = 0 ev, K-2-16K+12K -192 = 0 myZivs K = 16
‰`N©¨ 16 n‡j cÖ¯’ = 16-4 =12 I cwimxgv = 2(16+12) = 56

{gy‡L gy‡L : 1612 = 192 nq| †hLv‡b 16-12=4 myZivs DËi 2(16+12) = 56}

17. 12 Rb kÖwgK 3 w`‡b 720 UvKv Avq K‡i| 9 Rb kÖwgK mgcwigvY UvKv Avq K‡i KZw`‡b? [BADC (AC)-2017]
K.2w`‡b L.3w`‡b M.4 w`‡b N.5w`‡b O. †KvbwUB bq
mgvavb: (M) gy‡L gy‡L: 10 †m‡K‡Û:
12 Rb Avq Ki‡Z jv‡M = 3 w`b| cÖ_g `ywU msL¨v ¸Y K‡i c‡ii msL¨vwU w`‡q fvM|
1 Rb Avq Ki‡Z jv‡M = 312w`b| (12 Rb 720 UvKv Avq K‡i, Avevi 9 RbI 720 UvKv
3  12 Avq K‡i,ZvB 720UvKv wjL‡Z n‡e bv| )
9Rb Avq Ki‡Z jv‡M = = 4 w`b|
9

18. cvwbc~Y© GKwU Wªv‡gi IRb 20 †KwR| hw` Gi GK PZz_©vsk cvwbc~Y© _v‡K Zvn‡j Gi IRb nq 8 †KwR| Lvwj Wªv‡gi
IRb KZ †KwR? [BADC (AC)-2017]
K.3 ‡KwR L.4 †KwR M. 5 †KwR N.6 †KwR O. †KvbwUB bq
Khairul’s IBA Math Book - 45

www.bdniyog.com
mgvavb: (L)
3 1
Ask = 12 n‡j Ask = 4 n‡e| eZ©gv‡b cvwb Av‡Q 4 †KwR| myZivs Wªv‡gi IRb 8-4 = 4 †KwR|
4 4
1
19. GKwU Kv‡Ri Ask †kl nq 3 w`‡b| H Kv‡Ri 3 ¸Y KvR Ki‡Z KZw`b mgq jvM‡e? [BADC (AC)-2017]
23
K.69 L.207 M.138 N.23 O. 175
mgvavb: (L)
23 Zjv wewìs‡qi 1 Zjv evbv‡Z 3 w`b jvM‡j, cyiv wewìs evbv‡Z mgq jvM‡e 233=69w`b Ges GiKg 3wU wewìs
evbv‡Z mgq jvM‡e 693 = 207 w`b|]
20. GKwU ¯‹z‡ji †gvU QvÎQvÎx‡`i A‡a©K dzUej †L‡j Ges Aewkó‡`i GK Z…Zxqvsk †Uwbm †L‡j | Aewkó 300 Rb `ywU
†Ljvi †KvbwUB †L‡j bv| ¯‹z‡j KZRb QvÎ-QvÎx Av‡Q? [BADC (AC)-2017]
K.450 Rb L.600Rb M. 900 Rb N.1200Rb O. †KvbwUB bq
mgvavb: (M)
awi, †gvU QvÎ-QvÎx = K Rb
K K K 1 K K K 3K  K 2K K
dzUej †L‡j = ,Aewkó = , †Uwbm †L‡j = Gi = Aewkó = - = = =
2 2 2 3 6 2 6 6 6 3
K
cÖkœg‡Z, = 300 K = 900 Rb| DËi: 900 Rb|
3
gy‡L gy‡L †k‡li w`K †_‡K: 3 fv‡Mi 1 fvM wµ‡KU †Lj‡j wµ‡KU †L‡j bv 3 fv‡Mi 2 fv‡Mi gvb = 300 myZivs
wµ‡KU †L‡j 3 fv‡Mi 1 fvM = 150| GLvb wµ‡KU+‡KvbwUB bv = 150+300 = 450 n‡jv †gv‡Ui A‡a©K myZivs
†gvU QvÎ-QvÎx = 4502 = 900 Rb| ) DËi:

====================================

Khairul’s IBA Math Book - 46

www.bdniyog.com
Bangladesh Agricultural Development Corporation (BADC)
Post Name: Store Keeper Exam Date:11-08-2017
Exam Taker: IBA, DU

1. cÖwZwU 3600 UvKv K‡i `ywU †Uwej weµq Kiv nj| GKwU 20% jv‡f Ges Ab¨wU 20% ÿwZ‡Z weµq Kiv nj|
me©‡gvU KZ jvf ev ÿwZ n‡q‡Q? [BADC-(Store Keeper)-2017]
a. 200UvKv jvf b. 300UvKv jvf c. 600UvKv ÿwZ d. 300UvKv ÿwZ Ans: b
Solution:
wjwLZ mgvavb:
jv‡fi †ÿ‡Î ,
3600 UvKvi ‡Pqv‡ii µqg~j¨ n‡e
20% jv‡f 100 UvKvi †Pqv‡ii `vg = 120 UvKv|
GLb 120 UvKv weµqg~j¨ n‡j µqg~j¨ = 100 UvKv |
Avevi 1 ÕÕ ÕÕ ÕÕ ÕÕ
100
=
120
Avevi 3600 ÕÕ ÕÕ ÕÕ ÕÕ =
100  3600 = 3000 UvKv|
120
Avevi, ÿwZi ‡ÿ‡Î,
20% ÿwZ‡Z 100 UvKvi ‡Pqv‡ii weµqg~j¨ =80 UvKv
GLb weµqg~j¨ 80UvKv n‡j µqg~j¨ = 100UvKv
100
ÕÕ 1 ÕÕ ÕÕ ÕÕ
80
100  3600
ÕÕ 3600 ÕÕ ÕÕ ÕÕ = 4500 UvKv|
80
GLb †gvU µqg~j¨ = 3000+4500 = 7500 Ges †gvU weµqg~j¨ = 3600+3600 = 7200 myZivs †gvU ÿwZ 7500-
7200 = 300UvKv| A_ev jv‡fi mgq 600 jvf wKš‘ ÿwZi mgq 900 ZvB †gv‡U ÿwZ 900-600=300UvKv
Learning points: GgwmwKD cixÿvi Rb¨ Dc‡ii AsKwU `ªæZ mgvavb Kiv hvq Gfv‡e,
GiKg GKB `v‡g `ywU cY¨ weµq Kivq GKB nv‡i jvf ev ¶wZ n‡j, †gv‡Ui Dci memgq ¶wZ nq|
KZUzKz ¶wZ nq Zv †ei Kivi Rb¨ wb‡Pi m~ÎwU cÖ‡qvM Kiæb:
2
 mgvb mgvb jvf ev ÿwZi nvi 
¶wZi nvi =  %
 10 
2
 20 
20% K‡i jvf- -ÿwZ n‡j †gv‡Ui Dci ÿwZ =   % = 4% (Dc‡ii m~Î cÖ‡qvM K‡i)
 10 
GLb 3600 UvKv K‡i 2wU 7200 UvKvq wewµ Kivq 4% ÿwZ n‡j †gvU ÿwZ n‡e 4% Gi mgvb|
7200 7200  4
96% = 7200 1% = 4% =  300 UvKv [ †h‡nZz †gvU ÿwZ 4%]
96 96

Khairul’s IBA Math Book - 47

www.bdniyog.com
2. wZbwU avivevwnK we‡Rvo msL¨vi †hvMdj 177| ga¨g msL¨vwU KZ? [BADC-(Store Keeper)-2017]
a. 47 b. 59 c. 65 d. 69 Ans: b
Solution:
msL¨v wZbwUi Mo = 1773 = 59 B n‡jv ga¨vg msL¨v| KviY msL¨v wZbwU n‡e 58,59 Ges 60| †hLv‡b 59B ga¨g
3. ‡Kvb fMœvskwU ÿz`ªZg? [BADC-(Store Keeper)-2017]
a. 5/6 b. 12/15 c. 11/14 d. 17/21 Ans: c
Solution:
5 25 12 24
K I L Gi g‡a¨  Ges  myZivs cÖ_g fMœvskwU eo Ges ZvB †mwU ev` w`‡q c‡ii fMœvskwU ivwL|
6 30 15 30
11 33 17 34 11
Avevi, M I N Gi g‡a¨  Ges  GLv‡b Ackb N eo ZvB Zv ev` w`‡q ivwL|
14 42 21 42 14
12 11
GLb: Ges Gi g‡a¨ cÖ_g fMœvskwU eo|
15 14
KviY `ywU fMœvs‡ki gv‡Si cv_©K¨ mgvb n‡j †h fMœvs‡ki ni eo †mwUB eo
11
nq| Avevi †h‡nZz †k‡li Ack‡bi †k‡li fMœvskwU GKwU AcÖKZ … fMœvsk ZvB DËi |
14
(GLv‡b A‡bK¸‡jv wbq‡gi K_v ejv n‡jI G¸‡jv cÖ_‡g GKevi eyS‡Z n‡e|
A_ev AvovAvwo ¸Y K‡iI mn‡R †ei Kiv hvq)
4. ‡mŠif 8% nvi my‡` †gv‡gb‡K Ges 12% nvi my‡` iæ‡ej‡K mgcwigvY UvKv avi w`j| †m wZb eQi ci `yR‡bi KvQ
†_‡K me©‡gvU 720 UvKv my` †cj| †m cÖ‡Z¨K‡K KZ UvKv avi w`‡qwQj? [BADC-(Store Keeper)-2017]
a. 1200 UvKv b. 1400 UvKv c. 1600 UvKv d. 1800 UvKv Ans: a
Solution:
`yRb‡K 100 UvKv K‡i w`‡j 1 eQi ci my` cvIqv hv‡e 8+12 = 20UvKv
3 eQi ci cvIqv hv‡e 203 =60UvKv|
‡gvU my` 60 UvKv n‡j GKRb‡K w`‡qwQj = 100UvKv
‡gvU my` 720UvKv (60 Gi †_‡K 12¸Y †ewk) n‡j Avmj = 10012 =1200UvKv|
5. wcZv I cy‡Îi eq‡mi AbycvZ 4:1 Ges Zv‡`i eq‡mi ¸Ydj 256| 4 eQi ci Zv‡`i eq‡mi AbycvZ n‡e-[BADC-
(Store Keeper)-2017]
a. 3:1 b. 4:1 c. 5:1 d. 5:2 Ans: a
Solution:
awi, wcZv I cy‡Îi eqm h_vµ‡g, 4K I K eQi|
cÖkœg‡Z
4KK = 256
ev, 4K2 = 256 ev, K2 = 64 K = 8 myZivs Zv‡`i eqm = 48 = 32 eQi Ges 8 eQi|
4 eQi c‡i Zv‡`i eq‡mi AbycvZ n‡e 32+4:8+4 = 36:12 = 3:1
6. GKwU K‡j‡R 70% cwiÿv_©x Bs‡iRx‡ZI 80% cwiÿv_©x evsjvq cvm K‡i‡Q| wKš‘ 10% wkÿv_x© Dfq wel‡q †dj
K‡i‡Q| hw` Dfq wel‡q 300 Rb cwiÿv_©x cvm K‡i _v‡K, Z‡e H K‡j‡R KZ Rb cwiÿv_©x cixÿv w`‡q‡Q?
[BADC-(Store Keeper)-2017]
a. 300 b. 400 c. 500 d. 550 Ans: c

Khairul’s IBA Math Book - 48

www.bdniyog.com
Solution:
cÖ_‡g GKwU wel‡q †d‡ji nvi †ei Kiæb, ‡Kbbv Dfq wel‡q †d‡ji nvi †`qv Av‡Q 10%|
ZvB ïay Bs‡iRx †dj 100-70 = 30% Ges ïay evsjvq †dj = 100-80=20%
‡gvU †dj = 30+20 -10 = 40%, GB 40% wK??? GwU n‡jv †gvU †dj ( nq GK wel‡q A_ev Dfq wel‡q
†dj) Zvn‡j Dfq wel‡q cvm n‡jv 100-40% = 60% | GLb GB 60% = 300 Rb †Kbbv 300 Rb nj
100
Dfq wel‡q †gvU cvk Kiv wk¶v_©xi msL¨v| GLb 100% Gi gvb n‡e, 300 = 500 Rb
60
E Total =100%
B
cv‡ki wPÎwU eyS‡j GK jvB‡b Gfv‡e mgvavb Kiv hvq|
{100% - (30+20-10)} = 60%
GLb 60%= 300 n‡j 100% = 500 30-10 =20% 10% 20-10 =10%
40%
60%

7. GKwU _‡j‡Z 25 cqmv, 10 cqmv, I 5 cqmvi gy`ªv 3:4:5 Abycv‡Z Av‡Q| hw` me¸‡jv wgwj‡q 28UvKv nq, Zvn‡j 10
cqmvi gy`ªv KZwU? [BADC-(Store Keeper)-2017]
a. 80wU b. 60wU c. 100wU d. 110wU Ans: a
Solution:
awi,
_‡jwU‡Z 25 cqmvi gy`ªv Av‡Q 3K, 10 cqmvi gy`ªv Av‡Q 4K Ges 5 cqmvi gy`ªv Av‡Q 5KwU|
cÖkœg‡Z,
253K + 104K + 55K = 28100 [me¸‡jv †h‡nZz cqmv ZvB 28 UvKv‡K cqmv evbv‡bv n‡jv|]
75K+40K+25K = 2800
gy‡L gy‡L:
140K = 2800
Ackb ‡_‡K 10 cqmvi gy`ªvi AbycvZ 4 Ges 4 w`‡q wefvR¨
K = 20 msL¨vi g‡a¨ 80 wb‡jB 80wU 10 cqmvi gy`ªv 8UvKv Ges
GLb 10 cqmvi gy`ªv Av‡Q = 420 = 80wU| Ab¨ `ywU wg‡j 20 UvKv n‡j †gvU 28UvKv wg‡j hvq|

8. `yB AsK wewkó GKwU msL¨v, msL¨vwUi AsK؇qi †hvMd‡ji 4 ¸Y| msL¨vwUi mv‡_ 27 †hvM Ki‡j AsKØq ¯’vb cwieZ©b
K‡i| msL¨vwU KZ? [BADC-(Store Keeper)-2017]
a. 27 b. 36 c. 39 d. 45 Ans: b
Solution:
G ai‡Yi cÖkœ x a‡i mgvavb Kiv hvq| wKš‘ Zv‡Z A‡bK mgq jv‡M|
ZvB †h †Kvb wcÖwji mnR AsK¸‡jv Ackb a‡i mgvavb KivB DËg|
GLv‡b cÖ`Ë Ackb¸‡jvi g‡a¨ ïaygvÎ L †Z †`qv 36 Gi 3+6 = 9 Ges 36 n‡jv 9 Gi 4¸Y| GLb Ab¨¸‡jv a‡i bv
†f‡e GUv a‡i B wØZxq kZ© wgwj‡q †bB 36 Gi mv‡_ 27 †hvM Ki‡j 63 nq hv‡Z msL¨vwUi AsKØq ¯’vb wewbgq K‡i|
ZvB DËi: 36|

9. GKwU evm cÖwZw`b GKwU wbw`©ó MwZ‡Z 60 gvBj c_ AwZµg K‡i| GKw`b hvwš¿K ÎæwUi Kvi‡Y ev‡mi MwZ 10
gvBj/N›Uv K‡g hvq Ges H c_ AwZµg Ki‡Z 3 N›Uv †ewk jv‡M| evmwUi ¯^vfvweK MwZ KZ? [BADC-(Store
Keeper)-2017]
a. 20gvBj/N›Uv b. 25gvBj/N›Uv c. 32gvBj/N›Uv d. 45gvBj/N›Uv Ans:a

Khairul’s IBA Math Book - 49

www.bdniyog.com
Solution: gy‡L gy‡L:
awi, evmwUi MwZ‡eM = K Ack‡bi g‡a¨ ïay 20 w`‡qB 60 ‡K fvM Kiv hvq|
Zvn‡j 60gvBj †h‡Z cÖ_‡g mgq jv‡M 6020 = 3 N›Uv
Ges MwZ‡eM 10 gvBj Kg‡j mgq jv‡M 6010 = 6 N›Uv|
cÖkœg‡Z,
hv Av‡Mi mg‡qi †_‡K 3 N›Uv †ewk|
60 60
- = 3 [Av‡Mi mgq - MwZ Kgvi c‡ii mgq = 3 N›Uv]
K K  10
mgvavb K‡i cvIqv hvq, K = 20 gvBj|
10. wZbwU msL¨vi †hvMdj 110| cÖ_g msL¨vwU wØZxq msL¨vi wظY Ges Z…Zxq msL¨vwU cÖ_g msL¨vi GK Z…Zxqvsk| wØZxq
msL¨vwU KZ? [BADC-(Store Keeper)-2017]
a. 30 b. 40 c. 50 d. 60 Ans: a
Solution:
awi, Z…Zxq msL¨vwU = 2K (Gfv‡e ai‡j fMœvsk Avm‡e bv)
myZivs cÖ_g msL¨vwU = 6K
Ges wØZxq msL¨vwU = 3K
cÖkœg‡Z,
6K+3K+2K = 110
ev, 11K = 110  K = 10 myZivs wØZxq msL¨vwU = 30
11. `ywU msL¨vi e‡M©i †hvMdj 68 Ges G‡`i cv_©‡K¨i eM© 36| msL¨v؇qi ¸Ydj KZ? [BADC-(Store Keeper)-
2017]
a. 16 b. 20 c. 24 d. 28 Ans: „
Solution:
awi, GKwU msL¨v = x Ges Aci msL¨vwU = y
cÖkœg‡Z, gy‡L gy‡L:
`ywU msL¨vi e‡M©i †hvMdj 68 †`‡L ‡evSv Svq 64+4 = 68
x2+y2 = 68 Ges (x-y)2 = 36
A_©vr 82+22 = 68 nq Ges G‡`i e‡M©i cv_©K¨ (8-2)2 =
Avgiv Rvwb, 36 nq| Zvn‡j ¸Ydj n‡e 82 = 16
x2+y2 = (x-y)2+2xy ev, 68 = 36+2xy
ev, 68 - 36 = 2xy ev, 32 = 2xy xy = 16
12. ‡Kvb msL¨v H msL¨vi 25% A‡cÿv 60 †ewk| msL¨vwU KZ? [BADC-(Store Keeper)-2017]
a. 72 b. 80 c. 100 d. 120 Ans: b
Solution:
msL¨vwU 100% Ges Zv †_‡K 25% we‡qvM Ki‡j _v‡K 75% Avi GB 75% ev 4 fv‡Mi 3 fvM = 60 n‡j 1 fvM n‡e
20 Ges m¤ú~Y© msL¨vwU n‡e 4 fvM A_©vr 204 = 80|
13. 9wU msL¨vi Mo 12| Gi g‡a¨ cÖ_g 7wU msL¨vi Mo 10| evKx msL¨v `yBwUi Mo KZ? [BADC-(Store Keeper)-
2017]
a. 17 b. 18 c. 19 d. 20 Ans: c
Solution:
9wU msL¨vi mgwó = 912 = 108 gy‡L gy‡L: 7wU msL¨vq K‡g †Mj 72 = 14
7wUi mgwó = 710 = 70 GLb GB 14 Ab¨ `ywU msL¨vq fvM K‡i w`‡j n‡e 12+7 = 19

Khairul’s IBA Math Book - 50

www.bdniyog.com
evKx `ywUi mgwó = 108-70 = 38
myZivs Mo = 382 = 19

14. GKwU mgevû wÎfz‡Ri cÖwZwU evûi ˆ`N©¨ 2 wgUvi eov‡j †ÿÎdj 3 3 eM© wgUvi †e‡o hvq| mgevû wÎfzRwUi cÖwZwU
evûi ˆ`N©¨ KZ wgUvi? [BADC-(Store Keeper)-2017]
a. 1 b. 2 c. 3 d. 4 Ans: b
Solution:
3 2
mgevû wÎfz‡Ri †ÿÎdj = a,
4
3
Avevi mgevû wÎfz‡Ri bZzb evûi ˆ`N©¨ = a + 2 wgUvi myZivs mgevû wÎfz‡Ri bZzb †ÿÎdj = (a + 2)2
4
kZ©g‡Z,
3 3 2
ev, (a + 2)2 - a =3 3
4 4
3 2
ev, (a + 4a + 4 - a2) = 3 3
4
3 3
ev, (4a + 4) = 3 3 ev, 4 (a + 1) = 3 3 ev, a + 1 = 3  a = 2
4 4
15. ‰`wbK 7 N›Uv KvR K‡i 16 Rb †jvK 60 w`‡b GKwU `vjvb ˆZix Ki‡Z cv‡i| ˆ`wbK 15 N›Uv KvR K‡i 14 Rb ‡jvK
H `vjvb KZ w`‡b ˆZix Ki‡Z cvi‡e? [BADC-(Store Keeper)-2017]
a. 24 b. 30 c. 32 d. 42 Ans: c
Solution:
7 N›Uv KvR K‡i 16 Rb †jvK `vjvbwU ‰Zix K‡i = 60 w`‡b
1 N›Uv KvR K‡i 1 Rb †jvK `vjvbwU ‰Zix K‡i =60716 (Kg N›Uv Ki‡j †ewk w`b +Kg ‡jvK Ki‡j Av‡iv
†ewk )
60  7  16
15 N›Uv KvR K‡i 14 Rb †jvK `vjvbwU ‰Zix K‡i = (†ewk N›Uv I †ewk †jvK KvR Ki‡j Kg w`b
15  14
jvM‡e)
= 32 w`b|
16. GKwU QvÎvev‡m 200 Rb Qv‡Îi 120 w`‡bi Lvevi Av‡Q| 30 w`b ci 20 Rb QvÎ QvÎvevm †Q‡o P‡j †Mj| Aewkó
Lv`¨ Qv·`i KZw`b Pj‡e? [BADC-(Store Keeper)-2017]
a. 90 w`b b. 95 w`b c. 100 w`b d. 120 w`b Ans: c
Solution:
‡gvU w`b = 120-30 = 90 Ges †gvU QvÎ = 200-20 = 180
200 R‡bi Lvevi Pj‡e = 90 w`b| 1 R‡bi Pj‡e = 90200 w`b|
90  200
myZivs 180 R‡bi Pj‡e =100 w`b|
180

Khairul’s IBA Math Book - 51

www.bdniyog.com
17. K I L GKwU e¨emv ïiæ K‡i GK eQi ci DcvwR©Z A_© 12,000 UvKv †_‡K K 8,000 UvKv cvq| hw` K, 30,000
UvKv 4 gv‡mi Rb¨ wewb‡qvM K‡i _v‡K, Zvn‡j L mviv eQ‡ii Rb¨ KZ UvKv wewb‡qvM K‡iwQj? [BADC-(Store
Keeper)-2017]
a. 5000 UvKv b. 6000 UvKv c. 6500 UvKv d. 7000 UvKv Ans: a
Solution:
K 8000 UvKv †c‡j L cvq = 12000-8000 = 4000 ( A_©vr K Gi wewb‡qv‡Mi A‡a©K n‡”Q L Gi wewb‡qvM)
GLb K Gi gvwmK wewb‡qvM = 300004 = 120000 UvKv|  L Gi gvwmK wewb‡qvM = 120000 2 = 60000
UvKv| myZivs L Gi evrmwiK wewb‡qvM = 6000012 = 5000 UvKv | DËi: 5000 UvKv|
18. `yBwU bj w`‡q GKwU U¨vsK h_vµ‡g 12 I 15 N›Uvq c~Y© nq| Z„Zxq GKwU bj Øviv †mwU 20 N›Uvq Lvwj nq| wZbwU bj
GKmv‡_ Ly‡j w`‡j U¨vsKwU KZ mg‡q c~Y© n‡e? [BADC-(Store Keeper)-2017]
a. 12 N›Uvq b. 10 N›Uvq c. 8 N›Uvq d. 7 N›Uvq Ans: b
Solution:
1 1 1 543 6 1
1 N›Uvq c~Y© n‡e =      Ask|
12 15 20 60 60 10
myZivs m¤ú~Y© Ask c~Y© n‡Z mgq jvM‡e = 10 N›Uv|
19. 882 UvKv wZbRb Qv‡Îi g‡a¨ Ggbfv‡e fvM K‡i ‡`qv nj †h, 1g Rb 2q R‡bi A‡a©K UvKv cvq Ges 2q Rb 3q
R‡bi A‡a©K UvKv cvq| 3q Rb KZ UvKv cvq? [BADC-(Store Keeper)-2017]
a. 427 UvKv b. 441UvKv c. 502 UvKv d. 504 UvKv Ans: d
Solution:
awi, cÖ_g Rb cvq = K UvKv (GKcvk †_‡K A‡a©K A_© wecixZ cvk †_‡K wظY, Gfv‡e Ki‡j fMœvsk QvovB mn‡R
Kiv hv‡e)
Zvn‡j 2q Rb cvq = 2K UvKv Ges 3q Rb cvq = 4K UvKv|
cÖkœg‡Z,
K+2K+4K = 882 ev, 7K = 882K = 126 myZivs 3q Rb cv‡e = 4126 = 504 UvKv|
(ey‡S †M‡j gy‡L gy‡L : 882 Gi 7 fv‡Mi 4 fvM|)
20. 5 eQi Av‡M K Ges L Gi Mo eqm wQj 10 eQi| eZ©gv‡b K, L Ges M Gi Mo eqm 15 eQi| 2 eQi ci M Gi eqm
KZ n‡e? [BADC-(Store Keeper)-2017]
a. 15 eQi b. 17 eQi c. 20 eQi d. 22 eQi Ans: b
Solution:
5 eQi Av‡M `yR‡bi Mo 10 eQi n‡j ZLb mgwó wQj = 102 = 20 eQi Ges eZ©gv‡b mgwó 20+(52) = 30
eQi|
Avevi K, L Ges M Gi eZ©gv‡b †gvU eqm = 153 = 45 eQi|
myZivs M Gi eZ©gvb eqm = 45-30 = 15 eQi| myZivs 2 eQi ci M Gi eqm n‡e 15+2 = 17 eQi|

====================================

Khairul’s IBA Math Book - 52

www.bdniyog.com
Controller General Defense Finance (CGDF)
Post name: Auditor Exam [Exam held: 4.11.2017]

1. GKRb we‡µZv GKwU eB Gi weµq g~‡j¨i Dci 5% Qvo w`‡q µqg~‡j¨i Dci 25% jvf Ki‡jv| hw` H eB Gi µq
g~j¨ 380 UvKv n‡q _v‡K Z‡e H eB Gi weµq g~j¨ KZ †jLv wQj? [CGDF Auditor Exam-2017]
a. 400 b. 450 c. 500 d. †KvbwUB bq Ans: c
Solution:
25% jv‡f 380 UvKvi c‡Y¨i weµqg~j¨ = 380+380 Gi 25% = 380+95 = 475 UvKv|
Avevi 5% Qvo w`‡j 100 UvKvi c‡Y¨i weµqg~j¨ n‡e 95 ev 95%|
475 475  100
Zvn‡j 95% = 475, 1% = , myZivs 100% = = 500 UvKv|
95 95
2. wZbwU msL¨vi Mo x| hw` cÖ_g 2 wU msL¨vi Mo y nq Ges †kl 2 wU msL¨vi Mo z nq , Z‡e wØZxq msL¨vwU KZ?
[CGDF Auditor Exam-2017]
a. 2y – 2z + 3x b. 3x – y - z c. 2y + 2z - 3x d. †KvbwUB bq Ans: c
Solution:
wZbwU msL¨vi Mo x n‡j msL¨v wZbwUi †hvMdj = 3x|
Avevi cÖ_g `ywU msL¨vi Mo y n‡j Zv‡`i mgwó 2y Ges †k‡li `ywU msL¨vi Mo z n‡j Zv‡`i mgwó n‡e 2z
myZivs wØZxq msL¨vwU = 2y + 2z - 3x DËi:
[ Note: GLv‡b `ywU + `ywU = 4wU msL¨vi †hvMdj 2y + 2z ‡_‡K wZbwU msL¨vi †hvMdj 3x we‡qvM Ki‡j ‡h msL¨vwU
`yevi e¨eüZ n‡q‡Q A_©vr 2q msL¨vwU †ei n‡e|]
3. Avwid GKwU KvR 60 w`‡b Ki‡Z cv‡i| Avwid Ges evei GK mv‡_ wg‡j H KvR 20 w`‡b †kl Ki‡Z cv‡i| evei
GKv KZw`‡b H KvR †kl Ki‡Z cvi‡e? [CGDF Auditor Exam-2017]
a. 30 b. 45 c. 60 d. †KvbwUB bq Ans: a
Solution:
1
`yR‡b wg‡j 1 w`‡b Ki‡Z cv‡i = Ask|
20 60  20 60  20
Shortcut:   30 w`b|
1 60  20 40
Avevi Avwid GKv GKw`‡b Ki‡Z cv‡i = Ask|
60
1 1 2 1
Zvn‡j evei GKvKx GKw`‡b Ki‡Z cv‡i = - =  Ask|
20 60 60 30
1
Ask KvR Ki‡Z 1 w`b jvM‡j m¤ú~Y© Ask KvR Ki‡Z mgq jvM‡e = 30w`b| DËi:30w`b|
30
4. 70 wjUvi †c‡Uªvj AK‡Ub wgkª‡Y †c‡Uªvj I AK‡U†bi AbycvZ 5 t 2| GB wgkª‡Y Avi KZ wjUvi AK‡Ub wgkv‡j
†c‡Uªvj I AK‡U†bi AbycvZ 2 t 1 n‡e? [CGDF Auditor Exam-2017]
a. 5 b. 7 c. 10 d. †KvbwUB bq Ans: a
Solution:
‡c‡Uªvj I AK‡U‡bi cwigvb = 70 Gi 7 fv‡Mi 5 fvM = 50 Ges 2 fvM = 20|
Khairul’s IBA Math Book - 53

www.bdniyog.com
awi, AK‡Ub †gkv‡Z n‡e = x wjUvi|
cÖkœg‡Z,
50:20+x = 2:1
ev, 40+2x = 50 (cÖ_g4_© ivwk = 2q  3q ivwk)
ev, 2x = 10  ev, x = 5 wjUvi| DËi:
5. Kgjvcyi †ókb †_‡K GKwU †Uªb mKvj 8 Uvi mgq NÈvq 30 wKwg MwZ‡Z PUªMÖv‡gi D‡Ï‡k¨ iIqvbv w`j| mKvj 9 Uvq
Av‡iKwU †Uªb N›Uvq 40 wKwg MwZ‡Z PUªMÖv‡gi D‡Ï‡k¨ iIqvbv w`j| †Uªb `ywU Kgjvcyi †_‡K KZ `~‡i wgwjZ n‡e?
[CGDF Auditor Exam-2017]
a. 120 b. 240 c. 360 d. †KvbwUB bq Ans: a
Solution:
30 wKwg †e‡M Pjv cÖ_g †UªbwU 1 N›Uv Av‡M Pjvq `~iZ¡ ˆZix nq =30wKwg|
2q †Uªb 1 N›Uvq cÖ_g †Uª‡bi mv‡_ `~iZ¡ Kgvq = 40-30 = 10 wKwg|
10wKwg `~iZ¡ Kgv‡j Kgjvcyi †_‡K †h‡Z n‡e = 40 wKwg|
40  30
myZivs 30 wKwg `~iZ¡ Kgv‡Z n‡j Kgjvcyi †_‡K †h‡Z n‡e = 120wKwg| DËi:
10
[ Note: hLb †Uªb؇qi gv‡Si `~iZ¡ 30 wKwg _vK‡e bv, ZLwb Zviv wgwjZ n‡e| ]
6. Avwmd Awd‡m hvevi mgq A‡a©K `~iZ¡ NÈvq 5 wKwg †e‡M AwZµg Ki‡jv Ges evwK A‡a©K `~iZ¡ NÈvq 3 wKwg †e‡M
AwZµg Ki‡jv| Zvi Mo †eM NÈvq KZ wKwg? [CGDF Auditor Exam-2017]
a. 4 b. 15/8 c. 15/4 d. †KvbwUB bq Ans: c
Solution:
‡ gvU AwZµvšÍ c_
Mo MwZ‡eM †ei Kivi wbqg n‡”Q GLv‡b `yfv‡eB Kiv hvq = awi, cÖ_g A‡a©K 15 Ges
† gvU AwZevwnZ mgq |
c‡ii A‡a©K 15 wKwg| (5 Ges 3 Gi j.mv.¸)
†gvU AwZµvšÍ c_ = 15+15 = 30 wKwg Ges †gvU mgq 155 = 3N›Uv Ges 153 = 5 N›Uv| 3+5 = 8N›Uv
15  15 30 15
myZivs Mo MwZ‡eM = 
35 8 4
2 xy 2  5  3 30 15
Av‡iv mn‡R Ki‡Z n‡j Mo MwZ‡eM = =  
xy 53 8 4

7. GKwU c‡Y¨i weµq g~j¨ wظY n‡j we‡µZvi gybvdv †e‡o 3 ¸Y n‡e| g~j¨e„w× bv K‡i cY¨wU weµq Ki‡j we‡µZvi
kZKiv KZ gybvdv n‡e (%)?[CGDF Auditor Exam-2017]
a. 50 b. 100 c. 150 d. †KvbwUB bq Ans: b
Solution:
awi, cÖ_g weµqg~j¨ = 100 UvKv | Zvn‡j 2q weµqg~j¨ = 1002 = 200UvKv|
†hLv‡b jvf evo‡jv 200-100= 100UvKv| GB 100 UvKv AwZwi³ jv‡fi Kvi‡Y jvf †e‡o †Mj 2¸Y |
GLv†b 2¸Y jvf = 100 n‡j 1 ¸Y jvf = 1002 = 50 UvKv| Zvn‡j 100UvKvq wewµ Kivq jvf 50| myZivs µqg~j¨
= 100-50 = 50 UvKv|
GLb 50UvKvq jvf 50UvKv n‡j jv‡fi nvi 100%| DËi:
8. GKRb we‡µZv 17 wU Kjg 720 UvKvq wewµ K‡i †h †jvKmvb Ki‡jv Zv 5 wU Kj‡gi µq g~‡j¨i mgvb| GKwU Kj‡gi
µq g~j¨ KZ? [CGDF Auditor Exam-2017]
a. 52 b. 54 c. 60 d. †KvbwUB bq Ans: c
Khairul’s IBA Math Book - 54

www.bdniyog.com
Solution:
17wUi µqg~j¨ - 17wU weµqg~j¨ = 5wUi µqg~j¨ (µqg~j¨ - weµqg~j¨ = ÿwZ)
ev,17wUi µqg~j¨ -5wUi µqg~j¨ = 17wUi weµqg~j¨
ev, 12wUi µqg~j¨ = 720 UvKv| ( †h‡nZz cÖ‡kœ †`qv Av‡Q †h 17wUi weµqg~j¨ = 720 UvKv|)
myZivs 1wUi µqg~j¨ = 72012 = 60UvKv| DËi:
9. `yB nvRvi QqkZ UvKv wZb R‡bi g‡a¨ Ggb fv‡e fvM Kiv n‡jv †h cÖ_g e¨w³ wØZxq e¨w³i †P‡q wظb UvKv †cj Ges
wØZxq e¨w³ Z…Zxq e¨w³i 1/10 ¸Y UvKv †cj| Z…Zxq e¨w³ KZ UvKv †cj? [CGDF Auditor Exam-2017]
a. 1800 b. 1625 c. 2000 d. †KvbwUB bq Ans: c
Solution:
awi, wØZxq e¨vw³ cvq x UvKv myZivs cÖ_g e¨w³ cvq 2x UvKv Ges Z…Zxq e¨w³ cvq 10x UvKv|
cÖkœg‡Z,
x+ 2x+10x = 2600
13x = 2600
 x = 200
myZivs Z…Zxq e¨w³ cvq = 10200 = 2000 UvKv| DËi:
10. PweŸk UvKv `‡i µq Kiv 25 †KwR Pv‡ji mv‡_ 42 UvKv `‡i µq Kiv KZ †KwR Pvj wgwk‡q 40 UvKv `‡i wewµ Ki‡j
25% gybvdv n‡e? [CGDF Auditor Exam-2017]
a. 20 b. 12.5 c. 16 d. †KvbwUB bq Ans: a
Solution:
awi, 42 UvKv `‡i µq Kiv Pv‡ji cwigvY = x ‡KwR|
Zvn‡j †gvU µqg~j¨ = 2425 + 42x
Ges †gvU weµq g~j¨ = 40(25+ x)
cÖkœg‡Z,
2425 + 42x Gi 125% = 40(25+ x) [µqg~j¨ + 25% jvf = weµqg~j¨]
5
ev, (600+42x)  = 1000+40x
4
ev, 3000+210x = 4000+160x ev, 50x = 1000  x = 20 DËi:

24 42
Average
32

10 8
10:8 or 5:4 GLb 5=25 myZivs

11. GKwU †cwÝj I GKwU Kj‡gi µqg~‡j¨i AbyycvZ 3:7| Zv‡`i weµqg~‡j¨i AbycvZ 1:4| hw` cY¨ `ywU weµq K‡i
†jvKmv‡bi cwigvY mgvb nq, Z‡e Kj‡gi µqg~j¨ I weµqg~‡j¨I AbycvZ KZ ? [CGDF Auditor Exam-2017]
a. 3:2 b. 14:9 c. 21:16 d. †KvbwUB bq Ans: c
Solution:
awi, †cwÝj I Kj‡gi µqg~j¨ h_vµ‡g: 3x:7x Ges weµqg~j¨ = y:4y
Khairul’s IBA Math Book - 55

www.bdniyog.com
Kj‡gi µqg~j¨ I weµqg~‡j¨i AbycvZ =7x:4y
cÖkœg‡Z, 3x-y=7x-4y [ ‡h‡nZz `ywU‡ZB mgvb †jvKmvb n‡q‡Q]
4x
ev, 3y = 4x y =
3
4x 16 x
myZivs KjgwUi µqg~j¨ Ges weµqg~‡j¨i AbycvZ n‡e =7x:4 =7x: = 21x:16x = 21:16 DËi:
3 3
12. †Kv‡bv ¯‹z‡ji QvÎ-QvÎxi msL¨v wQj 1800 Rb| wKQyw`b c‡i 4% QvÎ P‡j †Mj , Avi 5% bZzb QvÎx fwZ© n‡jv| Gi
d‡j H ¯‹y‡ji ‡gvU QvÎ-QvÎxi msL¨v 2% †e‡o †Mj| eZ©gv‡b H ¯‹z‡j ‡gvU QvÎ-QvÎxi msL¨v KZ ? [CGDF Auditor
Exam-2017]
a. 1850 b. 1872 c. 1836 d. †KvbwUB bq Ans: c
Solution:
1800 QvÎ - QvÎxi g‡a¨ 2% evov A_© n‡jv 1800 Gi 2% = 36 e„w× cvIqv| Zvn‡j e„w× cvIqvi ci n‡e
1800+36 = 1836 Rb|
[Note: gv‡Si 4% I 5% As‡Ki Dci †Kvb cÖfve †dj‡Z cvi‡Q bv e‡j G¸‡jv wb‡q fve‡j ïay mgq bó n‡e|]
13. hw` `ywU msL¨vi AbycvZ 4:3 nq, Z‡e wb‡Pi ‡Kvb msL¨vwU H `ywU msL¨vi ‡hvMdj n‡Z cvi‡e bv | [CGDF
Auditor Exam-2017]
a. 112 b. 154 c. 178 d. †KvbwUB bq Ans: c
Solution:
`ywU msL¨vi AbycvZ 4:3 n‡j H msL¨v`ywUi †hvMdj Aek¨B Abycv‡Zi †hvMdj w`‡q wb:‡k‡l wefvR¨ n‡Z n‡e|
cÖ‡kœ cÖ`Ë Ackb ¸‡jvi g‡a¨ ïay 178 †K 7 w`‡q fvM Kiv hv‡”Q bv e‡j GUvB DËi:
14. GKwU wbe©vP‡b `yBRb cÖv_x© Rvgvb Ges †bvgvb cÖwZØw›`Zv Kij| Rvgvb wbe©vP‡b cÖ`Ë †fv‡Ui 40% ‡fvU †cj| †bvgvb
Rvgv‡bi †P‡q 298 ‡fvU †ekx †c‡q wbe©vP‡b Rqjvf Ki‡jv| H wbe©vP‡b KZRb †fvU w`‡qwQj? [CGDF Auditor
Exam-2017]
a. 1490 b. 1520 c. 1540 d. †KvbwUB bq Ans: a
Solution:
wbe©vP‡b †gvU ‡fvU = 100%|
Rvgvb 40% †c‡j †bvgvb cvq 60% Ges Zv‡`i †fv‡Ui cv_©K¨ = 60-40 = 20%|
GLb 20% = 298 n‡j (Df‡qB †fv‡Ui e¨veavb)
100% Gi gvb n‡e 5 ¸Y †ewk| A_©r 2985 = 1490 (300 Gi 5 ¸Y 1500 †_‡K 10 Kg n‡e|) DËi:
15. GKRb we‡µZv GKwU cY¨ 30% g~j¨ Qvo w`‡q weµq Kivq Zvi 16% †jvKmvb n‡jv| cY¨wU 10% g~j¨ Qv‡o wewµ
Ki‡j Zvi KZ ÿwZ ev jvf n‡Zv? [CGDF Auditor Exam-2017]
a. 5% jvf b. 8% jvf c. 5% ÿwZ d. †KvbwUB bq Ans: b
Solution:
16% ÿwZ‡Z 100 UvKvi c‡Y¨i weµqg~j¨ = 100-16 = 84|
84
Avevi 30% Qvo †`qvi ci weµqg~j¨ = 70% = 84 ev, 1% = |
70
84  100
myZivs wjwLZ gyj¨ = 100% =  120
70
120 UvKvi Dci 10% Qvo w`‡j bZzb weµqg~j¨ n‡e 120-120 Gi 10% = 120-12 = 108 UvKv|
Zvn‡j jvf n‡e 108-100 = 8%| DËi:
Khairul’s IBA Math Book - 56

www.bdniyog.com
16. 50 Rb †jv‡Ki g‡a¨ 35 Rb Bs‡iwR, 25 Rb Bs‡iwR I evsjv DfqB Ges cÖ‡Z¨‡KB `yBwU fvlvi AšÍtZ GKwU fvlvi
K_v ej‡Z cv‡ib| evsjvq KZRb K_v ej‡Z cv‡ib? [CGDF Auditor Exam-2017] [ 35-Zg wewmGm]
a. 10 b. 15 c. 40 d. 30 Ans: c

Solution:
cv‡ki wPÎwU eyS‡j ‡fbwP‡Îi AsK Lye mnR g‡b n‡e| English Bangla
cÖ‡kœi 50 Rb nj ‡gvU, hv‡`i‡K `ywU fv‡M fvM Ki‡Z n‡e|
G‡ÿ‡Î evsjv I Bs‡iRxi Rb¨ `ywU
e„Ë AsKb Ki‡Z n‡e| Ges cÖ‡kœ †h‡nZz ejv n‡q‡Q 25 Rb Dfq n(E)=3 n(EB) n(B)=
5 =25 ?
fvlvq K_v ej‡Z cv‡i ZvB e„Ë `ywU‡K Ggbfv‡e wgwj‡q w`‡Z n‡e †hb
Zv‡`i gv‡SGgb GKwU †QvU e„Ë ˆZix nq ‡hUv Øviv Dfq 25 Rb †K †evSvq|

Zvn‡j Bs‡iRxi e„‡Ëi 35 †_‡K Df‡q †Mj 25 Zvn‡j _vK‡jv 35-25 = 10 Avevi evsjvi KZRb Zv RvbvB †bB|
wKš‘ Dfq 25 Rb evsjvi I †jvK| Zvn‡j 50 Rb †gjv‡Z Av‡iv †jv‡Ki cÖ‡qvRb 50-35 = 15 Rb| 15 Rb evsjvq
K_v ej‡Z cv‡i| Avevi Df‡qi ga¨Kvi 25 Rb evsjvq K_v ej‡Z cv‡i| Zvn‡j evsjvfvlx †jv‡Ki msL¨v 25+15 =
40 Rb| DËi:

(AšÍZ GKwU fvlvq K_v ejvi A_© †Kvb fvlvqB K_v ej‡Z cv‡i bv Ggb †jvK †bB)

myÎ cÖ‡qv‡M mgvavb:


awi,
†gvU †jvK msL¨v = n(EB) = 50 Rb ïay Bs‡iRx‡Z K_v ej‡Z cv‡i, n(E) = 35 Rb, ïay evsjvq K_v ej‡Z cv‡i
n(B) =?, evsjv Ges Bs‡iRx Dfq fvlvq ej‡Z cv‡i n(EB) = 25 Rb GLb myÎ cÖ‡qvM K‡i cvB,
n(EB) = n(E) + n(B) - n(EB) (GB myÎwU-B me mgq, ïay hvi hvi gvb ewm‡q mgvavb Ki‡Z n‡e)
ev, 50 = 35 + n(B) - 25
ev, 50 = 10 + n(B) ev, 50 -10 = n(B)  n(B) = 40 myZivs evsjvq K_v ej‡Z cv‡i 40 Rb| DËi:
17. GKwU K¬v‡mi 40% QvÎ evsjvq Ges 25% QvÎ As‡K Ges 10% QvÎ Dfq wel‡q AK…ZKvh© n‡q‡Q| H K¬v‡mi kZKiv
KZRb QvÎ Dfq wel‡q K…ZKvh© n‡q‡Q? [CGDF Auditor Exam-2017]
a. 25% b. 35% c. 45% d. †KvbwUB bq Ans: c
Solution: evsjv Total =100% AsK
Dfq wel‡q K…ZKvh© = [100- {(40+25)- 10}]% = 45%

40-10=30% 10% 25-10 =15%


55%

45%
18. myg‡bi †eZb iwn‡gi ‡eZ‡bi 210% | wjU‡bi †eZb wjRvi †eZ‡bi 70%| wjRvi †eZb iwn‡gi †eZ‡bi wظY |
mygb Ges wjU‡bi †eZ‡bi AbycvZ KZ? [CGDF Auditor Exam-2017]
a. 3:2 b. 2:1 c. 1:1.5 d. †KvbwUB bq Ans: a
Solution:
awi, iwn‡gi †eZb = 100 myZivs myg‡bi ‡eZb = 210

Khairul’s IBA Math Book - 57

www.bdniyog.com
wjRvi †eZb = 1002 = 200
wjU‡bi †eZb = 200 Gi 70% = 140
myZivs mgb Ges wjU‡bi †eZ‡bi AbycvZ = 210:140 = 3:2 DËi:
19. wcZvi eZ©gvb eqm cy‡Îi eq‡mi wظY A‡cÿv 2 eQi †ekx| 5 eQi ci wcZv Ges cy‡Îi eq‡mi mgwó 102 eQi n‡j
cy‡Îi eZ©gvb eqm KZ eQi? [CGDF Auditor Exam-2017]
a. 30 b. 35 c. 40 d. †KvbwUB bq Ans: a
Solution:
5 eQi ci wcZv I cy‡Îi eq‡mi mgwó 102 eQi n‡j eZ©gv‡b mgwó = 102-(52) = 92 eQi|
awi, cy‡Îi eZ©gvb eqm = x myZivs wcZvi eZ©gvb eqm = 2x+2
cÖkœg‡Z, Shortcut: wظ‡Yi †_‡K 2 †ewk bv n‡j 90
x+2x+2 = 92 ev, 3x = 90 x=30 DËi: n‡Zv| ZLb 3¸Y = 90 n‡j cyÎ 1 ¸Y = 30

20. Kwig GKwU KvR iwn‡gi †P‡q 60w`b Kg mg‡q Ki‡Z cv‡i| Kwi‡gi Kv‡Ri MwZ hw` iwn‡gi Kv‡Ri MwZi 3 ¸Y nq
Z‡e Kwig GKv H KvR KZw`‡b †kl Ki‡Z cvi‡e? [CGDF Auditor Exam-2017]
a. 15 b. 21 c. 30 d. †KvbwUB bq Ans: c
Solution:
awi, Kwig‡K jv‡M = x w`b| Zvn‡j iwng‡K jvM‡e 3x w`b| (hvi MwZ Kg Zv‡K †ewk w`b mgq jvM‡e)
GLv‡b w`b bv a‡i MwZ ai‡j c‡i As‡Ki wn‡me †gjv‡bv KwVb n‡e ZvB w`b‡K aiv DËg)
cÖkœg‡Z,
3x-x = 60 ( ‡h‡nZz `yR‡bi jvMv mg‡qi cv_©K¨ = 60 w`b|)
2x = 60 x = 30 w`b| Zvn‡j Kwig‡K mgq jvM‡e: 30 w`b| DËi:

====================================

Khairul’s IBA Math Book - 58

www.bdniyog.com
BREB
Exam taker: IBA, DU
Exam Taker : IBA DU Exam date: 18-01-2019

36. Kalam has three daughters: Shaila, Meena and Tina. Three years ago when Kalam was
twice as old as Tina, he was 30 years older than Meena. Now he is 47 years older than
Shaila. In 4 years, Shaila will be half as old as Tina. What is the sum of the current ages
of Kalam and his three daughters?
(a) 138 (b) 144 (c) 154 (d) 180 (e) None Ans:a
Solution:
Let, 3 years ago, Tina was, x years, So, Kalam was 2x, and Meena was 2x-30 years

Now, Tina is x+3, Kalam is 2x+3, Meena 2x-30+3 = 2x-27 and Shaila is 2x+3-47 = 2x-44

After 4 years Tina will be x+3+4 = x+7 & Shaila will be = 2x-44+4 = 2x-40
ATQ,
2(2x-40) = x+7
4x-80 = x+7
3x = 87
x = 29
So, total present age of all of them is x+3 + 2x+3 + 2x-27 + 2x-44 = 7x-65 = 729-65=
203-65 = 138

wb‡Pi e·wU †_‡K mevi eqm¸‡jv wgwj‡q wb‡Z cv‡ib


Tina Kalam Meena Shaila
3 years ago x 2x 2x-30
Now x+3 2x+3 2x-27 2x-44
After 4 years x+7 2x+7 2x+23 2x+40

37. In a garden there are 10 rows and 12 colums of mango trees. The distance between two
trees is 2 meters and a distance of one meter is left from all sides of the boundary of the
garden. What is the length of the garden? (GKwU evMv‡b 10 mvwi MvQ Av‡Q Ges cÖwZ mvwi‡Z 12wU K‡i
MvQ Av‡Q| GKwU MvQ †_‡K Aci Mv‡Qi `~iZ¡ 2 wgUvi, Pvicv‡ki mxgvbvq 1 wgUvi K‡i RvqMv dvuKv ivLv n‡q‡Q|
evMv‡bi ‰`N©¨ KZ?) (National Bank. PO.-2014 and Agrani Bank Seni Offi-2013)
(a) 20m (b) 22 m (c) 24 (d) 26 (e) None Ans: c
Solution:
12wU MvQ jvMv‡bvi Rb¨ gvSLv‡b duvKv Ask Av‡Q 11wU hvi cÖwZwU 2 wgUvi K‡i Ges Mv‡Qi c‡iI `ycv‡k 1wgUvi K‡i
†gvU 1+1 = 2wgUvi RvqMv Av‡Q | ZvB †jLv hvq,
length of garden is 112+ (1+1) = 22+2 = 24

Khairul’s IBA Math Book - 59

www.bdniyog.com
38. Due to reduction in the bus price by 15%, the number of passengers on a certain route
increased by 40% . What will be the percentage increase in revenue?
(a) 17 (b) 18 (c) 19
(d) 20 (e) None Ans: c
Solution:
Let, at first price was = 100 and passengers were = 100
So, Total revenue = 100100 = 10000
New price = 85 and new passengers = 140
So, new total revenue = 85140 = 11900
Revenue increase = 11900-10000 = 1900
1900 100
Increase % = = 19%
10000

xy (-15)  ( 40)


Shortcut: xy+ So, (-15)+40 + = 25+(-6) = 19%
100 100

gy‡L gy‡L: 15% K‡g nq 100-15 = 85 Gici 40% e„w× †c‡q 85 Gi 40% = 10% = 8.5 n‡j 40% n‡e 34|
Kgvi mgq 15 K‡g evovi mgq 34 evo‡j †gv‡Ui Dci evo‡e 34-15 = 19%

1
39. Hasan completed portion of a job in 8 days and left Then Jashim was recruited and
3
he worked for 5 days and left the job. Hasan again joined and completed the rest of the
work in 12 days. How many days would Jashim recuire to complete the job alone ?
(a) 10 days (b) 24 days (c) 30 days
(d) 36 days (e) None Ans:c

Solution:
Hasan worked for 8+12 = 20days
1
In, 8 days Hasan completed = part.
3
1
In, 1 day Hasan completed = part.
38
20 5
In, 20 day Hasan completed = = part. (GKB †jvK `yÕevi 8 I 12 w`b KvR K‡i‡Q ZvB
38 6
20w`b)
5 1
Remaining part completed by Jahsim – 1- = part, (nvmv‡bi Kiv Ask ev` w`‡q evKxUv Rwmg
6 6
K‡i‡Q)
1
part is completed by Jashim in = 5days
6
1 ‘’ is completed by jashim in = 56 = 30days.

Khairul’s IBA Math Book - 60

www.bdniyog.com
40. Jerin has 40% more books than Rashed. However, if she gives 45 her books to Rashed,
then Rashed will have 10% more books than Jerin. How many books did Jerin begin
with? (‡Rwi‡bi wbKU iv‡k‡`i †_‡K †_‡K 40% †ewk ÷¨v¤ú Av‡Q| hw` ‡m Zv †_‡K 45wU ÷¨v¤ú iwng‡K †`q
Zvn‡j iwn‡gi Kv‡Q Kwi‡gi †_‡K 10% ÷¨v¤ú †ewk _v‡K| Kwi‡gi Kv‡Q ïiæ‡Z KZwU ÷¨v¤ú wQj?) [Sonali Bank-
(Officer-Cash)-2018]
(a) 140 (b) 175 (c) 245
(d) 385 (e) None Ans: c
Solution:
Let, Rashed has = x stamps
So Jerin has = 140% of x = 1.4x
ATQ,
110% of (1.4x-45) = x+45(‡Rwi‡bi †_‡K 45 wU we‡qvM K‡i iv‡k`‡K †`qvq Zv 10% †ewk A_©vr 110% n‡q‡Q)
11
or,  (1.4x  45)  x  45 (‡Rwi‡bi bZzb ÷¨v‡¤úi 110% = iv‡k‡`i bZzb eBqi msL¨v)
10
or, 15.4x- 495 = 10x+450
or, 5.4x = 945
945
x = = 175
5.4
Rashed has = 175 books
So, Jerin has 175+ 40% of 175 = 175+70 = 245

ïw× cixÿv: Ackb a‡i Kivi Rb¨: †Rwi‡bi Kv‡Q 245 _vK‡j iv‡k‡`i Kv‡Q 175wU _vK‡e (KviY 40% †Rwi‡bi
†ewk)| Gici ‡Rwib 45wU w`‡j †Rwi‡bi _v‡K 245-45 = 200wU Ges iv‡k‡`i n‡q hvq 175+45 = 220wU| hv 200
Gi †_‡K 10% †ewk| ‡h‡nZz cÖ`Ë me¸‡jv kZ© wg‡j †M‡Q ZvB 245 B DËi|

41. Alilm was hired for a job for 7 days. Each day, he was paid Tk. 10 more than what he
was paid for previous day of work. The total amount he was paid in the first 4 days of
work equated the tolal amount he was paid in the last 3 days, what his starting pay?
(a) Tk.90 (b) Tk.138 (c) Tk.153
(d) Tk.160 (e) None Ans: a
Solution:
Let first day payment = x tk
ATQ,
x + (x+10) + (x+20) + (x+30) = (x+40) + (x+50) + (x+60) [cÖ_g 4w`‡bi gRyix = c‡ii 3w`‡bi gRyix]
or, 4x +60 = 3x+ 150
So, x = 90

Khairul’s IBA Math Book - 61

www.bdniyog.com
42. The captain of Bangladesh cricket team is 26 years old and the wicket keeper is 4 years
older. If the ages of these two are excluded, the average age of the remaining players is
one year less than the average age of the whole team. what is the average age of the team
?
(a) 21 (b) 22 (c) 23
(d) 24 (e) None Ans: e

Solution:
Captain age = 26
So, Wicket keeper age = 26+4 = 30
Let,
average age of 11 players = x
So, total age of 11 players = 11x
Without captain and wicketkeeper average age = (x-1)
Without captain and wicketkeeper total age = 9(x-1)
ATQ,
11x – 9(x-1) = 30+26 [11 R‡bi eqm - 9 R‡bi eqm = Aci 2 R‡bi eqm]
11x-9x+9 = 56
2x = 47
x = 23.5 Which is not given in the option.

1 3
43. A cement mixture is composed of 3 elements. By weight of the mixture is sand, is
3 5
water and the remaining 12 pounds of the mixture is gravel. What is the weight of the
entire mixture in pounds ? [BREB-2019]
(a) 60 (b) 80 (c) 90
(d) 180 (e) None Ans: d
Solution:
Let, the total mixture = x
ATQ,
x 3x
x- ( + ) = 12 [me©‡gvU †_‡K 2 Ask ev` w`‡j Ab¨ Dcv`v‡bi 12 cvDÛ]
3 5
5x  9x
or, x- = 12
15
15x - 14x
or, = 12
15
x = 180

44. If x8 and y3, then which of the following must be true?[BREB-2019]
(a) x+y5 (b) x+y11 (c) x-y5 (d) x-y5 (e) None Ans:d
[we¯ÍvwiZ e¨vL¨v c‡o eySzb Zvici ey‡S †M‡j Lye Kg mg‡q mgvavb Kiv hv‡e|]
Solution:
e¨vwmK Kb‡mÞ wK¬qvi bv _vKvi Kvi‡Y GB ai‡Yi cÖkœ¸‡jv A‡b‡Ki Kv‡Q KwVb g‡b n‡jI UwcK IqvBR e¨vwmK
Kb‡mÞ wK¬qvi Kivi ci cÖvKwUm Ki‡j Lye mn‡R jwRKvwj mgvavb Kiv hv‡e|
Khairul’s IBA Math Book - 62

www.bdniyog.com
cÖkœ¸‡jv‡Z cÖ`Ë K¬z Ges Ackb ¸‡jv †U÷ Kivi gva¨‡g DËi †ei Ki‡Z nq|

x8 A_© x Gi gvb 8 †_‡K ïiæ K‡i 8 Gi Dc‡i †h †Kvb msL¨v †hgb: 8,9,10 - - - 50---500
Avevi, y3 A_© y Gi gvb 3 †_‡K ïiæ K‡i 3 Gi wb‡P hZ msL¨v Av‡Q †hgb: 3, 2, 1, 0, -1 , -5, -10, -500
GLb †Lqvj Kiæb cÖ‡kœ ejv n‡q‡Q must be true A_©vr Aek¨B mZ¨ n‡e | A_©vr Ab¨ wKQz nIqvi my‡hvM _vK‡j Zv
†bqv hv‡e bv| GLb Ackb¸‡jv †U÷ Ki‡jB DËi †ei n‡q hv‡e|
(a) x+y5 [GLv‡b x Gi gvb 8 Ges y Gi gvb -3 wb‡j x+y = 8+(-3) = 5 Avevi GLv‡bB x Gi gvbUv GKUz eo
wb‡jB x+y Gi gvb 5Gi †_‡K eo I n‡e| Zvn‡j ejv hvq x+y5 wKš‘ x Gi gvb‡K eo bv K‡i y Gi gvb‡K
cwieZ©b Ki‡j ‡hgb: y=-4 wb‡j x+y = 8-+(-4) = 4 hv 5 Gi †_‡K †QvU| myZivs GUv mwVK bv I n‡Z cv‡i|]
(b) x+y11 [Dc‡ii gZB (b) x+y Gi gvb 11 Gi mgvb ev 11 Gi †_‡K Kg n‡Z cv‡i hw` †QvU msL¨v †_‡K eo
msL¨v we‡qvM Kiv nq| wKš‘ cÖ_‡gi msL¨Uv A‡bK eo †bqvi ci c‡ii msL¨vUv †QvU wb‡j we‡qvM Kivi ciI
we‡qvMdjUv‡K A‡bK eo evbv‡bv m¤¢e| †hgb:x = 500 Ges y =1 n‡j x+y = 500+1 = 501 hv 11 Gi †_‡K
A‡bK eo) ZvB GUvI mwVK bq| 500 Gi RvqMvq 5000 I †bqv hvq GB welqUv gv_vq ivL‡Z n‡e| A‡b‡K †QvU †QvU
msL¨vi g‡a¨B _vKvi Kvi‡Y KbwdDkb `~i n‡Z †ewk mgq jv‡M| ]
Dc‡ii Ackb `ywU †_‡K GUv †evSv ‡Mj †h †hvM K‡i †hvMdj wUi abvZ¡K ev FYvZ¥K gvb Avbwjwg‡UW n‡Z cv‡i|
GKUv wUª· †`B| GiKg cÖkœ¸‡jv gyjZ mxgv‡iLvi †kl gv‡bi Dci DËi †ei nq| wb‡P †`Lyb|
(c) x-y5 [8-3 = 5 nq wKš‘ 9-3 = 6 nq| A_©vr we‡qvMdj 5 †_‡K ïiæ K‡i eo n‡”Q | (Kg‡ZI cv‡i ) †h‡nZz
eo n‡Z cv‡i ZvB GUv mwVK bq]
(d) x-y5 [8-3 = 5 GLv‡b 8 Gi †_‡K †QvU Kivi Avi Dcvq †bB | †Zgwbfv‡e 3 Gi †_‡K eo Kivi Dcvq I
†bB| KviY G¸‡jv me©‡kl gvb| GLb 9-3 = 6 A_©vr 5 Gi †_‡K eo n‡”Q| Avevi y Gi gvb 3 bv wb‡q †QvU wb‡j
†hgb: 8-2 = 6, Avevi 8- (-5) = 13 A_©vr eo FYvZ¥K msL¨v we‡qvM Ki‡j we‡qvMdj †QvU bv n‡q Av‡iv eo n‡q
hv‡”Q | Zv‡nj †evSv hv‡”Q GLv‡b †Kvbfv‡eB we‡qvMdj‡K 5 Gi †_‡K †QvU Kiv hv‡”Q bv|]

Shortcut: x Gi me©wb¤œ gvb 8 ‡_‡K y Gi m‡ev©”P gvb 3 we‡qvM Kivi ciI we‡qvMdj 5 nq| Gi evB‡i Ab¨ †h †Kvb
gvb wb‡q wn‡me Ki‡j Zv 5 Gi †_‡K eo n‡e wKš‘ †Kvbfv‡eB Kg n‡e bv| ZvB DËi: d. x-y5

45. If w is 10% less than x, and y is 30% less then z, than wy is what percent less
then xz ? (hw` w, x Gi †_‡K 10% †QvU Ges y, z Gi †_‡K 30% ‡QvU nq Zvn‡j wy, xz Gi †_‡K
kZKiv KZ Kg?) [Sonali Bank –(SO)-2018]
(a) 10% (b) 20% (c) 37%
(d) 40% (e) None Ans: c
Solution:
if x = 100 then w = 90
 gy‡L gy‡L : 10% †QvU n‡j 100 †_‡K 90 nq | Gici 90 Gi
if z = 100 then y = 70
30% †QvU n‡j 27 ‡QvU nq Ges `ywU †QvU GKmv‡_ n‡q eo Gi
so, xz = 100100 = 10000
mv‡_ Zzjbv Ki‡j †gv‡Ui Dci †QvU nq 10%+27% = 37%
and wy = 9070 = 6300
xz – wy = 10000-6300 = 3700
3700 100
less % = = 37%
10000

Khairul’s IBA Math Book - 63

www.bdniyog.com
46. The operation @ is defined for all integers x and y as x@y = xy-y. If x and y are positive
integers, which of the following cannot be zero?
Ans: D. (x+1)@y
(a) y@x (b) x@y (c)(x-1)@y
(d) (x+1)@y (e ) None Ans:d
Solution:
x@y = xy-y
or,x @y = y (x-1)
or, (x+1)@ y = y (x+1-1) = (y)(x)
‡h‡nZz x I y `ywU abvZ¥K c~Y© msL¨v myZivs G‡`i ¸Ydj ‡Kvbfv‡eB FYvZ¥K n‡e bv|

47. Anik travels 35 kilometers(km) party at 4km/h and party at 5km/h. if he covers the
distance at 5 km/h he could cover 2 km more in the same time. The time taken to cover
the whole distance at original rate is---
(a) 4 hr 30 min (b) 5 hr 24 min (c) 7 hr 24 min
(d) 9 hr 24 min (c) None Ans: c
Solution:
Let, at 4km/hr he goes = x km
So, at 5km/hr he goes = 35-x
x 35  x
total time required = +
4 5
35  2 37
if he travel at 5km/hr whole distance he needs =  hrs
5 5
ATQ,
x 35  x 37
+ = [wfbœ wfbœ MwZ‡Z jvMv mgq m¤ú~Y© c_ 5 wKwg †e‡M hvIqv mgq mgvb Z‡e 2wKwg c_ †ewk hv‡e]
4 5 5

5x  140  4x 37
 =
20 5
x+140 = 148
x = 8
So, at 4km he goes = 8km and at 5km he goes 35-8 = 27km
Total time taken,
8 27
+ = 2+5hr 24min = 7hr 24min
4 5
 gy‡L gy‡L : cÖ_g As‡ki MwZ‡eM 4 †_‡K 5 Kivq 1wKwg cÖwZN›Uvq †ewk hv‡e| Zvn‡j †h‡nZz †gv‡Ui
Dci 2wKwg †ewk †M‡Q †evSv hv‡”Q 4 wKwg †e‡Mi 2 N›Uv B 5 wKwg †e‡M cwieZ©b n‡q‡Q|
GLb 4wKwg †e‡M 2 N›Uvq 8 wKwg †M‡j Aewkó 27wKwg 5wKwg MwZ‡Z †M‡j mgq jv‡M 5N›Uv 24 wgwbU|
myZivs †gvU mgq = 2+5 N›Uv 24 wgwbU = 7 N›Uv 24 wgwbU|

Khairul’s IBA Math Book - 64

www.bdniyog.com
48. Kalam bought two varieties of rice, Costing Tk. 50/kg and Tk. 60/kg each, and mixed
them in some ratio. Then he sold the mixture at Tk. 70/kg. making a profit of 20%.
What was the ratio of the mixture? (Kvjvg 50 UvKv I 60 UvKv †KwR `‡i `yB ai‡bi wKQz Pvj wKbj
Ges GKwU wbw`©ó Abycv‡Z ‡gkvj Zvici †m 70 UvKv `‡i wewµ K‡i 20% gybvdv †cj| wgkÖ‡Yi AbycvZ KZ?)
(Union Bank. MTO.-2015) (IBA-MBA Admission Test, (Novem -2014 )
(a) 1:10 (b) 1:5 (c) 2:7
(d) 3:8 (e) None Ans:b
Solution:
g‡b Kwi, Kalam 50 UvKv `‡i wK‡bwQj x kg Pvj Ges 60 UvKv `‡i wK‡bwQj y kg Pvj|
Total cost price = 50x+60y (cÖwZ‡KRi `v‡gi mv‡_ cwigvY ¸Y )
and total selling price = 70 (x+y) (cÖwZ‡KwRi weµqg~‡j¨i mv‡_ cwigvY ¸Y)

cÖkœg‡Z, 120% of (50x + 60y) = 70 (x + y) [µqg~‡j¨i mv‡_ 20% jvf ‡hvM Ki‡j Zv weµqg~‡j¨i
mgvb]
6
 (50x + 60y) = 70x + 70y
5
x 1
 300x + 360y = 350x + 350y  - 50x = -10y  5x = y or.   x : y = 1 : 5
y 5

10 †m‡K‡Û Kivi Rb¨ Rule of Allegation

1st Item 120% of 50 2nd Item 120% of 60


60 72
Average
70

2 10
2:10 or 1:5
e¨vL¨v: GB cÖkœwU‡Z 50 I 60 µqg~j¨ wKš‘ 70 n‡”Q weµqg~j¨| A_©vr mgRvZxq n‡”Q bv| ZvB mgRvZxq evbv‡bvi
Rb¨ me¸‡jv‡K µqg~j¨ A_ev me¸‡jv‡K weµqg~j¨ evwb‡q wn‡me Ki‡Z n‡e| GLv‡b 50 I 60 UvKvi mv‡_ 20% jvf
hy³ K‡i me¸‡jv‡K weµqg~j¨ evbv‡bv n‡q‡Q| (Allegation G me¸‡jv GK RvZxq nIqv Avek¨K| )

49. In a class, 120 students are male and 100 students are female. 25% of the male students
and 20% of the female students are engineering students. 20% of male engineering
students and 25% of female engineering students passed the final exam. What
percentage of engineering students passed the exam?
(a) 5% (b) 10% (c) 16%
(d) 22% (c) None Ans:d
Solution:
Male engineer = 25% of 120 = 30
and Female engineer = 20% of 100 = 20
Total engineer = 30+20 = 50

Khairul’s IBA Math Book - 65

www.bdniyog.com
Now, male engineer passed = 20% of 30 = 6
Female engineer passed = 25% of 20 = 5
So, total passed = 6+5 = 11

11  100
Passed % of engineers =  22% (GLv‡b mevB A_©vr120+100 = 220 Gi Dci wn‡me bv K‡i
50
30+20= 50 Gi Dci wn‡me Ki‡Z n‡e KviY cÖ‡kœ †mfv‡eB ejv n‡q‡Q)

50. a2 is divisible by both 40 and 75. If ‘a’ has exactly three distinct prime factors (¯^Zš¿ †gŠwjK
Drcv`K), which of the following could be the value of ‘a’?
(a) 30 (b) 60 (c) 200
(d) 420 (c) None Ans:b
Solution:
40 = 85 = 235 (2Uv ¯^Zš¿ †gŠwjK Drcv`K Av‡Q)
75 = 253 = 52  3 (2Uv ¯^Zš¿ †gŠwjK Drcv`K Av‡Q hvi g‡a¨ 5 Av‡MiUv‡ZI Av‡Q wKš‘ bZzb 3)
GLb cÖ`Ë Ackb¸‡jvi g‡a¨ (2,3,5) GB wZbwU ¯^Zš¿ †gŠwjK Drcv`K Av‡Q Ggb †h †Kvb msL¨vB G¸‡jv Øviv
wefvR¨ n‡e Ges 2q kZ©wUI c~Y© Ki‡e|

(30)2 = 900 = GB msL¨vwU‡K 40 w`‡q fvM B Kiv hvq bv ZvB ev`|


(60)2 = 3600 G‡K 40 I 75 Dfq msL¨v w`‡q fvM Kiv hvq|
GLb 60 †K fv½‡j cvIqv hvq 60= (415) = (2235) A_©vr 3wU wfbœ †gŠwjK Drcv`K Av‡Q ZvB GUvB DËi|
420 n‡e bv KviY 60 w`‡q 420 ‡K fvM Ki‡j 7 nq|
Zvn‡ 60 Gi Drcv`K¸‡jvi mv‡_ bZzb K‡i 7 GKwU bZzb †gŠwjK msL¨v wb‡j (22357) = 4wU n‡q hv‡”Q|

51. There are 48 books on 3 shelves. If 3 books from third shelf are shifted to the second,
there will be same number of books on first and third shelves and double the number of
books on second shelf. How many books were there on three shelves originally?
(a) 10,15,13 (b) 12,21,15 (c) 11,23,14
(d) 13,19,16 (c) None Ans:b
Solution:
Let, after shifting total books at 3rd shelves = x and first shelves = also x
and 2nd shelves = 2x
ATQ,
x+2x+x = 48
4x = 48
x = 12
So, first shelves = 12 and 3rd shelves = 12 and 2nd Shelves = 212 =24
originally first shelves has = 12 books, 2nd shelves has 24-3 = 21 and 3rd shelves = 12+3 = 15

Khairul’s IBA Math Book - 66

www.bdniyog.com
52. Arafat has Tk 420. He purchased fifty mangoes and thirty oranges with the whole
amount. He then chose to return six mangoes for nine oranges as both quantities are
equally priced of each mango in Tk?
(a) 4.00 (b) 4.50 (c) 5.00
(d) 6.00 (c) None Ans: d
Solution:
6 6  30
9 oranges = 6 mangos 1 oranges = ‘’ 30 oranges = = 20 mangoes.
9 9

Now total mangos = 50+20 = 70 (Av‡Mi Avg 50Uv Avi 30wU Kgjv = 20 Avg )
So, price of 1 mango in Tk, = 42070 = 6 tk

53. Last year, Jaman bought two lamps. This year he sold them for BDT 2000 each. On one
lamp he made 25% profit, and on the other lamp he had 25% loss. What was his net
loss or profit?
(a) Loss BDT 1000 (b) Profit BDT 100 (c) Loss BDT 100
(d) No profit-Loss (e) None Ans:e
Solution:
in first lamp
125% = 2000
2000
So, 1 % =
125
2000 100
So, 100 % = = 1600
125
again ,
for first lamp
75% == 2000
2000
So, 1 % =
75
2000 100
So, 100 % = =2666.67
75

Total cost = 1600+2666.67 = 4266.67 Tk.


Totla sell = 2000+2000 = 4000
Total loss 4266.67 – 4000 Tk = 266.67 Tk

[¸iæZ¡c~Y© †bvU: 2000 UvKv †`qv Av‡Q Zv µqg~j¨ bq eis weµqg~j¨| myZivs 2000 Gi Dci jve-ÿwZ a‡i wn‡me
Ki‡j fzj DËi Avm‡e| ]

Khairul’s IBA Math Book - 67

www.bdniyog.com
54. If the average of 5 consecutive integers is 19 then what is the difference between the ;
least and the greatest of the integers?
(a) 4 (b) 5 (c) 8
(d) 10 (c) None Ans: a
Solution:
Let, first number = x
So, ATQ,
x + x+1 + x+2 + x+3 + x +4 = 195
5x = 85
x = 17
So, first number = 17 and last number is 17+4 = 21 and the difference is 21-17 = 4

GiKg AsK gy‡L gy‡LB Kiv Lye mnR| GLv‡b 19 †`qv bv _vK‡jI †h †Kvb 5wU msL¨v ci ci cvkvcvwk emv‡j ïiæi
Ges †k‡li msL¨v¸‡jvi g‡a¨ cv_©K¨ memgq 4 n‡e| wKš‘ mgm¨v n‡jv Gf‡e bv †f‡e mevB x w`‡qB ïiæ Kivi †Póv
K‡ib|

55. 150 individuals attend a marathon held in Sylhet. Of these only y participated is in the
marathon. If x of the 150 individuals from Sylhet and z of the individuals participated in
the marathon but were not from Sylhet. Which of the following represents the number
of individuals who did not participate in the marathon and were not from Sylhet?
(a) 150-x+2y (b) 150-x-y+z (c) 150-x+z
(d) 150-x-z (c) None Ans:d
Solution:
Total attendance = 150 (mevB wg‡j 150 Rb)
Participate = y (Giv AskMÖnY K‡iwQj)
Non participate = 150 –y (‡gvU- AskMÖnYKvix = AskMÖnY Kvix bq)
Total participate, y = x+z (AskMÖnYKvix = wm‡jwU + bb wm‡jwU)
Non participate and non Sylhetia =150-x-z

Khairul’s IBA Math Book - 68

www.bdniyog.com
Bangladesh Small and Cottage Industries Corporation (BSCIC)
Post name: Extension officer Exam date: 10-11-2018
Exam taker: IBA, DU.
1. Nine identical machines each working at same constant rate can stitch 27 jerseys in 4
minutes, How many minutes would it take 4 such machines to stitch 60 jerseys? (9wU †gwkb
GKwU wbw`©ó MwZ‡Z KvR K‡i 4 wgwb‡U 27 wU Rvwm© ˆZix Ki‡Z cv‡i| Zvn‡j GKB iK‡gi 4wU †gwkb w`‡q 60 wU
Rvwm© ˆZix Ki‡Z KZ wgwbU mgq jvM‡e?) [BSCIC-Extension Officer-2018]
a. 8 b. 12 c. 18 d. 20 e. None Ans: d
Solution:
9 machines stitch 27 jerseys in = 4 minutes
49
1 ‘’ ‘’ 1 ‘’ ‘’ = ‘’[‡gwkb Kg‡j †ewk mgq () Avevi Rvwm© Kg‡j Kg mgq () ]
27
4  9  60
4 ‘’ ‘’ 60 ‘’ ‘’ = ‘’[‡gwkb evo‡j Kg mgq Ges Rvwm© evo‡j †ewk mgq jv‡M]
27  4
= 20 minutes.

 Dc‡i wb‡P ¸Y fvM Kivi hyw³Uv Gfv‡e †gjv‡bv hvq| (HwKK wbq‡g me mgq 2q jvB‡bi D‡ëv KvR nq 3q
jvB‡b)
cÖ_gevi 9 wU †gwk‡bi 9 †h cv‡k hv‡e wb‡Pi 4 Zvi D‡ëvcv‡k hv‡e| ( 9 w`‡q ¸Y n‡j 4 w`‡q fvM n‡e)
Avevi, cÖ_g jvB‡bi 27 ‡h‡nZz wb‡P †M‡Q ZvB c‡ii 60 Dc‡i hv‡e|

2. A chemist is mixing acid and water. Out of the 30 ml mixed, 10 ml is acid. How much ml
of acid must be mixed such that a 50/50 mixture of water and acid is attained if no
additional water added (GKRb imvqbwe` wKQz GwmW Ges cvwb †gkv‡jb| 30 wgwj wjUv‡ii wgkÖ‡Y Gwm‡Wi
cwigvY 10 wgwj| H wgkÖ‡Y Avi KZ wgwj GwwmW †gkv‡j Ges cvwb bv †gkv‡j m¤ú~Y© wgkÖ‡Y cvwb Ges Gwm‡Wi cwigvY
50/50 n‡e?) [BSCIC-Extension Officer-2018]
a. 20 b. 10 c. 40 d. 30 e. None Ans: b
Solution: e¨vL¨v: 50/50 A_© n‡jv GwmW †gkv‡bvi ci
Acid in 30ml mixture = 10ml
wgkÖYwU‡Z GwmW Ges cvwbi cwigvY mgvb mgvb n‡e|
So, Water in the mixture = 30-10 = 20ml
Av‡M GwmW 10 I cvwb 20 _vKvq cvwbi mgvb GwmW
to make 50/50 water and acid in the mixture
Kivi Rb¨ GwmwW †gkv‡Z n‡e 20-10 = 10 wgwj|
Acid should be added = 20-10 = 10ml

3. The remainder when the positive integer m is divided by n is r. What is the remainder
when 2m is divided by 2n? (GKwU c~Y© abvZ¥K msL¨v m ‡K n w`‡q fvM Ki‡j fvM‡kl r _v‡K| hLb 2m ‡K
2n w`‡q fvM Kiv n‡e ZLb fvM‡kl KZ n‡e?) [BSCIC-Extension Officer-2018]
a. r b. 2r c. 2n d. m-nr e. None
Ans: b
Solution:
m ‡K n w`‡q fvM Ki‡j hw` fvM‡kl r _v‡K Zvn‡j 2m ‡K 2n w`‡q fvM Ki‡j fvM‡kl 2r _vK‡e|

Khairul’s IBA Math Book - 69

www.bdniyog.com
cÖgvY: aiæb: m = 10 Ges n = 8 Zvn‡j 8 w`‡q 10 †K fvM Ki‡j fvM‡kl 2 n‡e| GLb 10 ‡K wظY Ki‡j n‡e 20
Ges 8 †K wظY Ki‡j n‡e 16 | 16 w`‡q 20 †K fvM Ki‡j fvM‡kl 4 n‡e| hv Av‡Mi fvM‡kl 2 Gi wظY|

4. A tank is 40% full of water. If 9 liters of water is removed, the tank becomes 1/4th full.
What is the capacity of the tank in liter? (GKwU U¨vs‡Ki 40% cvwb w`‡q c~Y© Av‡Q| hw` 9 wjUvi cvwb
†ei Kiv nq Zvn‡j U¨vsKwUi 1/4 Ask c~Y© _v‡K| U¨vsKwUi aviY ÿgZv KZ wjUvi?) [BSCIC-Extension Officer-
2018]
a. 20 b. 30 c. 40 d. 60 e. None Ans: d
Solution:
mvaviY wbq‡g mgvavb:
Let, the capacity of the tank = x liter
ATQ,
1
40% of x – 9 = x Shortcut: (cixÿvi n‡j GB wbq‡g Kiv me‡_‡K mnR n‡e)
4
1
2x x = 25% myZivs 40% †_‡K 9 wjUvi †ei K‡i 25% n‡j
  =9 4
5 4 9 9  100
8x  5x 40-25= 15% = 9, 1% = 100% = = 60wjUvi|
 =9 15 15
20
 3x = 180
x = 60 myZivs aviY ÿgZv = 60 wjUvi|

5. Which one of the following fractions is a result of the sum of an integer and its
reciprocal? (wb‡Pi †Kvb fMœvskwU GKwU c~Y© msL¨v Ges Zvi wecixZ fMœvs‡ki †hvMd‡ji mgvb?) [BSCIC-
Extension Officer-2018]
15 17 36 65
a. b. c. d. e. None Ans: d
8 5 7 8
Solution:
1
Let the number be = x so, reciprocal of x =
x
1 x2 1
Sum = x+ = (GLvb †_‡K †evSv hv‡”Q †h msL¨vwU hv B †nvK bv †Kb msL¨vwU I Zvi wecixZ
x x
fMœvs‡ki †hvMdj n‡e GKwU fMœvsk hvi wb‡P msL¨vwU Ges Dc‡i H msL¨vwUi e‡M©i mv‡_ 1 †hvM )
65 x 2  1 82  1
Ackb¸‡jvi g‡a¨ ïaygvÎ d ‡Z cÖ`Ë fMœvskwU †K = Gfv‡e cÖKvk Kiv hvq|
8 x 8

Khairul’s IBA Math Book - 70

www.bdniyog.com
6. Which one of the following is the minimum value of the sum of two integers whose
product is 36 ? (`ywU msL¨vi ¸Ydj 36 n‡j H msL¨v `ywUi me©wb¤œ †hvMdj wb‡Pi †KvbwU?) [BSCIC-Extension
Officer-2018]
a. 12 b. 13 c. 15 d. 20 e. None Ans: a
Solution:
36 = 66, Zvn‡j 6+6 = 12
36 = 94, Zvn‡j 9+4 = 13 g‡b ivLyb:
36 = 123, Zvn‡j 12+3 = 15 GiKg cÖ‡kœi †ÿ‡Î mvaviYZ: me‡_‡K KvQvKvwQ
36 = 182, Zvn‡j 18+2 = 20 msL¨v `ywU wb‡jB †hvMdj me©wb¤œ nq|
36 = 361, Zvn‡j 36+1 = 37
‡`Lv hv‡”Q cÖ_g jvB‡bi 36 = 66, n‡j 6+6 = 12 B n‡”Q me©wb¤œ |
7. If A,B,C,D and E are points in a plane such that the line CD bisects angle ACB and line
CB bisects right angle ACE, then angle DCE = ? (A, B, C, D Ges E n‡jv GKB mgZ‡j 5wU we›`y |
†hLv‡b CD ‡iLv ACB ‡KvY‡K mgwØLwÛZ K‡i‡Q Ges CB ‡iLv ACE mg‡KvY‡K mgwØLwÛZ K‡i‡Q| Zvn‡j
DCE ‡Kv‡Yi cwigvc KZ?) [BSCIC-Extension Officer-2018]
a. 450 b. 22.50 c. 67.50 d. 850 e. None Ans: c
Solution:
cv‡ki wPÎwU †`Lyb:
cÖ_‡g ACE GK mg‡KvY ev 900 †K CB mgvb `yÕfv‡M fvM Ki‡j
Dfq cv‡k 450 wWwMÖ K‡i `ywU bZzb †KvY Kivi ci Avevi CD ‡iLv 2
2
w`‡q ACB ‡KvY †K 2 fv‡M fvM Ki‡j 450 Gi A‡a©K 22.50
n‡e| wKš‘ cÖ‡kœ Rvb‡Z †P‡q‡Q DCE = KZ? cv‡ki wPÎwU †_‡K
†`Lv hv‡”Q DCE = BCE+DCB = 450+22.50 = 67.50
= fv‡jvfv‡e †evSvi Rb¨ cv‡ki wPÎwU †`Lyb|
 civgk©: GiKg cÖ‡kœi DËi ïay cÖkœ c‡o DËi ‡`qv KwVb|
R¨vwgwZi GiKg †h †Kvb cÖ‡kœi DËi †`qvi Rb¨ ïiæ‡Z wPÎ Kíbv Kiv A‡bK †ewk ¸iæZ¡c~Y©|
ZvB wPÎ Kíbv Kivi ci GB ai‡Yi cÖ‡kœi DËi †`qv A‡bK mn‡R mgvavb Kivi civgk© iB‡jv|
8. What is the average of the series 1, 5, 9 ……….. 81 ? (1, 5, 9... 81 avivwUi Mo KZ?)
[BSCIC-Extension Officer-2018]
a. 39 b. 40 c. 41 d. 42 e. None Ans: c
Solution:
1, 5, 9... 81 wmwiRwU GKwU mgvšÍi aviv myZivs Gi Mo n‡e: cÖ_g I †klc‡`i M‡oi mgvb|
81  1 82
A_©vr =  41 (mgvšÍi avivi †h †Kvb wmwiR Avm‡j Zv‡`i Mo Gfv‡e †ei Ki‡Z nq|)
2 2
9. If 7 workers can assemble a car in 8 hours, how long would it take 12 workers to
assemble the same car? (7 Rb kÖwgK GKwU Kvi 8 NÈvq mvRv‡Z cv‡ib| 12 Rb kÖwg‡K H KviwU
mvRv‡Z KZ mgq jvM‡e?) [BSCIC-Extension Officer-2018]
a. 3 hours b. 3 hours 30 minutes
c. 5 hours d. 4 hours 40 minutes e.None Ans: d

Khairul’s IBA Math Book - 71

www.bdniyog.com
Solution:
7 workers need = 8 hours.
 1 ‘’ ‘’ = 87 hours.
8 7 14
 1 ‘’ ‘’ = hours. = = 4 hrs 40min
12 3
wk‡L ivLyb: fMœvsk Iqvjv NÈv‡K wKfv‡e wgwbU evbv‡Z nq?
14
NÈvi Dc‡ii 14 ‡K 3 w`‡q fvM Ki‡j fvMdj 4 Gici fvM‡kl 2 NÈv‡K 60w`‡q ¸Y Ki‡j 120 wgwbU nq
3
| GLb 120 wgwbU ‡K Avevi wb‡Pi 3 w`‡q fvM Ki‡j fvMdj 40 wgwbU nq| DËi: 4 NÈv 40 wgwbU|
10. When squared value of a number is subtracted from 5 times of a number the result is 6.
What is the number? (hLb GKwU msL¨vi 5 ¸Y †_‡K H msL¨vi eM©‡K we‡qvM Kiv nq ZLb we‡qvMdj 6 nq|
msL¨vwU KZ?) [BSCIC-Extension Officer-2018]
a. -2 b. 2 c. -3 d. 4 e. None Ans: b
Solution:
Let the number be x Shortcut:
ATQ, Ackb †_‡K 2wb‡q †`Lv hvq 2 Gi 5 ¸Y = 10 Ges 2 Gi
2
5x – x = 6 eM© we‡qvM Ki‡j 10-4 = 6 nq| (wg‡j hvIqvq DËi 2)
or, -x2+5x-6 = 0 (-2) A_ev (-3) wb‡q KbwdDkb jvM‡j:
or, x2-5x+6 = 0
(-2)5 - (-2)2 = -10 -4 = -14
or, x2 – 3x – 2x +6 = 0
or, (x-3) (x-2) = 0 (-3)5 - (-3)2 = -15 -9 = -24 (`y‡UvB wgj‡Q bv|)
So,
Either x-3 = 0 x = 3
or, x-2 = 0  x = 2 Since x = 3 is not given, so ans is 2
11. A completes a job in 6 days, B in 12 days, C in 20 days and D in 24 days. If Tk 8670 is to
be divided among them, then B will get how much more than C ? (A, B, C Ges D GKwU
KvR h_vµ‡g 6, 12, 20 Ges 24 w`‡b †kl Ki‡Z cv‡i| hw` Zv‡`i gv‡S 8670 UvKv †`qv nq Zvn‡j B, C Gi
†_‡K KZUvKv †ewk cv‡e?) [BSCIC-Extension Officer-2018]
a. 570 b. 680 c. 1020 d. 1200 e. None Ans: e
1 1 1 1
Solution: A, B, C, & D’s 1 day’s work =   
6 12 20 24
1 1 1 1 1 1 1 1
Ratio of their work = : : : or,  120 :  120 :  120 :  120 = 20:10:6:5
6 12 20 24 6 12 20 24
4
So, Difference of amount of B and C is 8670 = Tk. 846 (Approx)
41
G ai‡Yi cÖkœ wKfv‡e mgvavb Ki‡Z nq?
cÖ_‡g Zv‡`i 1 w`‡bi Kv‡Ri Ask †ei K‡i †mB Ask‡K AbycvZ mvRv‡Z nq| Gici cÖvß UvKv ¸‡jv‡K †mB
Abycv‡Z fvM K‡i w`‡jB n‡q hv‡e|
jwRK: †h Kg w`‡b KvR Ki‡e †m Kg UvKv cv‡e Ggb fvev fzj| KviY †h Kg w`‡b KvRwU †kl Ki‡Z cvi‡e †m
Ges †h †ewkw`‡b KvRwU †kl Ki‡Z cvi‡e †m mn `yR‡b GKB mgq a‡i KvR Ki‡j Kg w`‡b †kl Ki‡Z cviv
†jvKwU Aí mg‡q †ewk KvR Ki‡Z cvi‡e| ZvB UvKvi fv‡Mi mgq †m †ewk UvKv cv‡e|

Khairul’s IBA Math Book - 72

www.bdniyog.com
12. Karim has bought some pens for 120 Taka. If each pen cost him 2 taka less then he
could buy 2 more pens . How many pens did he buy? (Kwig 120 UvKv w`‡q wKQz Kjg wKbj| hw`
cÖwZwU Kj‡gi `vg 2 UvKv Kg n‡Zv Zvn‡j †m Av‡iv 2wU AwZwi³ Kjg µq Ki‡Z cvi‡Zv| †m KZwU Kjg wK‡bwQj?)
[BSCIC-Extension Officer-2018]
a. 8 b. 10 c. 12 d. 14 e. None
Ans: b
Solution:
Let,
The number of pens Karim bought = x (‡h‡nZz Kj‡gi msL¨v Rvb‡Z †P‡q‡Q ZvB Kjg‡K aiv †eUvi)
ATQ,
120 120
  2 (wb‡Pi msL¨v¸‡jv n‡jv Kj‡gi msL¨v Ges Kj‡gi msL¨v w`‡q 120 †K fvM Kivq cÖ_g I 2q
x x2
ev‡ji Kj‡gi `v‡gi cv_©K¨ = 2 UvKv|)
120 x  240  120 x
2
x ( x  2)
Shortcut: 10 ‡m‡K‡Û DËi †ei Kivi Rb¨ Gfv‡e fveyb:
240
 2 2 120 UvKv‡K Ggb 2wU msL¨vq fv½‡Z n‡e hv‡Z cv_©K¨ 2 nq| (KviY Av‡Mi
x  2x Kj‡gi †_‡K eZ©gvb Kjg 2wU †ewk)
2x2+4x = 240
GLb 1012 = 120 ai‡j cÖ_‡g 10 Kjg wK‡bwQj|
 x2+2x-120 = 0
 x2+12x-10x-120 = 0 (GB jvBbUv †_‡K kU©KvU evbv‡bvi jwRK ‡Wfjf Kiv hvq|)
 x (x+12)-10(x+12) = 0
(x+12) (x-10) = 0
Either x+12 = 0 x=-12 (Negative value is not acceptable)
or, x-10 = 0 x = 10
g‡b ivLyb: Dˇii mc‡ÿ cÖgvY:
120 UvKvq cÖ_‡g 10wU Kjg wKb‡j cÖwZwUi `vg 12010 = 12UvKv|
c‡i 2wU Kjg †ewk †c‡j cÖwZwUi `vg 120  12 = 10UvKv| hv Av‡Mi `v‡gi †_‡K 2UvKv Kg|

13. If x+y = 7 and x2+y2 = 25, then which of the following equals the value of x3+y3 ? (x+y =
7 Ges x2+y2 = 25 n‡j wb‡Pi †KvbwU x3+y3 Gi gvb?) [BSCIC-Extension Officer-2018]
a. 7 b. 25 c. 65 d. 91 e. None Ans: d
Solution:
myÎ cÖ‡qv‡M mgvavb:
Avgiv Rvwb,
x2+y2 = (x+y)2 – 2xy (GB m~ÎwU cÖ‡qvM Ki‡j xy Gi †h gvb cvIqv hv‡e Zv x3-y3 Gi m~‡Î Kv‡R jvM‡e|)

Khairul’s IBA Math Book - 73

www.bdniyog.com
25 = 72 -2xy
2xy = 49-25 g‡b ivLyb:
24 GB ai‡Yi cÖkœ m~Î cÖ‡qvM K‡i Ki‡Z †M‡j 73 Gi gZ eo eo ¸Y
xy =  12
2 Ki‡Z wM‡q †ewk mgq bó n‡e|
Zvn‡j, A_P KZ mn‡R fvev hvq †`Lyb|
x3+y3 = (x+y)3-3xy(x+y) x+y = 7 Ges x2+y2 = 25 n‡j 4+3 = 7 I 42+32 = 16+9 = 25
= 73 –3127 nq| A_©vr x = 4 I y = 3 Zvn‡j x3+y3 = 43+33 64+27 = 91
= 343 –252
= 91

14. The selling price of 15 items equals the cost of 20 items. What is the percentage profit
earned by the seller? (hw` 15wU AvB‡U‡gi weµqg~j¨ 20wU AvB‡U‡gi µqg~‡ji¨ mgvb nq Zvn‡j †mB we‡µZvi
kZKiv KZ jvf n‡q‡Q?) [BSCIC-Extension Officer-2018]
a. 15% b. 20% c. 25% d. 33.33% e. None Ans: d
Solution:
Here,
CP of 20 items.= SP of 15 items
Let,
CP of 1 item = 1Tk So, CP of 20 items = 20tk, and CP of 15 items = 15Tk.
Since SP of 15 items=20 Tk(‡h‡nZz 15Uvi weµqg~j¨ =20Uvi µqg~‡j¨i mgvb A_©vr 20UvKv=20
UvKv|)
So, profit = 20-15 = 5Tk (GB 5 UvKv n‡jv 15UvKvq Kiv jvf)
5  100
Profit % = = 33.33%
15
Super Shortcut:
GiKg cÖ‡kœi DËi gvÎ 5 †m‡K‡Û w`‡Z PvB‡j Gfv‡e fveyb:
‡hUv weµq msL¨v †mUvB Dc‡ii wbq‡g µqg~j¨ n‡q hvq| ZvB ej‡Z n‡e 15Uv‡Z 5Uv jvf| A_©vr 3
fv‡Mi 1 fvM jvf n‡j gy‡L gy‡L ejv hvq jv‡fi nvi 33.33% |

15. In a class, 25 students play cricket 25 students play football & 10 students play both. 10
students play neither cricket nor football. What is the number of students in the class?
(GKwU K¬v‡mi †gvU QvÎ-QvÎx‡`i g‡a¨ 25Rb wµ‡KU †L‡j, 25 Rb dzUej †L‡j, 10 Rb wµ‡KU †L‡j Ges 10 Rb
‡KvbwUB †L‡j bv| H K¬v‡m †gvU KZRb QvÎ-QvÎx Av‡Q?) [BSCIC-Extension Officer-2018]
a. 40 b. 50 c. 60 d. 65 e. None Ans: b
Solution: Cricket Football=2
cv‡ki wPÎwU Abymv‡i Ki‡Z cv‡ib| =25 5
A_ev m~Î cÖ‡qv‡M wb‡Pi wbq‡g:
Total = all single – both + none 2 2
5- 25-
5-
Total = 25+25-10+10 1 10
1
Total = 50
Total: 15+10+15+10 = 50

Khairul’s IBA Math Book - 74

www.bdniyog.com
16. The length of a rectangular field is 30 meter. A fence surrounding the field has a total
length of 140 meter, What is the length (in meter) of the diagonal path of this
rectangular field? (GKwU AvqZvKvi gv‡Vi ˆ`N©¨ 30 wgUvi| H gv‡Vi 4 cv‡k 140 wgUvi ˆ`‡N©¨i GKwU †eov
Av‡Q| H gv‡Vi KY© eivei iv¯Ívi ˆ`N©¨ KZ? ) [BSCIC-Extension Officer-2018]
a. 40 b. 50 c. 60 d. 70 e. None Ans: b
Solution:
Let,
another side of the rectangle = x A B
ATQ,
2(30+x) = 140 ?
or, 30+x = 70 30
x = 40
Now diagonal of the rectangle: C 40 D
( 40)  (30) = 1600  900 = 2500 = 50
2 2

17. The product of two consecutive negative even integers is 24. What is the larger
number? (`ywU avivevwnK †Rvo FYvZ¥K msL¨vi ¸Ydj 24| e„nËg msL¨vwU KZ? ) [BSCIC-Extension Officer-
2018]
a. - 4 b. -6 c. 4 d. 6 e. -8 Ans: a
Solution:
24 = 64 (`ywU avivevwnK msL¨v)
wKš‘ FYvZ¥K avivevwnK ejvq: 24 = (-6)(-4) n‡e | †hLv‡b -6 ‡QvU msL¨v Ges -4 eo msL¨v|
18. If (x+3)2 = 225, which of the following can be the value of x-1 ? (hw` (x+3)2 = 225 nq Zvn‡j
wb‡Pi †KvbwU x-1 Gi gvb n‡Z cv‡i?) [BSCIC-Extension Officer-2018]
a. 13 b. 12 c. -12 d. -16 e. -19 Ans: e
Solution:
(x+3)2 = 225  x+3 = 225  x+3 =  15 (exRMwY‡Zi †ÿ‡Î + Ges - `y‡Uv gvb B wb‡Z n‡e)
x+3 = +15 n‡j, x = 15-3 = 12 myZivs cÖ‡kœi cÖ`Ë x-1 Gi gvb n‡e 12-1 = 11
Avevi,
x+3 = -15 n‡j, x = -15-3 = -18 myZivs cÖ‡kœi cÖ`Ë x-1 Gi gvb n‡e -18-1 = -19
GLb Ack‡b 11 bv‡g †Kvb Ackb bv _vKvq DËi n‡e -19
19. Tahmid is four times as old as Koushik. In x years, Tahmid will be three times as old
Koushik. How old is Koushik in terms of x ? (Zvnwg‡`i eqm Zvi fvB †KŠwk‡Ki 4¸Y| x eQ‡ii g‡a¨
Zvnwg‡`i eqm Zvi ‡KŠwk‡Ki eq‡mi 3¸Y n‡e| x Gi wfwˇZ †KŠwk‡Ki eqm KZ?) [BSCIC-Extension Officer-
2018]
a. 2x b. 3x c. 4x d. 8x e. 12x Ans: a
Solution:
Let, Koushik is = p year old (‡h‡nZz cÖ‡kœ ejv n‡q‡Q x eQi ci| ZvB eqm x bv a‡i p aiv n‡jv|)
So, Tahmid is = 4p
ATQ, ‡R‡b ivLyb:
3(p+x) = (4p+x) ‡h †Kvb cÖ‡kœ In terms of x ‡jLv _vK‡j Ggbfv‡e
3p + 3x = 4p + x DËi †ei Ki‡Z n‡e †hLv‡b x _vK‡Z n‡e|

Khairul’s IBA Math Book - 75

www.bdniyog.com
 - p = -2x
 p = 2x (‡h‡nZz p = ‡KŠwk‡Ki eqm| ZvB p Gi gvb B n‡jv ‡KŠwk‡Ki eqm|)
20. On a youth soccer team, the ratio of boys to girls is 6 in 7. If there are 2 more girls than
boys on the team, how many boys are on the team ? (GKwU hye dzUej `‡j, †Q‡j I †g‡q‡`i AbycvZ
6:7| hw` H `‡j †Q‡j‡`i †_‡K †g‡qi msL¨v 2 Rb †ewk nq Zvn‡j H `jwU‡Z †gvU KZRb †Q‡j Av‡Q?) [BSCIC-
Extension Officer-2018]
a. 36 b. 24 c. 18 d. 12 e. None Ans: d
Solution:
Let, boys = 6x and 7x
ATQ,
7x-6x = 2
x=2
So, boys are = 62 = 12
================================

Khairul’s IBA Math Book - 76

www.bdniyog.com
City Bank Ltd.
Post name : Management Trainee officer Exam date: 20-07-2018
Exam taken by: IBA, DU.

1. A and B started a partnership business investing some amount in the ratio of 3 : 5. C


joined them after six months with an amount equal to that of B. In what proportion
should the profit at the end of the first year be distributed among A, B and C? (A Ges B
GKwU e¨vemv ïiæ Ki‡jv ‡hLv‡b `yR‡bi wewb‡qv‡Mi AbycvZ 3:5| 6 gvm ci C Zv‡`i mv‡_ B Gi mgcwigvY wewb‡qvM
wb‡q †hvM`vb Ki‡jv| cÖ_g eQi †k‡l A, B Ges C Gi jv‡fi AbycvZ KZ n‡e? ) [DBBL (AO)-2017]& [City
Bank (MTO)-2018]
a. 3:5:2 b. 3:5:5 c. 6:10:5 d.None Ans: c
Solution:
Let, the initial investments of A and B be 3x and 5x.
A : B : C = (3x  12) : (5x  12) : (5x  6) [A= 12 gvm, B = 12 gvm wKš‘ C = 6gvm|]
= 36 : 60 : 30 = 6 : 10 : 5

2. Solution Y is 40 percent sugar by volume, and solution X is 20 percent sugar by volume.


How many gallons of solution X must be added to 150 gallons of solution Y to create a
solution that is 25 percent sugar by volume? (wgkÖY Y G 40% wPwb Ges wgkÖY X G 20% wPwb Av‡Q|
Y wgkÖ‡Yi 150 M¨vj‡Yi mv‡_ X wgkÖ‡Yi KZ M¨vjY †gkv‡j bZzb wgkÖ‡Y wPwbi cwigvY 25% n‡e?) [DBBL (AO)-
2017] & [City Bank (MTO)-2018]
a. 75 b. 150 c. 240 d. 450 Ans: d
Solution:
Let, the mixture of X = P gallons
ATQ
40% of 150 +20% of P = 25% of (150+P)
P 150  P 300  P 150  P
60+     750+5P = 1200+4P P = 450
5 4 5 4

Shortcut: 10 ‡m‡K‡Û mgvav‡bi Rb¨ me‡_‡K Kvh©Kix iæjm:


By the rule of allegation,
Mixture Y Mixture X
40% 20%
Final mixt:
25%

5% 15%
st nd
Ratio of 1 and 2 mixture = 1 : 3
Now 1 part of Y = 150 So, 3 parts of X = 1503 = 450
Khairul’s IBA Math Book - 77

www.bdniyog.com
3. It takes 1 pound of flour to make y cakes. The price of flour is w dollars for x pounds. In
terms of w, x and y; what is the dollar cost of the flour required to make 1 cake? (y wU
‡KK evbv‡Z 1 cvDÛ gq`vi cÖ‡qvRb| Avevi, x cvDÛ gq`vi `vg w Wjvi| Zvn‡j 1wU †KK evbv‡Z KZ Wjvi LiP
n‡e Zv w,x Ges - y Gi mv‡c‡ÿ cÖKvk Kiæb?) [City Bank (MTO)-2018]
a. xy/w b. w/xy c. y/wx d. wx/y Ans: b
Solution:
x cvD‡Ûi `vg = w Wjvi n‡j
w ‡evSvi Av‡Q A‡bK wKQz:
1 ÕÕ ÕÕ = Wjvi| (Kg jvM‡e ZvB fvM)
x &GB cÖkœ mn †h †Kvb cÖ‡kœ in terms of
Avevi, w,x, y ej‡j w,x y Gi mv‡c‡ÿ A_©
w Ggbfv‡e DËi †ei Ki‡Z n‡e hv‡Z
y wU †K‡Ki Rb¨ jv‡M 1 cvDÛ ‡KK hvi g~j¨ = Wjvi|
x Dˇii g‡a¨ w,x,y _v‡K|
w w
myZivs 1wU †KK evbv‡Z LiP n‡e = Wjvi| DËi:
xy xy

3 2
4. If of the number of women working at Company X is equal to of the number of
4 3
men, what fraction employees at Company X are women? (GKwU ‡Kv¤úvwb‡Z Kg©iZ kÖwg‡Ki g‡a¨
bvix kÖwg‡Ki 3/4 Ask cyiæl kÖwg‡Ki 2/3 As‡ki mgvb| H ‡Kv¤úvwbi KZ Ask kÖwgK bvix ? [DBBL (AO)-2017]
& [City Bank (MTO)-2018]
5 1 7 8
a. b. c. d. Ans: d
12 2 12 17
Solution:
awi, bvix kÖwg‡Ki msL¨v x Rb
Shortcut: fMœvsk `ywUi j‡ei j.mv.¸ 6 a‡i
Ges cyiæl kÖwg‡Ki msL¨v y Rb
Zvn‡j †gvU kÖwgK msL¨v ( x + y ) Rb
3 4
bvix = 6 n‡j ‡gvU 1 Ask = 6 = 8,
4 3
 x 
‡gvU kÖwg‡Ki Zzjbvq bvix kÖwg‡Ki Ask =   2 3
x  y Avevi, cyiæl
3
= 6 n‡j 1 Ask = 6
2
=9
cÖkœg‡Z,
Zvn‡j bvix + cyiæl †gvU 8+9 = 17
3x 2 y 9x
 y= 8
4 3 8 ‡hLv‡b bvix = 8 myZivs bvixi Ask
17
 x  x x x 8 {Gfv‡e kZ© wgj †i‡L j.mv.¸ evwb‡q msL¨v ai‡j 2
GLb,   =   
x  y 9 x 8x  9 x 17 x 17 wgwb‡Ui DËi 20 †m‡K‡Û †ei Kiv m¤¢e|}
x
8 8 8

5. If the circle with center O has area 9π, what is the area of equilateral triangle ABC? ( O
‡K›`ª wewkó GKwU e„‡Ëi †ÿÎdj 9π . Zvn‡j wP‡Î AswKZ ABC wÎfz‡Ri †ÿÎdj KZ ?) [DBBL (AO)-2017] &
[City Bank (MTO)-2018]
a. 9 3 b. 12 3 c. 18 d. 24 Ans: b
Solution:
Khairul’s IBA Math Book - 78

www.bdniyog.com
awi, e„‡Ëi e¨mva© OA = r Ges e„‡Ëi ‡ÿÎdj πr2
cÖkœg‡Z,
πr2 = 9π  r2 = 9  r = 3
AZGe, AD = OA + OD = 3+3 = 6
GLb wÎfzR ABD mg‡KvYx wÎfz‡R [ †h‡nZz ADB = 90o ]
AwZf’R = AB, BD f~wg Ges AD j¤^ = 6|
Avgiv Rvwb, mgevû wÎfz‡Ri cÖ‡Z¨K †Kv‡Yi gvb 60o
‡mB wn‡m‡e ABD = 60o
Zvn‡j wÎfzR ABD ‡_‡K Avgiv cvB
AD
Sin 600 = [‡h‡nZz mg‡KvYx wÎfy‡Ri j¤^ = 6 †`qv Av‡Q Ges AwZfz‡Ri gvb †ei Ki‡Z n‡e|]
AB
3 6 12
= AB 3 =12 myZivs AB = 3
2 AB
2
3  12  3 144
GLb ABC mgevû wÎfz‡Ri †ÿÎdj =   =   12 3 Ans:
4  3  4 3

6. If the average (arithmetic mean) of seven consecutive integers is k + 2, then the product
of the greatest and least integer is : (7 wU avivewnK msL¨vi Mo n‡”Q k + 2. e„nËi I ÿz`ªZg msL¨v `ywUi
¸Ydj KZ n‡e?) -[DBBL (AO)-2017] & [City Bank (MTO)-2018]
a. k2+4k-5 b. k2-9 c. k2+6k+9 d. k2-2k+1 Ans: a
Solution:
The series of the 7 consecutive integers is like k-1, k, k+1, k+2, k+3, k+4 & k+5 (avivevwnK
msL¨vi †gvU msL¨v we‡Rvo n‡j Mo n‡e gv‡Si msL¨vwU Ges 7wU msL¨v _vKvq ev‡g 3wU I Wv‡b 3wU|)
So, the product of the greatest and least = (k-1)(k+5) = k2+5k-k-5 = k2+4k-5

e¨vL¨v: avivevwnK msL¨v †gvU 7wU Ges G‡`i Mo k+2 | GB Mo n‡”Q H msL¨v¸‡jvi ga¨vg msL¨v| ZvB eo msL¨vwU
n‡e k+2+3 = k+5 (KviY 7wU msL¨vi gv‡Si msL¨vi †_‡K 7g msL¨vwU 3 eo Ges 1g msL¨vwU 3 †QvU| )
e„nËi I ÿz`ªZg msL¨v `ywUi ¸Ydj n‡e = (k-1) (k+5) = k2 + 5k – k – 5 = k2 + 4k – 5

7. The sum of the pre-tax costs of item A and item B is Tk.300. In Dhaka, each item would
be charged a flat 7% tax. In Rangpur, Item A is subject to 5% tax and item B is subject
to 10% tax. If the tax in Rangpur on the purchase of both items is exactly Tk. 3 more
than it is in Dhaka, then what is the pre-tax price of item A ? (Ki ‡`qvi c~‡e© AvB‡Ug A Ges
AvB‡Ug B Gi †gvU LiP GK‡Î 300 UvKv| XvKvi g‡a¨ Dfq AvB‡U‡gi Dci 7% nv‡i Ki w`‡Z nq| iscy‡i AvB‡Ug
A Gi Dci 5% Ges AvB‡Ug B Gi Dci 10% nv‡i Ki w`‡Z nq| `ywU AvB‡Ug B iscyi †_‡K µq Ki‡j XvKvi
Zzjbvq †gv‡Ui Dci 3UvKv †ewk Ki w`‡Z nq| Ki †`qvi c~‡e© AvB‡Ug A Gi g~j¨ KZ?) [City Bank (MTO)-18]
a. 120 b. 200 c. 300 d. 320 Ans: a
Solution:
Written solution:

Khairul’s IBA Math Book - 79

www.bdniyog.com
Let, Item A cost = x and item B cost = y

1st Condition,
x+y = 300------ (i)
2nd Condition,
(5% of x + 10% of y) - (7% of x + 7% of y ) = 3 (‡h‡nZz iscy‡ii Ki - XvKvi Ki = 3 UvKv)
5x 10y 7 x 7 y
or,    3
100 100 100 100
or, 5x+10y-7x-7y = 300 [100 w`‡q Dfqcÿ‡K ¸Y K‡I, (me¸‡jvi wb‡P 100 _vK‡j Gfv‡e Ki‡Z nq)|]
or, -2x+3y = 300 ----- (ii)
By multiplying equation (i) by 3 we get,(3 w`‡q ¸Y Ki‡j y ev` hv‡e Ges AvB‡Ug A Gi gvb ‡ei n‡e)
3x+3y = 900
-2x+3y = 300 (we‡qvM Ki‡j wPý cv‡ë hv‡e|)
--------------------------
5x = 600 [by subtracting]
x = 120
So, the price of item A is 120. Ans: 120

Shortcut solution:
Ackb a‡i wn‡me wgwj‡q w`‡jB `ªæZ DËi †ei n‡e|

Ack‡bi 120 †K AvB‡Ug A Gi `g ai‡j AvB‡Ug B Gi `vg n‡e 300-120 = 180


Ges XvKvq ‡gvU U¨v· n‡e 300 Gi 7% = 21 UvKv | ( †h‡nZz Dfq †ÿ‡ÎB U¨v‡·i nvi mgvb)
Avevi iscy‡i U¨v‡·i cwigvY n‡e 120 Gi 5% = 6 Ges 180 Gi 10% = 18 †gvU 6+18 = 24
XvKvi †_‡K iscy‡ii U¨v‡·i cwigvY †ewk n‡e 24-21 = 3 UvKv| wg‡j hvIqvq AvB‡Ug A Gi `vg 120 UvKv|

8. If a, b, c and d are different integers between - 6 and 10 inclusive, what is the least
possible value of the product abcd? (a, b, c Ges d -6 †_‡K 10 ch©šÍ ( G `ywU mn) wfbœwfbœ c~Y© msL¨v|
Zvn‡j abcd Gi ¸Yd‡ji ÿz`ªZg gvb KZ n‡e?) [DBBL (AO)-2017] & [City Bank (MTO)-2018]
a. -480 b. -1800 c. -3600 d. -4320 Ans: d
Solution:
Let, a = - 6, b = 10 ; c = 9; and d = 8 [Since a, b, c, d are different integers]
Product of abcd = (- 6) (10  9  8) = - 6 720 = - 4320 Ans: - 4320

Interesting Point: ‡h‡nZz me‡_‡K ÿz`ªZg ¸Ydj †ei Ki‡Z ejv n‡q‡Q Ges - 6 Gi †_‡K 10 eo| ZvB eo
eo wZbwU abvZ¥K msL¨v ¸Y Kivi ci me‡_‡K eo FYvZ¡K msL¨v w`‡q ¸Y Kivq me‡_‡K ÿz`ªZg ¸Ydj †ei n‡q‡Q|

Khairul’s IBA Math Book - 80

www.bdniyog.com
9. A businessman has 1000 kg of sugar, part of which he sells at 8% profit and the rest at
18% profit. He gains 14% on the whole. The quantity sold at 18% profit is: (GKRb e¨emvqx
Gi Kv‡Q 1000 †KwR wPwb Av‡Q| Zvi ga¨ †_‡K wKQz wPwb 8% jv‡f wewµ Ki‡jb Ges Aewkó wPwb 18% jv‡f wewµ
Ki‡jb| †gv‡Ui Dci Zvi 14% (1000 ‡KwR wewµ‡Z) jvf n‡j 18% jv‡f KZ †KwR wewµ K‡iwQ‡jb?) [BD
House Building FC (OF)-2015] , [BD House Building FC (OF)-2017] , [Pubali Bank (TAT)-2017],
[DBBL (AO)-2017] & [City Bank (MTO)-2018]
a. 400kg. b. 600kg. c. 640kg. d. 700kg. Ans: b
 Written Solution:
Let the sugar of 18% profit is = x
So, the sugar of 8% profit = 1000-x
ATQ,
18% of x + 8% of ( 1000-x) = 14% of 1000  x = 600
Solution, By the rule of allegation, we have :
Profit on 1st Profit on 2nd part
8% part 18%
Mean
profit
14%

4 6
st nd
Ratio of 1 and 2 parts = 4 : 6 = 2 : 3
3 
 Quantity of 2nd kind =   1000kg = 600 kg
 5 
e¨vL¨v: cÖ_‡g Zv‡`i wgkÖ‡Yi ev wewµi AbycvZ †ei Kiv n‡q‡Q| A_©vr †Kvb ai‡Yi wPwb KZ Abycv‡Z wewµ Ki‡j ‡gv‡Ui
Dci jvf 14% n‡e| Zv †ei Kivi ci †gvU 1000 †KwR †_‡K 18% jv‡f wewµ Kiv wPwbi cwigvY †ei Kiv n‡q‡Q|

10. In a certain sock drawer, there are 4 pairs of black socks, 3 pairs of gray socks and 2
pairs of orange socks. If socks are removed at random without replacement, what is the
minimum number of socks that must be removed" in order to ensure that two socks of
the same color have been removed?(GKwU wbw`©ó Wªqv‡i 4 †Rvov Kv‡jv †gvRv , 3 †Rvov aymi Ges 2
†Rvov Kgjv i‡½i †gvRv Av‡Q| H Wªqvi †_‡K †gvRv ˆ`efv‡e D‡Ëvjb K‡i Zv cyYivq bv ivL‡j GKB is‡qi †gvRv
D‡Ëvjb Kivi Rb¨ me©wb¤œ KZwU ‡gvRv D‡Ëvjb Ki‡Z n‡e?) [DBBL (AO)-2017]& [City Bank (MTO)-2018]
a. 11 b. 4 c. 3 d. 7 Ans: b
Solution:
‡`qv Av‡Q, Black ‡gvRv = 8 wU
Gray ‡gvRv = 6 wU
Ges Orange ‡gvRv = 4 wU ( †gvU †gvRv 3 is‡qi ev 3 iK‡gi|)
cÖ_g wZbev‡i wZbwU wfbœ wfbœ is‡qi †gvRvB Avm‡Z cv‡i| ZvB PZz_©ev‡i †gvRv D‡Ëvjb Ki‡j Av‡Mi 3is‡qi †gvRvi
†h †Kvb GK is‡qi †gvRv wiwcU n‡e| GLv‡b 2 evi †gvRv Zzj‡jI GKB is‡qi †gvRv Avm‡Z cv‡i wKš‘ †mUv wbwðZ
bq| Z‡e 4_© evi †gvRv †Zvjvi ci GUv wbwðZ‡h H is‡qi GKwU †gvRv Av‡Mi 3 ev‡ii †h †Kvb GKevi †Zvjv n‡q‡Q|
†h‡Kv‡bv GK RvZxq is‡qi †gvRv D‡Ëvjb Kiv n‡e| ZvB Kgc‡ÿ PviwU †gvRv D‡Ëvjb Ki‡Z n‡e|

Khairul’s IBA Math Book - 81

www.bdniyog.com
11. In a class, average mark in an exam is 70. The average of students who scored below 60
is 50. The average of students who scored 60 or more is 75. If the total number of
students in this class is 20, how many students scored 60 or more?( GKwU †kªwYi Qv·`i
cixÿvq cÖvß Mo b¤^I 70, hviv 60 Gi Kg cvIqv QvÎ msL¨v n‡”Q 50 Rb| 60 wKsev Gi †P‡q †ewk b¤^i †c‡q‡Q
Giæc QvÎ msL¨v n‡jv 75 Rb| †gvU QvÎ msL¨v 20 Rb n‡j, KZ Rb QvÎ 60 wKsev Gi †P‡q †ewk †c‡q‡Q?)
[DBBL (AO)-2017] & [City Bank (MTO)-2018]
a. 4 b. 8 c. 10 d. 16 Ans: d
Solution:
Total numbers = 7020 = 1400
Let, the number of students scored 60 or more = x
So who scored below 60 = 20-x
ATQ,
50(20-x) +75x = 1400 (50 M‡o cÖvß †gvU b¤^i + 75 M‡o cÖvß †gvU b¤^i = me©‡gvU 1400 b¤^i)
1000-50x + 75x = 1400
25x = 400
x = 16 Ans: 16
myZivs 60 ev Gi †_‡K †ewk b¤^i cvIqv QvÎ-QvÎx‡`i msL¨v 16 Rb|

 Nve‡o hv‡eb bv:


GB cÖ‡kœ Below 60 Ges 60 or more e‡j †h msL¨v `ywU Av‡Q G¸‡jv †`‡L Nve‡o hvIqv hv‡e bv| eis g~j
msL¨v¸‡jv‡K ¸iZ¡ w`‡q AsK ïiæ Ki‡Z n‡e| cÖ‡kœi mgvav‡b 60 wKš‘ †Kvb Kv‡R jvM‡Q bv | eo 60 Gi †_‡K Kg ev
60 Gi †ewk bv‡g 2 `j QvÎ-QvÎx †ei n‡q‡Q hv‡`i Mo 50 I 75 B n‡”Q Avmj welq| ZvB 50 Ges 75 †K ¸iæZ¡
w`b| Ges GLvb †_‡K GB welq¸‡jv wk‡L ivLyb hv‡Z Ab¨ cÖ‡kœI Kv‡R jvMv‡Z cv‡ib|

Avevi Bank Math Gi cÖkœ¸‡jv †h K¬z ¸‡jv ïiæ‡Z w`‡j mgvavb Kiv mnR nq †m¸‡jv †k‡l †`qv _v‡K| ZvB
AsK †kl ch©šÍ co‡Z n‡e| GB cÖ‡kœ K¬v‡mi †gvU 20 Rb QvÎ-QvÎxi K_v ïiæ‡Z †`qv _vK‡j AsKUv mgvavb mnR
n‡Zv| †h‡nZz AwaKvsk cÖ‡kœB Gfv‡e †k‡l cÖ‡qvRbxq K¬z †`qv _v‡K ZvB †mfv‡eB wb‡R‡K ‰Zix Kivi civgk© iB‡jv|

12. Two tanks A and B are filled with petrol. Tank A holds 600 liters more than tank B. If
100 liters of petrol were taken out from each tank, tank A would then contain 3 times as
much petrol as tank B. What is the total number of liters of fuel in the two full tanks?
A_©: A I B `yBwU U¨vsK †RU R¡vjvbx w`‡q cy‡ivcywi c~Y© Av‡Q| U¨vsK A U¨vsK B "Gi †P‡q 6,00 M¨vjb †ewk aviY
Ki‡Z cv‡i| hw` cv¤ú K‡i `ywU U¨vsK †_‡KB 1,00 M¨vjb K‡i R¡vjvbx †ei K‡i †`Iqv n‡Zv, Zvn‡j U¨vsK A †Z,
U¨vsK B Gi †P‡q wZb¸Y R¡vjvbx _vKZ| cy‡ivcywi c~Y© `ywU U¨vs‡K me©‡gvU KZ M¨vjb R¡vjvbx aviY Ki‡Z cvi‡e?
[DBBL (AO)-2017] [City Bank (MTO)-2018]
a. 1400 b. 1200 c. 1000 d. 900 Ans: a
Solution:
awi, Tank B ‡Z Av‡Q x wjUvi
 Tank A ‡Z Av‡Q (x+600) ,,

cÖkœg‡Z,
x + 600 -100= 3(x-100) (`ywU U¨vsK †_‡KB 100 K‡i we‡qvM Kivi ci hv _v‡K Zv‡Z )
x + 500 = 3x-300
Khairul’s IBA Math Book - 82

www.bdniyog.com
 2x = 800 x = 400
 A and B ‡Z petrol a‡i ‡gvU = 400 + 400 +600 = 1,400 wjUvi|

13. How many positive integers less than 20 are equal to the sum of a positive multiple of 3
and a positive multiple of 4 ? (3 Ges 4 Gi abvZ¥K ¸wYZK ¸‡jvi †hvMdj 20 Gi †_‡K †QvU †h †Kvb c~Y©
msL¨v n‡e, Giæc KZ¸‡jv c~Y©msL¨v Av‡Q?) [City Bank (MTO)-2018]
a.2 b. 5 c.7 d.10 Ans: d
Solution:
Positive multiple of 3 = 3,6,9,12,15,18…….. (G¸‡jv 20 Gi †_‡K eo n‡jI mgm¨v †bB| wKš‘ 20 Gi
†_‡K 1Uv GKvB eo n‡q †M‡j Zv 4 Gi ¸wYZK Gi mv‡_ †hvM Kivi Av‡MB †hvMdjwU 20 Gi †_‡K eo n‡q hv‡e, ZvB
¸wYZK¸‡jv I Aek¨B 20 Gi †_‡K †QvU n‡Z n‡e|)

Positive multiple of 4 = 4,8,12,16,20……….


3 Gi ¸wYZK 3 Gi mv‡_ 4 Gi ¸wYZK, 4,8,12,16 †h †Kvb GKwU †hvM Ki‡j †hvMdj¸‡jv 20 Gi †_‡K †QvU c~Y©
msL¨v nq| A_©vr 3 w`‡q ˆZix n‡e 4wU msL¨v ‡hgb: (3+4=7, 3+8 = 11, 3+12 = 15 Ges 3+16 = 19) †Zgwb
fv‡e 3 Gi ¸wYZK 6 Gi mv‡_ 4 Gi ¸wYZ 4,8 I 12 †hvM K‡i 3wU, (18 wb‡j 6 Gi mv‡_ †hvM Kivi ci 20 Gi
†_‡K ‡ewk nq ZvB 18 bv wb‡q Gi AvM ch©ŠšÍ wb‡Z n‡e) †hgb: ( 6+4 = 10, 6+8 = 14 Ges 6+12 = 18)
9 w`‡q 2wU, (9+4=13, 9+8=17), 12 w`‡q 1wU (12+4=16) Ges 15 w`‡q 1wU (15+4 = 19 wKš‘ GB 19 msL¨v
Gi jvB‡b GKevi e¨eüZ n‡q‡Q ZvB GUv ev`|)

myZivs ‡gvU 20 Gi †_‡K †QvU c~Y©msL¨v 4+3+2+1 = 10wU|

14. If three metal cubes having sides of 3 cm, 4 cm, and 5cm, were melted to make a single
cube, what would be the length of the new cube ? (3 †m.wg., 4 †m.wg. Ges 5 †m.wg. evû wewkó 3wU
†jvnvi NbK‡K Mwj‡q 1wU byZb eo NbK evbv‡bv n‡j eo NbKwUi GK evûi ˆ`N¨© KZ n‡e?) [City Bank (MTO)18]
a.7.5cm b. 6.5cm c. 6cm d.7cm Ans: c
Solution: 5
Volume of small cube = 33 = 27 4 4 4 5
3
Volume of medium cube = 4 = 64 4
and Volume of large cube = 53 = 125 5

Total volume of the new cube = 27+64+125 = 216


New
one side of new cube a = 3
216 [Since a3 =216 Cube

=3 666 = 6

15. Two items have been sold, each for Tk 3600. on one 20% profit has been earned and on
the other 20% loss has been incurred, what is the total profit or loss? (3600 UvKv K‡i `ywU
`ªe¨ weµq Kiv n‡q‡Q| GKwU 20% jv‡f Ges AciwU 20% ‡jvKmv‡b weµq Kiv n‡q‡Q| me wgwj‡q KZ †jvKmvb
n‡q‡Q? ) [City Bank (MTO)-2018]
a. None b. loss of Tk120 c. Loss of Tk 300 d. Profit of Tk 1 Ans:c
Khairul’s IBA Math Book - 83

www.bdniyog.com
Solution:
Total sale = 36002 = 7200
3600 3600
total cost=   3000  4500  7500Tk So loss=7500-7200 = 300 Tk
1.20 0.80
16. Baki lives 2 miles west of Raju’s house. Rafiq lives 3 miles north of Raju’s house and 2
miles west of Diba’s house. What is the straight-line distance from Baki’s house to
Diba’s house? (evwK, ivRyi evoxi 2 gvBj cwð‡g _v‡K, iwdK, ivRyi evoxi 3 gvBj `wÿ‡Y Ges w`evi evoxi 2
gvBj cwð‡g _v‡K| evwKi evox †_‡K w`evi evoxi mivmwi `~iZ¡ KZ?) [City Bank (MTO)-2018]
a. 5 miles b. 6 miles c. 7 miles d. 10 miles Ans: a
Solution:
cv‡ki wPÎwU †`Lyb:
cÖ_‡g, ivRyi evox †_‡K cwð‡g 2 gvBj `~‡i evKx Raf Dib
2
Ges ivRyi Dc‡i 3gvBj Dˇi iwdK Avevi iwdK m D
†h‡nZz w`evi 2 gvBj cwð‡g ZvB w`ev iwd‡Ki 2 3 5
gvBj c~‡e©| GLb wb‡P evKxi evox †_‡K mivmwi w`evi m m 3 c
evoxi `~iZ¡ †ei Kivi Rb¨ w`evi evoxi wVK wbP B 2 m
m
eiveiv †iLv †U‡b Dcwi j¤^ AvuK‡j mg‡KvYx wÎfzR `
2 Raju
n‡e hvi fzwg 2+2 = 4 Ges j¤^ 3 Zvn‡j AwZf~R m (ïiæ)
2
n‡e 42+32 = 16+9 = 25 Gi eM©g~j = 5 +
17. If the average of 7,13 p and q is 17. What is the average of (p+11) and (q-9)? (hw` 7,13, p
Ges q Gi Mo 17 nq Zvn‡j (p+11) Ges (q-9) Gi Mo KZ?) [City Bank (MTO)-2018]
a. 50 b. 25 c. 21 d. 14 Ans: b
Solution:
Sum of 7+13+p+q = 174
or, p+q = 68-20 p+q = 48
Sum of (p+11) and (q-9) = p+11+q-9 = p+q+2 = 48+2 = 50 (since p+q = 48)
50
So average of (p+11) and (q-9) = = 25
2
18. M is a girl and has the same number of brothers as sisters. N is a boy and has twice as
many sisters as brothers. M and N are the children of Q. How many children does Q
have ? (M GKRb †g‡q, Zvi mgvb msL¨K fvB Ges †evb Av‡Q| Avevi N GKRb †Q‡j hvi fvB‡qi msL¨v Zvi
†ev‡bi msL¨vi wظY| hw` M Ges N Df‡q Q Gi mšÍvb nq Zvn‡j Q Gi me©‡gvU KZRb mšÍvb Av‡Q?) [City Bank
(MTO)-2018]
a. 2 b. 3 c. 5 d. 7 Ans: d
Solution:
G ai‡Yi Analytical ability Gi cÖkœ Ackb a‡i KivB †eUvi|
Z‡e Gfv‡e gv_v LvuUv‡j Kg †f‡eB DËi †ei n‡e| †h‡nZz M GKRb †g‡q Ges Zvi mgvb msL¨K fvB I †evb Av‡Q
Zvn‡j †m 1 Rb mn Zviv me fvB †evb wg‡j †gvU msL¨v †e‡Rvo n‡e|
Avevi N (‡Q‡j) Gi †ev‡bi msL¨v fvB‡qi msL¨vi wظY †`‡L †evSv hv‡”Q fvB 2,3 Rb n‡e bv eis GKUz †ewk eo
msL¨v n‡e |

Khairul’s IBA Math Book - 84

www.bdniyog.com
Q Gi mšÍvb7 Rb ai‡j, M+N = 2 R †g‡q I †Q‡j|
cÖ_g kZ© Abymv‡i M Gi 3 fvB (N mn) Ges mgvb mgvb 3 †evb Zvn‡j †m mn †gvU 1+3+3 = 7 Rb| †hLv‡b
3Rb †Q‡j ( N mn) I †m mn 4 Rb †g‡q (M) Av‡Q|
Avevi 2q kZ©: N Gi fvB 2 Rb Ges †evb Gi msL¨v Zvi fvB‡qi msL¨vi wظY A_©vr 2 = 4 Rb|
me kZ© wg‡j hvIqvq GB 7 Rb B DËi|

19. In a legislative body of 200 people, the number of democrats is 50 less than 4 times the
number of Republicans. If one fifth of the legislators are neither Republican nor
Democrat, how many of the legislators are Republicans? (GKwU AvBb cÖ‡qvMKvix KwgwU‡Z 200
Rb †jvK Av‡Q | hv‡`i g‡a¨ Democrates ‡`i msL¨v Republicans ‡`i msL¨vi 4 ¸‡Yi †_‡K 50 Kg| hw` GK
cÂgvsk †jvK Republians ev Democrats ‡Kvb `‡ji m`m¨B bv n‡q _v‡K Zvn‡j Zv‡`i g‡a¨ Republicans
KZRb? ) [City Bank (MTO)-2018]
a. 42 b. 50 c. 71 d. 95 Ans: a
Solution:
1
Neither Republicans nor Democrats = 200 = 40
5
So, either Republicans or Democrats = 200-40 = 160

Let, Republicans = x
So, Democrats = 4x-50
ATQ,
x+4x-50 = 160
or, 5x = 210
 x = 42
cÖgvb: Republicans = 42 n‡j Democrats n‡e 442-50 = 168-50 = 118 Rb|
GLb Republicans + Democrats = 42+118 = 160 Rb| Dfq kZ©B wg‡j †M‡Q|

20. Arafat has Tk. 420. He purchased fifty mangoes and thirty orange with the whole
amount. He then chooses to return six mangoes for nine oranges as both quantities are
equally priced. What is the price of each mango? (AvivdvZ Zvi Kv‡Q _vKv 420 UvKv w`‡q 50wU Avg
Ges 30wU Kgjv‡jey µq Ki‡jv| Gici †m 6wU Av‡gi cwie‡Z© 9wU Kgjv‡jey wb‡Z PvB‡jv ‡hLv‡b 6wU Avg I 9wU
Kgjv‡jeyi `vg mgvb| cÖwZwU Av‡gi g~j¨ KZ? ) [City Bank (MTO)-2018]
a. 4.50 b. 5.0 c. 5.50 d. 6.0 Ans: d
Solution:
‡h‡nZz Av‡gi `vg †ei Ki‡Z ejv n‡q‡Q ZvB ïiæi Avg+Kgjv‡jey Gi g‡a¨ Kgjv‡jey ¸‡jv‡K Av‡g cwieZ©b Ki‡Z
n‡e| wKfv‡e Kiv hv‡e? †k‡l 9wU Kgjv‡jey = 6wU Avg Gi K¬z †_‡K|
6 6  30
9 oranges = 6 mangoes, 1 orange = mangos 30 oranges = = 20 mangos
9 9
Now, Price of 50mangoes+30 oranges = 50mangoes+20 mangoes = 70 mangoes = Tk. 420
420
So, price of each mango = 6
70
Khairul’s IBA Math Book - 85

www.bdniyog.com
21. P starts jogging form point X to point Y. 30 minutes later his friend R who jogs 1 km/hr
slower than twice P’s rate starts from the same point and follows the same path, if R
overtakes P in 2 hours, how many kilometers will R have covered? (P, c‡q›U X ‡_‡K c‡q›U
Y Gi w`‡K ‡`Šo ïiæ Kivi 30 wgwbU ci Zvi eÜz R GKB c‡_ GKB w`‡K ‡`Šov‡Z ïiæ Kij| hw` R Gi MwZ‡eM P
Gi MwZ‡e‡Mi wظ‡Yi ‡_‡K 1 Kg nq, Ges R, 2 NÈv ci P ‡K AwZµg K‡i, Zvn‡j R KZUzKz c_ wM‡qwQj?)
(Union Bank. MTO.-2015) & [City Bank (MTO)-2018]
a. 2.2km b. 3.33km c. 4km d. 6km Ans: b
Solution:
awi,
P Gi MwZ‡eM = x wKwg
myZivs R Gi MwZ‡eM = 2x -1 wKwg| ( cÖ‡kœi gv‡S †`qv Av‡Q wظ‡Yi †_‡K 1 Kg)
P Gi 2.5NÈvq (2 NÈv + ïiæi 30wg) hvIqv c_ = R Gi 2 NÈvq hvIqv c_|

cÖkœg‡Z
2 4
2.5x = 2(2x-1) 2.5x = 4x-2 1.5x = 2 x = x =
1.5 3
4 10
Zvn‡j 2 NÈvq R Gi AwZµvšÍ c_ = 2(2x-1) = 2(2 -1) =  3.33
3 3
22. In a class, 120 students are male and 100 students are female. 25% of the male students
and 20% of the female students are engineering students. 20% of the male engineering
students and 25% of the female engineering students passed the final exam. What
percentage of engineering students passed the exam ? (GKwU †kÖYxK‡ÿ 120 Rb QvÎ Ges 100 Rb
QvÎx Av‡Q| G‡`i g‡a¨ 25% QvÎ Ges 20% QvÎx BwÄwbqvwis G c‡o| 20% BwÄwbqvi QvÎ Ges 25% BwÄwbqvi QvÎx
evwl©K cixÿvq cvk Ki‡jv| BwÄwbqvi QvÎ-QvÎx‡`i g‡a¨ kZKiv KZRb cvk Ki‡jv? ) [City Bank (MTO)-2018]
a. 5% b. 10% c. 16% d. 22% Ans: d
Solution:
Male engineer students = 25% of 120 = 30
and Female engineer students = 20% of 100 =20
Total engineer students = 30+20 = 50
Again,
Male engineer students passed = 20% of 30 = 6
Female engineer students passed = 25% of 20 = 5
Total passed (M+F) = 6+5 = 11
11  100
Required % =  22%
50
 Confusion Clear: wb‡P 50 †Kb?? KviY DËi †ei Ki‡Z ejv n‡q‡Q BwÄwbqvi QvÎ-QvÎx‡`i g‡a¨ kZKiv
KZRb cvk? Avi BwÄwbqvi QvÎ-QvÎx‡`i msL¨v = 50 Rb| hw` ejv n‡Zv me©‡gvU QvÎ-QvÎx‡`i kZKiv KZRb cvk
K‡i‡Q? ZLb wb‡P 120+100 = 220 wj‡L wn‡me Kiv n‡Zv| )

Khairul’s IBA Math Book - 86

www.bdniyog.com
23. If the integer A is multiple of 6, the integer B is a multiple of 3, and A  B  0, then A2-
B2 must be multiple of which of the following integers? (hw` GKwU c~Y© msL¨v A, 6 Gi ¸wYZK nq
Ges Aci GKwU c~Y© msL¨v B , Gi ¸wYZK nq, Avevi A  B  0 n‡j A2-B2 Aek¨B wb‡Pi †Kvb msL¨vwUi
¸wYZK? ) [City Bank-(MTO)-2018]
a. 6 b. 8 c. 9 d. 10 Ans: c
Solution:
A  B  0 †_‡K †evSv hv‡”Q A Ges B DfqB abvZ¥K msL¨v †hLv‡b A, B Gi †_‡K eo|
6 Gi ¸wYZK = 6 A_ev 12 †h †Kvb msL¨v †bB | GLb 3 Gi ¸wYZK 3,6,9,12,15 ‡_‡K cÖ_g kZ© wgj †i‡L 3
A_ev 9 †bqv hvq| A_©vr A = 6 wb‡j B = 3 Ges A = 12 wb‡j B = 9 wb‡Z n‡e|(KviY A=B ‡bqv hv‡e bv eis
†QvU wb‡Z n‡e)
GLb, A2-B2 = 62-32 = 36-9 = 27
A_ev, A2-B2 = 122-92 = 144-81 = 63
‡`Lv hv‡”Q 27 Ges 63 Dfq msL¨vB 9 Gi ¸wYZK ev 9 Øviv wefvR¨| myZivs DËi c. 9
Ans: 9

24. Six years ago, Rahim was P times as old as Kalam was. If Rahim is now 17 years old,
how old is Kalam now in terms of P? (6 eQi Av‡M iwn‡gi eqm Kvjv‡gi eq‡mi †_‡K P ¸Y †ewk wQj|
hw` iwn‡gi eZ©gvb eqm 17 eQi nq, Zvn‡j P Gi wfwˇZ Kvjv‡gi eqm KZ?) [City Bank-(MTO)-2018]
11 P 17  P 17
a. +6 b. +6 c. d. Ans:a
P 11 6 P
Solution:
6 eQi Av‡M iwn‡gi eqm wQj 17-6 =11 eQi| iwn‡gi eqm 11 eQi wQj ZLb Kvjv‡gi †_‡K iwng P ¸Y eo _vKvq
11 11
H mg‡q (6 eQi Av‡M) Kvjv‡gi eqm wQj Zvn‡j Kvjv‡gi eZ©gvb eqm +6 |
P P

==================================

Khairul’s IBA Math Book - 87

www.bdniyog.com
www.bdniyog.com

You might also like